Extensions and refinements of Jensen’s inequalities [Volume 4]

University of Ploiesti, 2015. - 219 c.Vasile Cîrtoaje is a Professor at the Department of Automatic Control and Computer

600 38 815KB

English Pages 0 [219] Year 2015

Report DMCA / Copyright

DOWNLOAD FILE

Polecaj historie

Extensions and refinements of Jensen’s inequalities [Volume 4]

  • Commentary
  • 1854247
Citation preview

Vasile Cîrtoaje C j

MATHEM MATICAL INEQUA Q ALITIES Volume 4

EXTENSIONS AND REFINEMENTS OF JENSEN’SS INEQUALITY OF JENSEN

UNIVERSITY OF PLOIESTI, ROMANIA 2015

Contents 1 Half Convex Function Method 1 1.1 Theoretical Basis . . . . . . . . . . . . . . . . . . . . . . . . . . . . . . . . . . . 1 1.2 Applications . . . . . . . . . . . . . . . . . . . . . . . . . . . . . . . . . . . . . . 7 1.3 Solutions . . . . . . . . . . . . . . . . . . . . . . . . . . . . . . . . . . . . . . . . 17 2 Half Convex Function Method for Ordered Variables 97 2.1 Theoretical Basis . . . . . . . . . . . . . . . . . . . . . . . . . . . . . . . . . . . 97 2.2 Applications . . . . . . . . . . . . . . . . . . . . . . . . . . . . . . . . . . . . . . 103 2.3 Solutions . . . . . . . . . . . . . . . . . . . . . . . . . . . . . . . . . . . . . . . . 109 3 Partially Convex Function Method 141 3.1 Theoretical Basis . . . . . . . . . . . . . . . . . . . . . . . . . . . . . . . . . . . 141 3.2 Applications . . . . . . . . . . . . . . . . . . . . . . . . . . . . . . . . . . . . . . 145 3.3 Solutions . . . . . . . . . . . . . . . . . . . . . . . . . . . . . . . . . . . . . . . . 151 4 Partially Convex Function Method for Ordered Variables 193 4.1 Theoretical Basis . . . . . . . . . . . . . . . . . . . . . . . . . . . . . . . . . . . 193 4.2 Applications . . . . . . . . . . . . . . . . . . . . . . . . . . . . . . . . . . . . . . 197 4.3 Solutions . . . . . . . . . . . . . . . . . . . . . . . . . . . . . . . . . . . . . . . . 201 5 Bibliography

215

i

ii

Vasile Cîrtoaje

Chapter 1

Half Convex Function Method 1.1

Theoretical Basis

Half Convex Function Theorem (Vasile Cirtoaje, 2004). Let f (u) be a function defined on a real interval I and convex on Iu≥s or Iu≤s , where s ∈ I. The inequality f (a1 ) + f (a2 ) + · · · + f (an ) ≥ n f

a + a + ··· + a  1 2 n n

holds for all a1 , a2 , . . . , an ∈ I satisfying a1 + a2 + · · · + an = ns if and only if f (x) + (n − 1) f ( y) ≥ n f (s) for all x, y ∈ I such that x + (n − 1) y = ns. Proof (for the right convexity). The necessity is obvious. Without loss of generality, assume that a1 ≤ a2 ≤ · · · ≤ an . If a1 ≥ s, then the required inequality follows from Jensen’s inequality for convex functions. Otherwise, there exists k ∈ {1, 2, . . . , n − 1} such that a1 ≤ · · · ≤ ak < s ≤ ak+1 ≤ · · · ≤ an . Since f is convex on Iu≥s , we can apply Jensen’s inequality to get f (ak+1 ) + · · · + f (an ) ≥ (n − k) f (z),

z=

ak+1 + · · · + an . n−k

Thus, it suffices to show that f (a1 ) + · · · + f (ak ) + (n − k) f (z) ≥ n f (s). Let b1 , . . . , bk ∈ I defined by ai + (n − 1)bi = ns, 1

i = 1, . . . , k.

2

Vasile Cîrtoaje

We claim that z ≥ b1 ≥ · · · ≥ bk > s. Indeed, we have b1 ≥ · · · ≥ bk , bk − s =

s − ak > 0, n−1

(n − 1)b1 = ns − a1 = (a2 + · · · + ak ) + ak+1 + · · · + an ≤ (k − 1)s + ak+1 + · · · + an = (k − 1)s + (n − k)z ≤ (n − 1)z. By hypothesis, we have f (a1 ) + (n − 1) f (b1 ) ≥ n f (s), ··· f (ak ) + (n − 1) f (bk ) ≥ n f (s), hence f (a1 ) + · · · + f (ak ) + (n − 1)[ f (b1 ) + · · · + f (bk )] ≥ kn f (s). Consequently, it suffices to show that kn f (s) − (n − 1)[ f (b1 ) + · · · + f (bk )] + (n − k) f (z) ≥ n f (s), which is equivalent to p f (z) + (k − p) f (s) ≥ f (b1 ) + · · · + f (bk ),

p=

n−k ≤ 1. n−1

By Jensen’s inequality, we have p f (z) + (1 − p) f (s) ≥ f (w),

w = pz + (1 − p)s ≥ s.

Thus, we only need to show that f (w) + (k − 1) f (s) ≥ f (b1 ) + · · · + f (bk ). Since the decreasingly ordered vector A~k = (w, s, . . . , s) majorizes the decreasingly ordered vector B~k = (b1 , b2 , . . . , bk ), this inequality follows from Karamata’s inequality for convex functions. The proof for the left convexity is similar. Remark 1. Let us denote g(u) =

f (u) − f (s) , u−s

h(x, y) =

g(x) − g( y) . x−y

Half Convex Function Method

3

In many applications, it is useful to replace the hypothesis f (x) + (n − 1) f ( y) ≥ n f (s) in Half Convex Function Theorem (HCF Theorem) by the equivalent condition h(x, y) ≥ 0 for all x, y ∈ I such that x + (n − 1) y = ns. This equivalence is true since f (x) + (n − 1) f ( y) − n f (s) = [ f (x) − f (s)] + (n − 1)[ f ( y) − f (s)] = (x − s)g(x) + (n − 1)( y − s)g( y) n−1 (x − y)[g(x) − g( y)] = n n−1 = (x − y)2 h(x, y). n Remark 2. Assume that f is differentiable on I, and let H(x, y) =

f 0 (x) − f 0 ( y) . x−y

Then, the desired inequality of Jensen’s type in HCF Theorem holds true by replacing the hypothesis f (x) + (n − 1) f ( y) ≥ n f (s) with the more restrictive condition H(x, y) ≥ 0 for all x, y ∈ I such that x + (n − 1) y = ns. To prove this claim, we will show that the new condition implies f (x)+(n−1) f ( y) ≥ n f (s) for all x, y ∈ I such that x + (n − 1) y = ns. Write this inequality as f1 (x) ≥ n f (s), where  ns − x  f1 (x) = f (x) + (n − 1) f ( y) = f (x) + (n − 1) f . n−1 From  ns − x  n = f 0 (x) − f 0 ( y) = (x − s)H(x, y), f10 (x) = f 0 (x) − f 0 n−1 n−1 it follows that f1 is decreasing for x ≤ s and increasing for x ≥ s; therefore, f1 (x) ≥ f1 (s) = n f (s). Remark 3. The inequality in HCF Theorem becomes an equality for a1 = a2 = · · · = an = s and also for a1 = x,

a2 = · · · = an = y

4

Vasile Cîrtoaje

(or any cyclic permutation), where x, y ∈ I satisfy the equations x + (n − 1) y = ns,

f (x) + (n − 1) f ( y) = n f (s).

For x 6= y, these equations are equivalent to x + (n − 1) y = ns, h(x, y) = 0. Remark 4. HCF Theorem is also valid in the case when I = [a, b] \ {u0 } or I = (a, b) \ {u0 }, where a, b, u0 are real numbers such that a < u0 < b. Clearly, two cases are possible: (1) u0 < s - when f is right convex, i.e. convex on Iu≥s ; (2) u0 > s - when f is left convex, i.e. convex on Iu≤s . Remark 5. In HCF Theorem for the right convexity (when HCF Theorem is called RHCF Theorem), it suffices to consider x ≤ s ≤ y. This claim is true because in the proof of RHCF Theorem, the hypothesis f (x) + (n − 1) f ( y) ≥ n f (s) is used to get the inequalities f (ai ) + (n − 1) f (bi ) ≥ n f (s),

i = 1, 2, · · · , k,

where ai ≤ s ≤ bi . Similarly, in HCF Theorem for the left convexity (when HCF Theorem is called LHCF Theorem), it suffices to consider x ≥ s ≥ y. The following theorem (LCRCF-Theorem) is also useful to prove some symmetric inequalities. Left Convex-Right Concave Function Theorem (Vasile Cirtoaje, 2004). Let a ≤ c be real numbers, let f be a continuous function on I = [a, ∞), strictly convex on [a, c] and strictly concave on [c, ∞), and let E(a1 , a2 , . . . , an ) = f (a1 ) + f (a2 ) + · · · + f (an ). If a1 , a2 , . . . , an ∈ I such that a1 + a2 + · · · + an = S = const ant, then (a) E is minimum for a1 = a2 = · · · = an−1 ≤ an ;

Half Convex Function Method

5

(b) E is maximum for either a1 = a or a < a1 ≤ a2 = · · · = an . Proof. Without loss of generality, assume that a1 ≤ a2 ≤ . . . ≤ an . Since E(a1 , a2 , . . . , an ) is a continuous function on the compact set Λ = {(a1 , a2 , . . . , an ) : a1 + a2 + · · · + an = S, a1 , a2 , . . . , an ∈ I}, E attains its minimum and maximum values. (a) For the sake of contradiction, suppose that E is minimum at a point (b1 , b2 , . . . , bn ) with b1 ≤ b2 ≤ . . . ≤ bn and b1 < bn−1 . For bn−1 ≤ c, by Jensen’s inequality for strictly convex functions, we have ‹  b1 + bn−1 , f (b1 ) + f (bn−1 ) > 2 f 2 while for bn−1 > c, by Karamata’s inequality for strictly concave functions, we have f (bn−1 ) + f (bn ) > f (c) + f (bn−1 + bn − c). The both results contradict the assumption that E is minimum at (b1 , b2 , . . . , bn ). (b) For the sake of contradiction, suppose that E is maximum at a point (b1 , b2 , . . . , bn ) with a < b1 ≤ b2 ≤ . . . ≤ bn and b2 < bn . There are three cases to consider. Case 1: b2 ≥ c. By Jensen’s inequality for strictly concave functions, we have  ‹ b2 + bn f (b2 ) + f (bn ) < 2 f . 2 Case 2: b2 < c and b1 + b2 −a ≤ c. By Karamata’s inequality for strictly convex functions, we have f (b1 ) + f (b2 ) < f (a) + f (b1 + b2 − a). Case 3: b2 < c and b1 + b2 −c ≥ a. By Karamata’s inequality for strictly convex functions, we have f (b1 ) + f (b2 ) < f (b1 + b2 − c) + f (c). Clearly, the result of each case contradicts the assumption that E is msximum at (b1 , b2 , . . . , bn ). Remark 6. The part (a) in LCRCF-Theorem is also true in the case where I = (a, ∞) and lim x→a f (x) = ∞.

6

Vasile Cîrtoaje

Half Convex Function Method

1.2

7

Applications

1.1. If a, b, c are real numbers such that a + b + c = 3, then 3(a4 + b4 + c 4 ) + a2 + b2 + c 2 + 6 ≥ 6(a3 + b3 + c 3 ).

1.2. If a1 , a2 , . . . , an ≥

1 − 2n such that a1 + a2 + · · · + an = n, then n−2 a13 + a23 + · · · + an3 ≥ n.

1.3. If a1 , a2 , . . . , an ≥

−n such that a1 + a2 + · · · + an = n, then n−2 a13 + a23 + · · · + an3 ≥ a12 + a22 + · · · + an2 .

1.4. If a1 , a2 , . . . , an are real numbers such that a1 + a2 + · · · + an = n, then (n2 − 3n + 3)(a14 + a24 + · · · + an4 − n) ≥ 2(n2 − n + 1)(a12 + a22 + · · · + an2 − n).

1.5. If a1 , a2 , . . . , an are nonnegative real numbers such that a1 + a2 + · · · + an = n, then (n2 + n + 1)(a13 + a23 + · · · + an3 − n) ≥ (n + 1)(a14 + a24 + · · · + an4 − n).

1.6. If a, b, c are real numbers such that a + b + c = 3, then (a) (b)

p a4 + b4 + c 4 − 3 + 2(7 + 3 7)(a3 + b3 + c 3 − 3) ≥ 0; p a4 + b4 + c 4 − 3 + 2(7 − 3 7)(a3 + b3 + c 3 − 3) ≥ 0.

1.7. Let k ≥ 3 and n ≥ 3 be integer numbers such that k ≤ n + 1. If a1 , a2 , . . . , an are nonnegative real numbers such that a1 + a2 + · · · + an = n, then a1k + a2k + · · · + ank − n a12 + a22 + · · · + an2 − n

≥ (n − 1)

•

˜ n k−1 −1 . n−1

8

Vasile Cîrtoaje

1.8. Let k ≥ 3 and n ≥ 3 be integer numbers. If a1 , a2 , . . . , an are nonnegative real numbers such that a1 + a2 + · · · + an = n, then a1k + a2k + · · · + ank − n a12 + a22 + · · · + an2 − n



nk−1 − 1 . n−1

1.9. If a1 , a2 , . . . , an are positive real numbers such that a1 + a2 + · · · + an = n, then  ‹ 1 1 1 2 n + + ··· + − n ≥ 4(n − 1)(a12 + a22 + · · · + an2 − n). a1 a2 an

1.10. If a1 , a2 , . . . , a8 are positive real numbers such that a1 + a2 + · · · + a8 = n, then 1 a12

+

1 a22

+ ··· +

1 a82

≥ a12 + a22 + · · · + a82 .

1.11. If a1 , a2 , . . . , an are positive real numbers such that a12

+ a22

+ · · · + an2



p

−n≥2 1+

1 1 1 + + ··· + = n, then a1 a2 an

 n−1 (a1 + a2 + · · · + an − n). n

1.12. If a, b, c are nonnegative real numbers, no two of which are zero, then  ‹ 1 1 2 1 1 1 1 + + ≤ + + . 3a + b + c 3b + c + a 3c + a + b 5 b+c c+a a+b

1.13. If a, b, c, d ≥ 3 −

p 7 such that a + b + c + d = 4, then 1 1 1 1 4 + + + ≥ . 2 2 2 2 2+a 2+ b 2+c 2+d 3

1.14. If a1 , a2 , . . . , an ∈ [0, n − 2] such that a1 + a2 + · · · + an = n, then 1 n + a12

+

1 n + a22

+ ··· +

1 n ≤ . 2 n + an n+1

Half Convex Function Method

9

1.15. If a, b, c are nonnegative real numbers such that a + b + c = 3, then 3− b 3−c 3 3−a + + ≥ . 2 2 2 9+a 9+ b 9+c 5 1.16. If a, b, c are nonnegative real numbers such that a + b + c = 3, then 1 1 1 3 + + ≥ . 2 2 2 5+a+a 5+ b+ b 5+c+c 7 1.17. If a, b, c, d are nonnegative real numbers such that a + b + c + d = 4, then 1 1 1 1 1 + + + ≤ . 10 + a + a2 10 + b + b2 10 + c + c 2 10 + d + d 2 3 1.18. Let a1 , a2 , . . . , an be nonnegative real numbers such that a1 + a2 + · · · + an = n. If 1 k ≥ 1 − , then n 1 n 1 1 ≥ . + + ··· + 1 + kan2 1+k 1 + ka12 1 + ka22 1.19. Let a1 , a2 , . . . , an be real numbers such that a1 + a2 + · · · + an = n. If 0 < k ≤ n−1 , then 2 n −n+1 1 n 1 1 ≤ . + + ··· + 2 2 2 1 + kan 1+k 1 + ka1 1 + ka2 1.20. Let a1 , a2 , . . . , an be nonnegative numbers such that a1 + a2 + · · · + an = n. If n2 k≥ , then 4(n − 1) a1 (a1 − 1) a12

+k

+

a2 (a2 − 1) a22

+k

+ ··· +

an (an − 1) ≥ 0. an2 + k

1.21. If a1 , a2 , . . . , an are nonnegative real numbers such that a1 + a2 + · · · + an = n, then 1 − an 1 − a1 1 − a2 + + ··· + ≤ 0. 2 2 (n − 2a1 ) (n − 2a2 ) (n − 2an )2

10

Vasile Cîrtoaje

1.22. Let a1 , a2 , . . . , an be nonnegative real numbers such that a1 + a2 + · · · + an = n. If n , then k ≥1+ p n−1 a12 − 1 (a1 + k)2

+

a22 − 1 (a2 + k)2

+ ··· +

an2 − 1 (an + k)2

≥ 0.

1.23. Let a1s , a2 , . . . , an be nonnegative real numbers such that a1 + a2 + · · · + an = n. If 2n − 1 0< k ≤1+ , then n−1 a12 − 1 (a1 + k)2

+

a22 − 1 (a2 + k)2

+ ··· +

an2 − 1 (an + k)2

≤ 0.

1.24. Let a1 , a2 , . . . , an be nonnegative real numbers such that a1 + a2 + · · · + an = n. If (n − 1)(2n − 1) k≥ , then n2 1 1 + ka13

+

1 1 + ka23

+ ··· +

1 n ≥ . 3 1 + kan 1+k

1.25. Let a1 , a2 , . . . , an be nonnegative real numbers such that a1 + a2 + · · · + an = n. If n−1 , then 0 0 for u ≥ 1, hence f is convex for u ≥ 1. By HCF Theorem, it suffices to show that f (x) + 2 f ( y) ≥ 3 f (1) for all real x, y such that x + 2 y = 3. Using Remark 1, we only need to show that h(x, y) ≥ 0, where h(x, y) =

g(x) − g( y) , x−y

g(u) =

f (u) − f (1) . u−1

We have g(u) = u3 + u2 + u + 1 + p(u2 + u + 1) + u + 1 = u3 + (p + 1)(u2 + u + 1), h(x, y) = x 2 + x y + y 2 ) + (p + 1)(x + y + 1) = 3 y 2 − (10 + p) y + 13 + 4p  ‹ 10 + p 2 =3 y− ≥ 0. 6 In accordance with Remark 3, the equality holds for a = b = c = 1, and also for a = −(1 + p)/3 and b = c = (10 + p)/6 (or any cyclic permutation). p (a) Since p = 2(7 + 3 7),p the equality holds for a = b = c = 1, and also for p a = −5 − 2 7 and b = c = 4 + 7 (or any cyclic permutation). p (b) Since p p = 2(7 − 3 7), p the equality holds for a = b = c = 1, and also for a = −5 + 2 7 and b = c = 4 − 7 (or any cyclic permutation). Remark. Similarly, we can prove the following generalization:

22

Vasile Cîrtoaje • If a1 , a2 , . . . , an are real numbers such that a1 + a2 + · · · + an = n, then a14 + a24 + · · · + an4 − n + p(a12 + a22 + · · · + an2 − n) ≥ 0,

where p=

2(n2 − n + 1) ± 2

p

3(n2 − n + 1)(n2 − 3n + 3) . (n − 2)2

The equality holds for a1 = a2 = · · · = an = 1, and also for a1 =

2(n2 − n − 1) + (n − 2)p −2(n2 − 3n + 1) − (n − 1)(n − 2)p , a = a = · · · = a = 2 3 n 2(n2 − 3n + 3) 2(n2 − 3n + 3)

(or any cyclic permutation).

P 1.7. Let k ≥ 3 and (n ≥ 3) be integer numbers such that k ≤ n + 1. If a1 , a2 , . . . , an are nonnegative real numbers such that a1 + a2 + · · · + an = n, then a1k + a2k + · · · + ank − n a12 + a22 + · · · + an2 − n

≥ (n − 1)

•

˜ n k−1 −1 . n−1 (Vasile Cirtoaje, 2012)

Solution. Denote •

m = (n − 1)

˜  n k−2  n k−3 n k−1 −1 = + + · · · + 1, n−1 n−1 n−1

and write the inequality as f (a1 ) + f (a2 ) + · · · + f (an ) ≥ n f (s),

s=

a1 + a2 + · · · + an = 1, n

where f (u) = uk − mu2 ,

u ∈ [0, n].

We will show that f (u) is convex for u ∈ [1, n]. Since f 00 (u) = k(k − 1)uk−2 − 2m ≥ k(k − 1) − 2m, we need to show that k(k − 1)  n k−2  n k−3 ≥ + + · · · + 1. 2 n−1 n−1

Half Convex Function Method

23

Since n ≥ k − 1, this inequality is true if  ‹  ‹ k(k − 1) k − 1 k−2 k − 1 k−3 ≥ + + · · · + 1. 2 k−2 k−2 By Bernoulli’s inequality, we have 

k−1 k−2

‹j

=

1 1−

1 k−1

j ≤

1 1−

j k−1

=

k−1 , k− j−1

0 ≤ j ≤ k − 2.

Therefore, it suffices to show that  ‹ k(k − 1) 1 1 ≥ (k − 1) 1 + + · · · + . 2 2 k−1 This is true if

k 1 1 ≥ 1 + + ··· + , 2 2 k−1 which can be easily proved by induction. According to HCF Theorem and Remark 1, we only need to show that h(x, y) ≥ 0 for x, y ≥ 0 such that x + (n − 1) y = n, where h(x, y) =

g(x) − g( y) , x−y

g(u) =

f (u) − f (1) . u−1

We have (uk − 1) − m(u2 − 1) = (uk−1 + uk−2 + · · · + 1) − m(u + 1), u−1  k−1  x − y k−1 x k−2 − y k−1 h(x, y) = + + ··· + 1 − m x−y x−y  k−1   k−2   2  x − y k−2  n k−3 x − y2 x − y k−1  n k−2 n + +· · ·+ = − − − . x−y n−1 x−y n−1 x−y n−1 g(u) =

It suffices to show that

x j+1 − y j+1  n  j ≥ x−y n−1

n i for x = 6 y and j = 1, 2, · · · , k − 2. This is true if f j ( y) ≥ 0 for y ∈ 0, , where n−1  n j f j ( y) = x j + x j−1 y + · · · + x y j−1 + y j − , x = n − (n − 1) y. n−1 h

Since x 0 = −(n − 1) and n − 1 ≥ k − 2 ≥ j, we get f j0 ( y) = −(n − 1)[ j x j−1 + ( j − 1)x j−2 y + · · · + y j−1 ] + x j−1 + 2x j−2 y + · · · + j y j−1

24

Vasile Cîrtoaje ≤ − j[ j x j−1 + ( j − 1)x j−2 y + · · · + y j−1 ] + x j−1 + 2x j−2 y + · · · + j y j−1

= −( j · j − 1)x j−1 − [ j · ( j − 1) − 2]x j−2 y − · · · − ( j · 2 − j + 1)x y j−2 ≤ 0. n Therefore, f j ( y) is decreasing, hence f j ( y) is minimal for y = , when x = 0. Thus, n−1  n  = 0. f j ( y) ≥ f j n−1 This completes the proof. From x + (n − 1) y = n and h(x, y) = 0, we get x = 0, n y= . Therefore, in accordance with Remark 3, the equality holds for a1 = 0 and n−1 n a2 = a3 = · · · = an = (or any cyclic permutation). n−1 Remark. For k = 3 and k = 4, we get the following statements (Vasile Cirtoaje, 2002): • If a1 , a2 , . . . , an are nonnegative real numbers such that a1 + a2 + · · · + an = n, then (n − 1)(a13 + a23 + · · · + an3 − n) ≥ (2n − 1)(a12 + a22 + · · · + an2 − n), with equality for a1 = a2 = · · · = an = 1, and also for a1 = 0 and a2 = a3 = · · · = an = n (or any cyclic permutation). n−1 • If a1 , a2 , . . . , an are nonnegative real numbers such that a1 + a2 + · · · + an = n, then (n − 1)2 (a14 + a24 + · · · + an4 − n) ≥ (3n2 − 3n + 1)(a12 + a22 + · · · + an2 − n), with equality for a1 = a2 = · · · = an = 1, and also for a1 = 0 and a2 = a3 = · · · = an = n (or any cyclic permutation). n−1

P 1.8. Let k ≥ 3 and n ≥ 3 be integer numbers. If a1 , a2 , . . . , an are nonnegative real numbers such that a1 + a2 + · · · + an = n, then a1k + a2k + · · · + ank − n a12 + a22 + · · · + an2 − n



nk−1 − 1 . n−1 (Vasile Cirtoaje, 2012)

Solution. Denote m=

nk−1 − 1 = nk−2 + nk−3 + · · · + 1, n−1

and write the inequality as f (a1 ) + f (a2 ) + · · · + f (an ) ≥ n f (s),

s=

a1 + a2 + · · · + an = 1, n

Half Convex Function Method

25

where f (u) = mu2 − uk ,

u ∈ [0, n].

We will show that f (u) is convex for u ∈ [0, 1]. Since f 00 (u) = 2m − k(k − 1)uk−2 ≥ 2m − k(k − 1), we need to show that nk−2 + nk−3 + · · · + 1 ≥

k(k − 1) . 2

3k−2 + 3k−3 + · · · + 1 ≥

k(k − 1) . 2

This is true if

By Bernoulli’s inequality, we have 3k−2 + 3k−3 + · · · + 1 ≥ [1 + 2(k − 2)] + [1 + 2(k − 3)] + · · · + [1 + 2 · 0] k(k − 1) . 2 According to HCF Theorem and Remark 1, we only need to show that h(x, y) ≥ 0 for x, y ≥ 0 such that x + (n − 1) y = n, where = (k − 1) + (k − 2)(k − 1) = (k − 1)2 >

h(x, y) =

g(x) − g( y) , x−y

g(u) =

f (u) − f (1) . u−1

We have g(u) =

m(u2 − 1) − (uk − 1) = m(u + 1) − (uk−1 + uk−2 + · · · + 1), u−1

x k−1 − y k−1 x k−2 − y k−1 − − ··· − 1 x−y x−y       x k−1 − y k−1 x k−2 − y k−2 x2 − y2 k−2 k−3 = n − + n − + ··· + n − . x−y x−y x−y h(x, y) = m −

It suffices to show that nj ≥

x j+1 − y j+1 x−y

for x 6= y and j = 1, 2, · · · , k − 2. We will show that n j ≥ (x + y) j ≥

x j+1 − y j+1 . x−y

The left inequality is true since n − (x + y) = x + (n − 1) y − (x + y) = (n − 2) y ≥ 0.

26

Vasile Cîrtoaje

The right inequality is also true since  ‹  ‹ j j−1 j j j (x + y) = x + x y + ··· + x y j−1 + y j 1 j−1 and

x j+1 − y j+1 = x j + x j−1 y + · · · + x y j−1 + y j . x−y

In accordance with Remark 3, the equality holds for a1 = n and a2 = a3 = · · · = an = 0 (or any cyclic permutation). Remark. For k = 3 and k = 4, we get the following statements (Vasile Cirtoaje, 2002): • If a1 , a2 , . . . , an are nonnegative real numbers such that a1 + a2 + · · · + an = n, then a13 + a23 + · · · + an3 − n ≤ (n + 1)(a12 + a22 + · · · + an2 − n), with equality for a1 = a2 = · · · = an = 1, and also for a1 = n and a2 = a3 = · · · = an = 0 (or any cyclic permutation). • If a1 , a2 , . . . , an are nonnegative real numbers such that a1 + a2 + · · · + an = n, then a14 + a24 + · · · + an4 − n ≤ (n2 + n + 1)(a12 + a22 + · · · + an2 − n), with equality for a1 = a2 = · · · = an = 1, and also for a1 = n and a2 = a3 = · · · = an = 0 (or any cyclic permutation).

P 1.9. If a1 , a2 , . . . , an are positive real numbers such that a1 + a2 + · · · + an = n, then  ‹ 1 1 1 n2 + + ··· + − n ≥ 4(n − 1)(a12 + a22 + · · · + an2 − n). a1 a2 an (Vasile Cirtoaje, 2004) Solution. Write the inequality as f (a1 ) + f (a2 ) + · · · + f (an ) ≥ n f (s), where f (u) =

s=

a1 + a2 + · · · + an = 1, n

n2 − 4(n − 1)u2 , u ∈ (0, n). u

For u ∈ (0, 1], we have f 00 (u) =

2n2 − 8(n − 1) ≥ 2n2 − 8(n − 1) = 2(n − 2)2 ≥ 0. u3

Half Convex Function Method

27

Thus, f is convex on (0, 1]. By HCF Theorem and Remark 1, it suffices to show that h(x, y) ≥ 0 for x, y > 0 such that x + (n − 1) y = n, where h(x, y) =

g(x) − g( y) , x−y

We have g(u) = and h(x, y) =

g(u) =

f (u) − f (1) . u−1

−n2 − 4(n − 1)(u + 1) u

[n − (2n − 2) y]2 n2 − 4(n − 1) = ≥ 0. xy xy

In accordance with Remark 3, the equality holds for a1 = a2 = · · · = an = 1, and also for a1 = n/2 and a2 = a3 = · · · = an = n/(2n − 2) (or any cyclic permutation).

P 1.10. If a1 , a2 , . . . , a8 are positive real numbers such that a1 + a2 + · · · + a8 = n, then 1 a12

+

1 a22

+ ··· +

1 a82

≥ a12 + a22 + · · · + a82 . (Vasile Cirtoaje, 2007)

Solution. Write the inequality as f (a1 ) + f (a2 ) + · · · + f (a8 ) ≥ 8 f (s), where f (u) =

s=

a1 + a2 + · · · + a8 = 1, 8

1 − u2 , u ∈ (0, n). u2

For u ∈ (0, 1], we have f 00 (u) =

6 − 2 ≥ 6 − 2 > 0. u4

Thus, f is convex on (0, 1]. By HCF Theorem and Remark 1, it suffices to show that h(x, y) ≥ 0 for x, y > 0 such that x + 7 y = 8, where h(x, y) =

g(x) − g( y) , x−y

g(u) =

We have g(u) = −u − 1 −

f (u) − f (1) . u−1

1 1 − 2 u u

28

Vasile Cîrtoaje

and h(x, y) = −1 + From 8 = x + 7 y ≥ 2

p

x+y 1 + 2 2. xy x y

7x y, we get x y ≤ 16/7. Therefore, 7(x + y) 112 y 2 − 170 y + 72 1 + = xy 16x y 16x y 2 2 112 y − 176 y + 72 14 y − 22 y + 9 > = > 0. 16x y 2x y

h(x, y) ≥ −1 +

The equality holds for a1 = a2 = · · · = a8 = 1. Remark. Similarly, we can prove the following generalization. • If a1 , a2 , . . . , an (n ≥ 4) are positive real numbers such that a1 + a2 + · · · + an = n, then  1 1 1 8 2 a1 + a22 + · · · + an2 . + 2 + ··· + 2 + 8 − n ≥ 2 an n a1 a2

P 1.11. If a1 , a2 , . . . , an are positive real numbers such that a12

+ a22

+ · · · + an2



p

−n≥2 1+

1 1 1 + + ··· + = n, then a1 a2 an

 n−1 (a1 + a2 + · · · + an − n). n (Vasile Cirtoaje, 2006)

Solution. Replacing each ai by 1/ai , we need to prove that f (a1 ) + f (a2 ) + · · · + f (an ) ≥ n f (s), where

s=

a1 + a2 + · · · + an = 1, n

  p 1 n−1 1 f (u) = 2 − 2 1 + , u ∈ (0, n). u n u

For u ∈ (0, 1], we have € Š € p 6 − 4 1 + n−1 u 6−4 1+ n f 00 (u) = ≥ u4 u4

p

n−1 n

Š

p 2( n − 1 − 1)2 = ≥ 0. nu4

Thus, f is convex on (0, 1]. By HCF Theorem and Remark 1, it suffices to show that h(x, y) ≥ 0 for x, y > 0 such that x + (n − 1) y = n, where h(x, y) =

g(x) − g( y) , x−y

g(u) =

f (u) − f (1) . u−1

Half Convex Function Method We have

29

  p −1 2 n−1 1 g(u) = 2 + 1 + u n u

and 1 h(x, y) = xy We only need to show that



 p 2 n−1 1 1 + −1− . x y n

p 2 n−1 1 1 + ≥1+ . x y n

Indeed, using the Cauchy-Schwarz inequality, we get p p p (1 + n − 1)2 (1 + n − 1)2 2 n−1 1 1 + ≥ = =1+ . x y x + (n − 1) y n n In accordance p with Remark 3, the equality holds forpa1 = a2 = · · · = an = 1, and also for 1+ n−1 n−1+ n−1 a1 = and a2 = a3 = · · · = an = (or any cyclic permutation). n n

P 1.12. If a, b, c are nonnegative real numbers, no two of which are zero, then  ‹ 1 1 1 2 1 1 1 + + ≤ + + . 3a + b + c 3b + c + a 3c + a + b 5 b+c c+a a+b (Vasile Cirtoaje, 2006) Solution. Due to homogeneity, we may assume that a + b + c = 3. So, we need to show that a+b+c f (a) + f (b) + f (c) ≥ 3 f (s), s = = 1, 3 where 2 5 − , u ∈ [0, 3). f (u) = 3 − u 2u + 3 For u ∈ [1, 3), we have f 00 (u) =

4 40 36[2u3 + 3u2 + 9(u − 1)(3 − u)] − = > 0; (3 − u)3 (2u + 3)3 (3 − u)3 (2u + 3)3

therefore, f is convex on [1, 3). By HCF Theorem and Remark 1, it suffices to show that h(x, y) ≥ 0 for x, y ≥ 0 such that x + 2 y = 3, where h(x, y) =

g(x) − g( y) , x−y

g(u) =

f (u) − f (1) . u−1

30

Vasile Cîrtoaje

We have g(u) =

2 1 + 3 − u 2u + 3

and 9(2x + 2 y − 3) 1 2 − = (3 − x)(3 − y) (2x + 3)(2 y + 3) (3 − x)(3 − y)(2x + 3)(2 y + 3) 9x ≥ 0. = (3 − x)(3 − y)(2x + 3)(2 y + 3)

h(x, y) =

The equality holds for a = b = c, and also for a = 0 and b = c (or any cyclic permutation).

P 1.13. If a, b, c, d ≥ 3 −

p 7 such that a + b + c + d = 4, then

1 1 1 1 4 + + + ≥ . 2 2 2 2 2+a 2+ b 2+c 2+d 3 (Vasile Cirtoaje, 2008) Solution. Write the inequality as f (a) + f (b) + f (c) + f (d) ≥ 4 f (s), where f (u) =

s=

a+b+c+d = 1, 4

p 1 , u ≥ 3 − 7. 2 2+u

For u ≥ 1, we have f 00 (u) =

3(3u2 − 2) > 0. (2 + u2 )3

Thus, f is convex for u ≥ 1. By HCF Theorem and Remark 1, it suffices to show that h(x, y) ≥ 0 for x, y ≥ 0 such that x + 3 y = 4. We have g(u) = and h(x, y) = since

f (u) − f (1) −1 − u = u−1 3(2 + u2 )

xy + x + y −2 g(x) − g( y) = ≥0 x−y 3(2 + x 2 )(2 + y 2 )

−x 2 + 6x − 2 (3 + xy + x + y −2= = 3

p p 7 − x)(x − 3 + 7) ≥ 0. 3

Half Convex Function Method

31

In accordance with Remark 3, the p equality holds for a = b = c = d = 1, and also for p 1+ 7 (or any cyclic permutation). a = 3 − 7 and b = c = d = 3 Remark. Similarly, we can prove the following generalization. p • If a1 , a2 , . . . , an ≥ n − 1 − n2 − 3n + 3 such that a1 + a2 + · · · + an = n, then 1

+

2 + a12

1 2 + a22

+ ··· +

n 1 ≥ , 2 + an2 3

with equality for a1 = a2 p= · · · = an = 1, and also for a1 = n − 1 − 1 + n2 − 3n + 3 a2 = a3 = · · · = an = (or any cyclic permutation). n−1

p

n2 − 3n + 3 and

P 1.14. If a1 , a2 , . . . , an ∈ [0, n − 2] such that a1 + a2 + · · · + an = n, then 1 n + a12

+

1 n + a22

+ ··· +

n 1 . ≤ 2 n + an n+1 (Vasile Cirtoaje, 2008)

Solution. Write the inequality as f (a1 ) + f (a2 ) + · · · + f (an ) ≥ n f (s), where f (u) =

s=

a1 + a2 + · · · + an = 1, n

−1 , u ∈ [0, n − 2], n ≥ 3. n + u2

For u ∈ [0, 1], we have f 00 (u) =

2(n − 3u2 ) ≥ 0. (n + u2 )3

Thus, f is convex on [0, 1]. By HCF Theorem and Remark 1, it suffices to show that h(x, y) ≥ 0 for x, y ≥ 0 such that x + (n − 1) y = n. We have g(u) = and h(x, y) =

f (u) − f (1) u+1 = u−1 (n + 1)(n + u2 )

g(x) − g( y) n− x − y − xy = . x−y (n + 1)(n + x 2 )(n + y 2 )

We need to show that n − x − y − x y ≥ 0.

32

Vasile Cîrtoaje

Indeed, we have n− x − y − xy =

(n − x)(n − 2 − x) ≥ 0. n−1

The equality holds for a1 = a2 = · · · = an = 1, and also for a1 = n − 2 and a2 = a3 = 2 · · · = an = (or any cyclic permutation). n−1

P 1.15. If a, b, c are nonnegative real numbers such that a + b + c = 3, then 3−a 3− b 3−c 3 + + ≥ . 9 + a2 9 + b2 9 + c 2 5 (Vasile Cirtoaje, 2013) Solution. Write the inequality as f (a) + f (b) + f (c) ≥ 3 f (s), where f (u) =

s=

a+b+c = 1, 3

3−u , u ∈ [0, 3]. 9 + u2

For u ∈ [1, 3], we have 1 00 u2 (9 − u) + 27(u − 1) f (u) = > 0. 2 (9 + u2 )3 Thus, f is convex on [1, 3]. By HCF Theorem and Remark 1, it suffices to show that h(x, y) ≥ 0 for x, y ≥ 0 such that x + 2 y = 3, where h(x, y) =

g(x) − g( y) , x−y

We have g(u) = and h(x, y) =

g(u) =

f (u) − f (1) . u−1

−(6 + u) 5(9 + u2 )

x y + 6x + 6 y − 9 x(9 − x) = ≥ 0. 5(9 + x 2 )(9 + y 2 ) 10(9 + x 2 )(9 + y 2 )

The equality holds for a = b = c = 1, and also for a = 0 and b = c = 3/2 (or any cyclic permutation). Remark. Similarly, we can prove the following generalization.

Half Convex Function Method

33

• If a1 , a2 , . . . , an (n ≥ 3) are nonnegative real numbers such that a1 + a2 + · · · + an = n, then n − a1 n2

+ (n2

− 3n + 1)a12

+

n − a2 n2

+ (n2

− 3n + 1)a22

+ ··· +

n2

n − an n ≥ , 2 − 3n + 1)an 2n − 1

+ (n2

with equality for a1 = a2 = · · · = an = 1, and also for a1 = 0 and a2 = a3 = · · · = an = n (or any cyclic permutation). n−1

P 1.16. If a, b, c are nonnegative real numbers such that a + b + c = 3, then 1 1 3 1 + + ≥ . 2 2 2 5+a+a 5+ b+ b 5+c+c 7 (Vasile Cirtoaje, 2008) Solution. Write the inequality as f (a) + f (b) + f (c) ≥ 3 f (s), where f (u) =

s=

a+b+c = 1, 3

1 , u ∈ [0, 3]. 5 + u + u2

For u ≥ 1, we have f 00 (u) =

2(3u2 + 3u − 4) > 0. (5 + u + u2 )3

Thus, f is convex on [1,3]. By HCF Theorem and Remark 1, it suffices to show that h(x, y) ≥ 0 for x, y ≥ 0 such that x + 2 y = 3. We have g(u) =

f (u) − f (1) −2 − u = u−1 7(5 + u + u2 )

and h(x, y) =

g(x) − g( y) x y + 2(x + y) − 3 x(5 − x) = = ≥ 0. 2 2 x−y 7(5 + x + x )(5 + y + y ) 14(5 + x + x 2 )(5 + y + y 2 )

In accordance with Remark 3, the equality holds for a = b = c = 1, and also for a = 0 and b = c = 3/2 (or any cyclic permutation). Remark. Similarly, we can prove the following generalization.

34

Vasile Cîrtoaje

• Let a1 , a2 , . . . , an be nonnegative real numbers such that a1 + a2 + · · · + an = n. If 2(2n − 1) 0 0. (10 + u + u2 )3

Thus, f is convex on [0,1]. By HCF Theorem and Remark 1, it suffices to show that h(x, y) ≥ 0 for x, y ≥ 0 such that x + 3 y = 4. We have g(u) = and h(x, y) =

f (u) − f (1) 2+u = u−1 12(10 + u + u2 )

g(x) − g( y) 8 − 2(x + y) − x y = . x−y 12(10 + x + x 2 )(10 + y + y 2 )

We need to show that 8 − 2(x + y) − x y ≥ 0. Indeed, we have 8 − 2(x + y) − x y = 3 y 2 ≥ 0.

Half Convex Function Method

35

The equality holds for a = b = c = d = 1, and also for a = 4 and b = c = d = 0 (or any cyclic permutation). Remark. Similarly, we can prove the following generalization. • Let a1 , a2 , . . . , an (n ≥ 4) be nonnegative real numbers such that a1 + a2 +· · ·+ an = n. If k ≥ 2n + 2, then 1 k + a1 + a12

1

+

k + a2 + a22

+ ··· +

1 n ≤ , 2 k + an + an k+2

with equality for a1 = a2 = · · · = an = 1. If k = 2n + 2, then the equality holds also for a1 = n and a2 = a3 = · · · = an = 0 (or any cyclic permutation).

P 1.18. Let a1 , a2 , . . . , an be nonnegative real numbers such that a1 + a2 + · · · + an = n. If 1 k ≥ 1 − , then n 1 n 1 1 ≥ . + + ··· + 2 2 2 1 + ka 1 + k 1 + ka1 1 + ka2 n (Vasile Cirtoaje, 2005) Solution. Write the inequality as f (a1 ) + f (a2 ) + · · · + f (an ) ≥ n f (s), where f (u) =

a1 + a2 + · · · + an = 1, n

1 , u ∈ [0, n]. 1 + ku2

For u ∈ [1, n], we have f 00 (u) = since

s=

2k(3ku2 − 1) > 0, (1 + ku2 )3

 ‹ 1 3 3ku − 1 ≥ 3k − 1 ≥ 3 1 − − 1 = 2 − > 0. n n 2

Thus, f is convex on [1, n]. By HCF Theorem and Remark 1, it suffices to show that h(x, y) ≥ 0 for x, y ≥ 0 such that x + (n − 1) y = n. We have g(u) = and h(x, y) =

f (u) − f (1) −k(u + 1) = u−1 (1 + k)(1 + ku2 )

g(x) − g( y) k2 (x + y + x y) − k = . x−y (1 + k)(1 + k x 2 )(1 + k y 2 )

36

Vasile Cîrtoaje

We need to show that k(x + y + x y) − 1 ≥ 0. Indeed, we have  ‹ 1 x(2n − 2 − x) ≥ 0. k(x + y + x y) − 1 ≥ 1 − (x + y + x y) − 1 = n n 1 The equality holds for a1 = a2 = · · · = an = 1. If k = 1 − , then the equality holds also n n for a1 = 0 and a2 = a3 = · · · = an = (or any cyclic permutation). n−1

P 1.19. Let a1 , a2 , . . . , an be real numbers such that a1 + a2 + · · · + an = n. If 0 < k ≤ n−1 , then n2 − n + 1 n 1 1 1 ≤ . + + ··· + 2 2 2 1 + ka 1 + k 1 + ka1 1 + ka2 n (Vasile Cirtoaje, 2005) Solution. Replacing all negative numbers ai by −ai , we need to show the same inequality for a1 , a2 , . . . , an ≥ 0 such that a1 + a2 + · · · + an ≥ n. Since the left side of the desired inequality is decreasing with respect to each ai , is sufficient to consider that a1 + a2 + · · · + an = n. Write the inequality as f (a1 ) + f (a2 ) + · · · + f (an ) ≥ n f (s), where f (u) =

s=

a1 + a2 + · · · + an = 1, n

−1 , u ∈ [0, n]. 1 + ku2

For u ∈ [0, 1], we have f 00 (u) =

2k(1 − 3ku2 ) ≥ 0, (1 + ku2 )3

since 1 − 3ku2 ≥ 1 − 3k ≥ 1 −

3(n − 1) (n − 2)2 = ≥ 0. n2 − n + 1 n2 − n + 1

Thus, f is convex on [0, 1]. By HCF Theorem and Remark 1, it suffices to show that h(x, y) ≥ 0 for x, y ≥ 0 such that x + (n − 1) y = n. We have g(u) =

f (u) − f (1) k(u + 1) = u−1 (1 + k)(1 + ku2 )

Half Convex Function Method and h(x, y) =

37

k − k2 (x + y + x y) g(x) − g( y) = . x−y (1 + k)(1 + k x 2 )(1 + k y 2 )

We need to show that 1 − k(x + y + x y) ≥ 0. Indeed, we have 1 − k(x + y + x y) ≥ 1 −

n−1 (x − n + 1)2 (x + y + x y) = ≥ 0. n2 − n + 1 n2 − n + 1

The equality holds for a1 = a2 = · · · = an = 1. If k = also for a1 = n − 1 and a2 = a3 = · · · = an =

n2

n−1 , then the equality holds −n+1

1 (or any cyclic permutation). n−1

P 1.20. Let a1 , a2 , . . . , an be nonnegative numbers such that a1 + a2 + · · · + an = n. If n2 , then k≥ 4(n − 1) a1 (a1 − 1) a12

+k

+

a2 (a2 − 1) a22

+k

+ ··· +

an (an − 1) ≥ 0. an2 + k (Vasile Cîrtoaje, 2012)

Solution. Write the inequality as f (a1 ) + f (a2 ) + · · · + f (an ) ≥ n f (s), where f (u) = From f 0 (u) =

u2 + 2ku − k , (u2 + k)2

s=

a1 + a2 + · · · + an = 1, n

u(u − 1) , u ∈ [0, n]. u2 + k f 00 (u) =

2(k2 − u3 ) + 6ku(1 − u) , (u2 + k)3

it follow that f is convex on [0, 1]. By HCF Theorem and Remark 1, it suffices to show that h(x, y) ≥ 0 for x, y ≥ 0 such that x + (n − 1) y = n, where h(x, y) =

g(x) − g( y) , x−y

g(u) =

f (u) − f (1) . u−1

We have g(u) =

u , 2 u +k

h(x, y) =

k−xy n2 − 4(n − 1)x y ≥ . (x 2 + k)( y 2 + k) 4(n − 1)(x 2 + k)( y 2 + k)

38

Vasile Cîrtoaje

We only need to show that n2 ≥ 4(n − 1)x y. Indeed, this follows by the AM-GM inequality, as follows: ” Æ —2 n2 = [x + (n − 1) y]2 ≥ 2 (n − 1)x y = 4(n − 1)x y. The equality holds for a1 = a2 = · · · = an = 1, and also for a1 = n/2 and a2 = a3 = · · · = an = n/(2n − 2) (or any cyclic permutation).

P 1.21. If a1 , a2 , . . . , an are nonnegative real numbers such that a1 + a2 + · · · + an = n, then 1 − an 1 − a1 1 − a2 + + ··· + ≤ 0. 2 2 (n − 2a1 ) (n − 2a2 ) (n − 2an )2 (Vasile Cîrtoaje, 2012) Solution. For n = 2, the inequality is an identity. Consider further n ≥ 3 and write the inequality as f (a1 ) + f (a2 ) + · · · + f (an ) ≥ n f (s), where f (u) = From f 0 (u) =

s=

a1 + a2 + · · · + an = 1, n

u−1 , u ∈ I = [0, n] \ {n/2}. (n − 2u)2 2u + n − 4 , (n − 2u)3

f 00 (u) =

8(u + n − 3) , (n − 2u)4

it follow that f is convex on [0, 1]. By HCF Theorem, Remark 1 and Remark 4, it suffices to show that h(x, y) ≥ 0 for x, y ∈ I such that x + (n − 1) y = n, where h(x, y) =

g(x) − g( y) , x−y

We have g(u) = and h(x, y) =

g(u) =

f (u) − f (1) . u−1

1 (n − 2u)2

4(n − x − y) 4(n − 2) y = ≥ 0. 2 2 (n − 2x) (n − 2 y) (n − 2x)2 (n − 2 y)2

In accordance with Remark 3, the equality holds for a1 = a2 = · · · = an = 1, and also for a1 = n and a2 = a3 = · · · = an = 0 (or any cyclic permutation).

Half Convex Function Method

39

P 1.22. Let a1 , a2 , . . . , an be nonnegative real numbers such that a1 + a2 + · · · + an = n. If n , then k ≥1+ p n−1 a12 − 1 (a1 + k)2

+

a22 − 1 (a2 + k)2

+ ··· +

an2 − 1 (an + k)2

≥ 0. (Vasile Cirtoaje, 2008)

Solution. Write the inequality as f (a1 ) + f (a2 ) + · · · + f (an ) ≥ n f (s), where f (u) =

s=

a1 + a2 + · · · + an = 1, n

u2 − 1 , u ∈ [0, n]. (u + k)2

For u ∈ [0, 1], we have f 00 (u) =

2(k2 − 3 − 2ku) 2(k2 − 2k − 3) 2(k + 1)(k − 3) ≥ = ≥ 0. (u + k)4 (u + k)4 (u + k)4

Thus, f is convex on [0, 1]. By HCF Theorem and Remark 1, it suffices to show that h(x, y) ≥ 0 for x, y ≥ 0 such that x + (n − 1) y = n. We have g(u) = and h(x, y) =

f (u) − f (1) u+1 = u−1 (u + k)2

g(x) − g( y) (k − 1)2 − (1 + x)(1 + y) = . x−y (x + k)2 ( y + k)2

Since (k − 1)2 ≥

n2 , n−1

we need to show that n2 ≥ (n − 1)(1 + x)(1 + y). Indeed, n2 − (n − 1)(1 + x)(1 + y) = n2 − (1 + x)(2n − 1 − x) = (x − n + 1)2 ≥ 0. Thus, the proof is completed. The equality holds for a1 = a2 = · · · = an = 1. If k = n 1 1+ p , then the equality holds also for a1 = n − 1 and a2 = a3 = · · · = an = n−1 n−1 (or any cyclic permutation).

40

Vasile Cîrtoaje

P 1.23. Let as 1 , a2 , . . . , an be nonnegative real numbers such that a1 + a2 + · · · + an = n. If 2n − 1 0< k ≤1+ , then n−1 a12 − 1 (a1 + k)2

+

a22 − 1 (a2 + k)2

+ ··· +

an2 − 1 (an + k)2

≤ 0. (Vasile Cirtoaje, 2008)

Solution. Write the inequality as f (a1 ) + f (a2 ) + · · · + f (an ) ≥ n f (s), where f (u) =

s=

a1 + a2 + · · · + an = 1, n

1 − u2 , u ∈ [0, n]. (u + k)2

For u ≥ 1, we have f 00 (u) =

2(2ku − k2 + 3) 2(2k − k2 + 3) 2(1 + k)(3 − k) ≥ = > 0. (u + k)4 (u + k)4 (u + k)4

Thus, f is convex on [1, n]. By HCF Theorem and Remark 1, it suffices to show that h(x, y) ≥ 0 for x, y ≥ 0 such that x + (n − 1) y = n. We have g(u) = and h(x, y) =

f (u) − f (1) −u − 1 = u−1 (u + k)2

2k − k2 + x + y g(x) − g( y) 2k − k2 + x + y + x y = ≥ . x−y (x + k)2 ( y + k)2 (x + k)2 ( y + k)2

Since x+y≥

x + (n − 1) y n = , n−1 n−1

we get 2k − k2 + x + y ≥ 2k − k2 +

n 2n − 1 = −(k − 1)2 + ≥ 0, n−1 n−1

hence h(x, y) ≥ 0.sThus, the proof is completed. The equality holds for a1 = a2 = · · · = 2n − 1 an = 1. If k = 1 + , then the equality holds also for a1 = 0 and a2 = a3 = · · · = n−1 n an = (or any cyclic permutation). n−1

Half Convex Function Method

41

P 1.24. Let a1 , a2 , . . . , an be nonnegative real numbers such that a1 + a2 + · · · + an = n. If (n − 1)(2n − 1) k≥ , then n2 1 1 n 1 + + ··· + ≥ . 3 3 3 1 + kan 1+k 1 + ka1 1 + ka2 (Vasile Cirtoaje, 2008) Solution. Write the inequality as f (a1 ) + f (a2 ) + · · · + f (an ) ≥ n f (s), where f (u) = For u ∈ [1, n], we have f 00 (u) =

s=

a1 + a2 + · · · + an = 1, n

1 , u ∈ [0, n]. 1 + ku3

6ku(2ku3 − 1) 6ku(2k − 1) ≥ > 0. (1 + ku3 )3 (1 + ku3 )3

Thus, f is convex on [1, n]. By HCF Theorem and Remark 1, it suffices to show that h(x, y) ≥ 0 for x, y ≥ 0 such that x + (n − 1) y = n, where h(x, y) =

g(x) − g( y) , x−y

We have g(u) = and

g(u) =

f (u) − f (1) . u−1

−k(u2 + u + 1) (1 + k)(1 + ku3 )

x 2 y 2 + x y(x + y − 1) + (x + y)2 − (x + y + 1)/k 1 h(x, y) = . k2 (1 + k)(1 + k x 3 )(1 + k y 3 )

Since x+y≥

x + (n − 1) y n = > 1, n−1 n−1

it suffices to show that k(x + y)2 ≥ x + y + 1. From x + y ≥ k(x + y) ≥

n , we get n−1

2n − 1 n

and

• ˜ n 2n − 1 − 1 = 1. n−1 n (n − 1)(2n − 1) The equality holds for a1 = a2 = · · · = an = 1. If k = , then the equality n2 n holds also for a1 = 0 and a2 = a3 = · · · = an = (or any cyclic permutation). n−1 k(x + y)2 − x − y ≥ (x + y)[k(x + y) − 1] ≥

42

Vasile Cîrtoaje

P 1.25. Let a1 , a2 , . . . , an be nonnegative real numbers such that a1 + a2 + · · · + an = n. If n−1 , then 0 0. (1 + ku3 )3 (1 + ku3 )3

Thus, f is convex on [0, 1]. By HCF Theorem and Remark 1, it suffices to show that h(x, y) ≥ 0 for x, y ≥ 0 such that x + (n − 1) y = n, where h(x, y) =

g(x) − g( y) , x−y

We have g(u) =

g(u) =

f (u) − f (1) . u−1

k(u2 + u + 1) (1 + k)(1 + ku3 )

and

(x + y + 1)/k − x 2 y 2 − x y(x + y − 1) − (x + y)2 1 h(x, y) = . k2 (1 + k)(1 + k x 3 )(1 + k y 3 ) It suffices to show that (n2 − 2n + 2)(x + y + 1) − x 2 y 2 − x y(x + y − 1) − (x + y)2 ≥ 0, n−1 which is equivalent to [2 + n y − (n − 1) y 2 ][1 − (n − 1) y]2 ≥ 0.

This is true since 2 + n y − (n − 1) y 2 = 2 + y[n − (n − 1) y] = 2 + x y > 0. In accord with Remark 3, the equality holds for a1 = a2 = · · · = an = 1. If k = n−1 1 , then the equality holds also for a1 = n − 1 and a2 = a3 = · · · = an = 2 n − 2n + 2 n−1 (or any cyclic permutation).

Half Convex Function Method

43

P 1.26. Let a1 , a2 , . . . , an be nonnegative real numbers such that a1 + a2 + · · · + an = n. If n2 k≥ , then n−1 v v v t a1 t a2 t an n + + ··· + ≤p . k − a1 k − a2 k − an k−1 (Vasile Cirtoaje, 2008) Solution. Write the inequality as f (a1 ) + f (a2 ) + · · · + f (an ) ≥ n f (s), where f (u) = −

s

s=

a1 + a2 + · · · + an = 1, n

u , u ∈ [0, n]. k−u

For u ∈ [0, 1], we have f 00 (u) =

k(k − 4u) 4u3/2 (k − u)5/2



k(k − 4) 4u3/2 (k − u)5/2

≥ 0.

Thus, f is convex on [0, 1]. By HCF Theorem, it suffices to prove that f (x) + (n − 1) f ( y) ≥ n f (1) for all x, y ≥ 0 such that x + (n − 1) y = n. We write the inequality as v v t (k − 1)x t (k − 1) y + (n − 1) ≤ n, k−x k− y – ™ v v t (k − 1) y t (n − 1)k(1 − y) 1+ ≤ 1 + (n − 1) 1 − . (n − 1) y + k − n k− y Let z=

v t (k − 1) y k− y

,

which yields y= 1− y =

(k − 1)(1 − z 2 ) , z2 + k − 1

kz 2 , z2 + k − 1

(n − 1) y + k − n =

(k − 1)(nz 2 + k − n) , z2 + k − 1

hence k(1 − y) k(1 − z 2 ) 1 = = 2 (n − 1) y + k − n k − n(1 − z ) 1/(1 − z 2 ) − n/k 1 n(1 − z 2 ) ≤ = . 1/(1 − z 2 ) − (n − 1)/n (n − 1)z 2 + 1

44

Vasile Cîrtoaje

Therefore, it suffices to show that v t n(n − 1)(1 − z 2 ) 1+ ≤ 1 + (n − 1)(1 − z). (n − 1)z 2 + 1 By squaring, we get the obvious inequality (z − 1)2 [(n − 1)z − 1]2 ≥ 0. So, we only need to show that 1 + (n − 1)(1 − z) ≥ 0, that is, z ≤ n/(n − 1). Since y= and

we have

n− x n ≤ n−1 n−1

1 n−1 1 ≤ 2 = 2 , k−1 n /(n − 1) − 1 n −n+1 v v v t t k−1 t k−1 n z= ≤ = k/ y − 1 k(n − 1)/n − 1 n − 1 − 1/(k − 1) p v t n n n2 − n + 1 ≤ = < . 2 n − 1 − (n − 1)/(n − n + 1) n−1 n−1

The proof is completed. The equality holds for a1 = a2 = · · · = an = 1. If k = n2 n(n − 1)2 , then the equality holds also for a1 = 2 and a2 = a3 = · · · = an = n−1 n − 2n + 2 n (or any cyclic permutation). 2 (n − 1)(n − 2n + 2)

P 1.27. If a1 , a2 , . . . , an are nonnegative real numbers such that a1 + a2 + · · · + an = n, then 2 2 2 n−a1 + n−a2 + · · · + n−an ≥ 1. (Vasile Cîrtoaje, 2006) Solution. Let k = ln n. Write the inequality as f (a1 ) + f (a2 ) + · · · + f (an ) ≥ n f (s), where

s=

a1 + a2 + · · · + an = 1, n

2

f (u) = n−u , u ∈ [0, n]. For u ≥ 1, we have 2

2

2

f 00 (u) = 2kn−u (2ku2 − 1) ≥ 2kn−u (2k − 1) ≥ 2kn−u (2 ln 2 − 1) > 0;

Half Convex Function Method

45

therefore, f is convex on [1, n]. By HCF Theorem and Remark 5, it suffices to show that f (x) + (n − 1) f ( y) ≥ n f (1) for 0 ≤ x ≤ 1 ≤ y and x + (n − 1) y = n. The desired inequality is equivalent to g(x) ≥ 0, where 2

2

g(x) = n−x + (n − 1)n− y ,

y=

n− x , 0 ≤ x ≤ 1. n−1

Since y 0 = −1/(n − 1), we get 2

2

2

2

g 0 (x) = −2x kn−x − 2(n − 1)k y y 0 n− y = 2k( y n− y − x n−x ). The derivative g 0 (x) has the same sign as g1 (x), where 2

2

g1 (x) = ln( y n− y ) − ln(x n−x ) = ln y − ln x + k(x 2 − y 2 ). From g10 (x) =

• ˜ y0 1 −1 2k + 2(n − 2)k x − + 2k(x − y y 0 ) = n + , y x x(n − x) (n − 1)2

we see that g10 (x) has for 0 < x ≤ 1 the same sign as h(x) =

−(n − 1)2 + x(n − x)(1 + nx − 2x). 2k

Since h0 (x) = n + 2(n2 − 2n − 1)x − 3(n − 2)x 2 ≥ nx + 2(n2 − 2n − 1)x − 3(n − 2)x = 2(n − 1)(n − 2)x ≥ 0, h is strictly increasing on [0, 1]. From −(n − 1)2 h(0) = < 0, 2k

 ‹ 1 h(1) = (n − 1) 1 − > 0, 2k 2

it follows that there is x 1 ∈ (0, 1) such that h(x) < 0 for x ∈ [0, x 1 ), h(x 1 ) = 0 and h(x) > 0 for x ∈ (x 1 , 1]. Therefore, g1 is strictly decreasing on (0, x 1 ] and strictly increasing on [x 1 , 1]. Since lim x→0 g1 (x) = ∞ and g1 (1) = 0, there is x 2 ∈ (0, x 1 ) such that g1 (x) > 0 for x ∈ (0, x 2 ), g1 (x 2 ) = 0 and g1 (x) < 0 for x ∈ (x 2 , 1). Consequently, g is strictly increasing on [0, x 2 ] and strictly decreasing on [x 2 , 1]. From g(0) > 0 and g(1) = 0, it follows that g(x) ≥ 0 for x ∈ [0, 1]. The proof is completed. The equality holds for a1 = a2 = · · · = an = 1.

P 1.28. If a, b, c, d, e are nonnegative real numbers such that a + b + c + d + e = 5, then then (a2 + 1)(b2 + 1)(c 2 + 1)(d 2 + 1)(e2 + 1) ≥ (a + 1)(b + 1)(c + 1)(d + 1)(e + 1).

46

Vasile Cîrtoaje

Solution. Write the inequality as f (a) + f (b) + f (c) + f (d) + f (e) ≥ n f (s),

s=

a+b+c+d+e = 1, 5

where f (u) = ln(u2 + 1) − ln(u + 1),

u ∈ [0, 5].

For u ∈ [0, 1], we have f 00 (u) =

2(1 − u2 ) 1 (u2 − u4 ) + 4u(1 − u2 ) + u2 + 3 + = > 0. (u2 + 1)2 (u + 1)2 (u2 + 1)2 (u + 1)2

Therefore, f is convex on [0, 1]. By HCF Theorem and Remark 2, we only need to show that H(x, y) ≥ 0 for x, y ≥ 0 such that x + 4 y = 5, where H(x, y) =

2(1 − x y) f 0 (x) − f 0 ( y) 1 = 2 + ; x−y (x + 1)( y 2 + 1) (x + 1)( y + 1)

that is, (x 2 + 1)( y 2 + 1)H(x, y) = 2(1 − x y) + Since

x2 + 1 x +1 ≥ , x +1 2

(x 2 + 1)( y 2 + 1) . (x + 1)( y + 1)

y2 + 1 y +1 ≥ , y +1 2

it suffices to prove that 2(1 − x y) +

(x + 1)( y + 1) ≥ 0, 4

which is equivalent to x + y + 9 − 7x y ≥ 0. Indeed, x + y + 9 − 7x y = 28x 2 − 38x + 14 = 28(x − 19/28)2 + 31/28 > 0. The proof is completed. The equality holds for a = b = c = d = e = 1.

P 1.29. If a1 , a2 , · · · , a10 are nonnegative real numbers such that a1 + a2 + · · · + a10 = 10, then 2 (1 − a1 + a12 )(1 − a2 + a22 ) · · · (1 − a10 + a10 ) ≥ 1. (Vasile Cîrtoaje, 2006)

Half Convex Function Method

47

Solution. Write the inequality as f (a1 ) + f (a2 ) + · · · + f (a10 ) ≥ 10 f (s),

s=

a1 + a2 + · · · + a10 = 1, 10

where f (u) = ln(1 − u + u2 ), From f 00 (u) =

u ∈ [0, 10].

1 + 2u(1 − u) , (1 − u + u2 )2

it follows that f 00 (u) > 0 for u ∈ [0, 1], hence f is convex on [0, 1]. According to HCF Theorem, we only need to show that f (x) + 9 f ( y) ≥ 10 f (1) for all x, y ≥ 0 such that x + 9 y = 10. Using Remark 2, it suffices to prove that H(x, y) ≥ 0, where H(x, y) = Since H(x, y) =

f 0 (x) − f 0 ( y) . x−y

1 + x + y − 2x y , (1 − x + x 2 )(1 − y + y 2 )

we need to show that 1 + x + y − 2x y ≥ 0. Indeed, 7 1 + x + y − 2x y = 18 y − 28 y + 11 = 18 y − 9 

2

‹2

+

1 > 0. 9

The proof is completed. The equality holds for a1 = a2 = · · · = a10 = 1.

P 1.30. If a1 , a2 , . . . , an (n ≥ 3) are positive real numbers such that a1 + a2 + · · · + an = 1, then       ‹ p p p 1 1 1 1 n p n− p . p − a2 · · · p − an ≥ p − a1 a1 a2 an n (Vasile Cîrtoaje, 2006) Solution. Apply HCF Theorem to the function  ‹ p 1 1 f (u) = ln p − u = ln(1 − u) − ln u, 2 u

u ∈ (0, 1),

for s = 1/n. From f 0 (u) =

−1 1 − , 1 − u 2u

f 00 (u) =

1 − 2u − u2 , 2u2 (1 − u)2

48

Vasile Cîrtoaje

p it follows that f is convex on (0, 2 − 1]. Since s=

1 1 p ≤ < 2 − 1, n 3

f is also convex on (0, s]. First Solution. By HCF Theorem, it suffices to show that f (x) + (n − 1) f ( y) ≥ n f (1/n) for all x, y > 0 such that x + (n − 1) y = 1; that is, to show that 

p 1 p − x x

‹

1 p p − y y

n−1 ≥

 p

1 n− p n

‹n

.

Write this inequality as nn/2 (1 − y)n−1 ≥ (n − 1)n−1 x 1/2 y (n−3)/2 . By squaring, this inequality becomes as follows nn (1 − y)2n−2 ≥ (n − 1)2n−2 x y n−3 , (2 − 2 y)2n−2 ≥ •

1 n · + x + (n − 3) y n

˜2n−2

(2n − 2)2n−2 x y n−3 , nn

≥ [n + 1 + (n − 3)]n+1+(n−3) ·

1 · x · y n−3 . nn

Clearly, the last inequality follows from the AM-GM inequality. The proof is completed. The equality holds for a1 = a2 = · · · = an = 1/n. Second Solution. By HCF Theorem and Remark 2, it suffices to prove that H(x, y) ≥ 0 for x, y > 0 such that x + (n − 1) y = 1, where H(x, y) =

f 0 (x) − f 0 ( y) . x−y

We have (n − 1)(1 + y) − 2 1− x − y − xy = 2x y(1 − x)(1 − y) 2x(1 − x)(1 − y) 2(1 + y) − 2 y ≥ = > 0. 2x(1 − x)(1 − y) x(1 − x)(1 − y)

H(x, y) =

Remark 1. We may write the inequality in P 1.30 in the form  Y  ‹ n  n Y p p 1 n 1 − 1 · (1 + a ) ≥ n − . p p i ai n i=1 i=1

Half Convex Function Method

49

On the other hand, by the AM-GM inequality and the Cauchy-Schwarz inequality, we have v !n ‚ Œn  ‹ u X n n n Y p 1 n 1 Xp t1 ai = 1+ p ai ≤ 1 + (1 + ai ) ≤ 1 + . n i=1 n i=1 n i=1 Thus, the following statement follows. • If a1 , a2 , . . . , an (n ≥ 3) are positive real numbers such that a1 + a2 + · · · + an = 1, then      p 1 1 1 n p − 1 · · · p − 1 ≥ ( n − 1) , p −1 a1 a2 an with equality for a1 = a2 = · · · = an = 1/n. Remark 2. By squaring, the inequality in P 1.30 becomes n Y (1 − ai )2 (n − 1)2n . ≥ ai nn i=1

1+ x is convex on (0, 1), by Jensen’s 1− x inequality,   a1 + a2 + · · · + an n ‹ ‹   n 1+ Y 1 + ai n+1 n   n ≥ . = a1 + a2 + · · · + an  1 − ai n−1 i=1 1− n Multiplying these inequality yields the following result (Kee-Wai Lau, 2000).

On the other hand, since the function f (x) = ln

• If a1 , a2 , . . . , an (n ≥ 3) are positive real numbers such that a1 + a2 + · · · + an = 1, then ‹ ‹  ‹  ‹  1 1 1 n 1 − a1 − a2 · · · − an ≥ n − , a1 a2 an n with equality for a1 = a2 = · · · = an = 1/n.

P 1.31. Let a1 , a2 , . . . , an be positive real numbers such that a1 + a2 + · · · + an = n. If  2 p 2 n−1 0< k ≤ 1+ , n then



1 ka1 + a1

‹

‹  ‹ 1 1 ka2 + · · · kan + ≥ (k + 1)n . a2 an (Vasile Cîrtoaje, 2006)

50

Vasile Cîrtoaje

Solution. Write the inequality as f (a1 ) + f (a2 ) + · · · + f (an ) ≥ n f (s), where

s=

 ‹ 1 f (u) = ln ku + , u

We have f 0 (u) =

ku2 − 1 , u(ku2 + 1)

f 00 (u) =

a1 + a2 + · · · + an = 1, n

u ∈ (0, n). 1 + 4ku2 − k2 u4 . u2 (ku2 + 1)2

For u ∈ (0, 1], we have f 00 (u) > 0, since 1 + 4ku2 − k2 u4 > ku2 (4 − ku2 ) ≥ ku2 (4 − k) ≥ 0. Therefore, f is convex on (0, 1]. By HCF Theorem and Remark 2, it suffices to prove that H(x, y) ≥ 0 for x, y > 0 such that x + (n − 1) y = n, where H(x, y) = Since H(x, y) =

f 0 (x) − f 0 ( y) . x−y

1 + k(x + y)2 − k2 x 6 y 2 k[(x + y)2 − k x 6 y 2 ] > , x y(k x 2 + 1)(k y 2 + 1) x y(k x 2 + 1)(k y 2 + 1)

it suffices to show that x+y≥

p

k x y.

Indeed, by the Cauchy-Schwarz inequality, we have p (x + y)[(n − 1) y + x] ≥ ( n − 1 + 1)2 x y, hence

p 1 p ( n − 1 + 1)2 x y ≥ k x y. n The equality holds for a1 = a2 = · · · = an = 1. x+y≥

P 1.32. If a, b, c, d are nonzero real numbers such that a, b, c, d ≥ then

−1 , 2

a + b + c + d = 4,

‹ 1 1 1 1 1 1 1 1 3 2 + 2 + 2 + 2 + + + + ≥ 16. a b c d a b c d 

Half Convex Function Method

51

Solution. Write the inequality as f (a) + f (b) + f (c) + f (d) ≥ 4 f (s), s = where

a+b+c+d = 1, 4

• ˜ 3 1 −1 11 f (u) = 2 + , u ∈ I = , \ {0}. u u 2 2

Clearly, f is convex for u ∈ I, u ≥ s. Therefore, by HCF Theorem and Remark 4, it suffices to prove that f (x) + 3 f ( y) ≥ 4 f (1) for all x, y ∈ I such that x + 3 y = 4. According to Remark 1, we only need to show that h(x, y) ≥ 0, where h(x, y) =

g(x) − g( y) , x−y

g(u) =

f (u) − f (1) . u−1

Indeed, we have 3 4 g(u) = − − 2 , u u h(x, y) =

4x y + 3x + 3 y 2(1 + 2x)(6 − x) = ≥ 0. 2 2 x y 3x 2 y 2

The proof is completed. In accord with Remark 3, the equality holds for a = b = c = −1 3 d = 1, and also for a = and b = c = d = (or any cyclic permutation). 2 2

P 1.33. If a1 , a2 , . . . , an are nonnegative real numbers such that a12 + a22 + · · · + an2 = n, then s n 3 3 3 a1 + a2 + · · · + an − n + (a1 + a2 + · · · + an − n) ≥ 0. n−1 (Vasile Cirtoaje, 2007) Solution. Replacing each ai by

p

ai , we have to prove that a1 + a2 + · · · + an = 1, n

f (a1 ) + f (a2 ) + · · · + f (an ) ≥ n f (s),

s=

p p f (u) = u u + k u,

n , n−1

where

For u ≥ 1, we have f 00 (u) =

k=

s

u ∈ [0, n].

3u − k 3−k p ≥ p > 0. 4u u 4u u

52

Vasile Cîrtoaje

Therefore, f is convex for u ≥ 1. According to HCF Theorem and Remark 1, it suffices to show that h(x, y) ≥ 0 for x, y ≥ 0 such that x + (n − 1) y = n. Since g(u) = and

f (u) − f (1) u+k =1+ p u−1 u+1

p p p x + y+ xy−k g(x) − g( y) h(x, y) = = p , p p p x−y ( x + y)( x + 1)( y + 1)

we need to show that

p

x+

p

y+

This is true if p

x+y≥

p

x y ≥ k.

s

n . n−1

Indeed, we have

x n +y= . n−1 n−1 The proof is completed. In accord with Remark 3, sthe equality holds for a1 = a2 = · · · = n an = 1, and also for a1 = 0 and a2 = · · · = an = (or any cyclic permutation). n−1 x+y≥

P 1.34. If a, b, c, d, e are nonnegative real numbers such that a2 + b2 + c 2 + d 2 + e2 = 5, then 1 1 1 1 1 + + + + ≤ 1. 7 − 2a 7 − 2b 7 − 2c 7 − 2d 7 − 2e (Vasile Cirtoaje, 2010) p p p p p Solution. Replacing a, b, c, d, e by a, b, c, d, e, we have to prove that f (a) + f (b) + f (c) + f (d) + f (e) ≥ 5 f (s), where

1 f (u) = p , 2 u−7

For u ∈ [0, 1], we have

s=

a+b+c+d+e = 1, 5

u ∈ [0, 5].

p 7−6 u f (u) = p > 0. p 2u u(7 − 2 u)3 00

Therefore, f is convex for u ∈ [0, 1]. According to HCF Theorem and Remark 1, it suffices to show that h(x, y) ≥ 0 for x, y ≥ 0 such that x + 4 y = 5. Since g(u) =

f (u) − f (1) −2 = p p u−1 5(7 − 2 u)(1 + u)

Half Convex Function Method

53

and p p 2(5 − 2 x − 2 y) g(x) − g( y) = p h(x, y) = p p p p p , x−y ( x + y)(1 + x)(1 + y)(7 − 2 x)(7 − 2 y) we need to show that

p

5 . 2 Indeed, by the Cauchy-Schwarz inequality, we have  ‹ p 1 25 p 2 ( x + y) ≤ 1 + (x + 4 y) = . 4 4 x+

p

y≤

The proof is completed. The equality holds for a = b = c = d = e = 1, and also for 1 a = 2 and b = c = d = e = (or any cyclic permutation). 2 Remark Similarly, we can prove the following generalization. • Let a1 , a2 , . . . , an be nonnegative real numbers such that a12 + a22 + · · · + an2 = n. If n k ≥1+ p , then n−1 1 1 1 n + + ··· + ≤ , k − a1 k − a2 k − an k−1 with equality for a1 = a2 = · · · = an = 1. If k = 1 + p for a1 =

p

n − 1 and a2 = · · · = an = p

P 1.35. If 0 ≤ a1 , a2 , . . . , an < then

1 n−1

n n−1

, then the equality holds also

.

k such that a12 +a22 +· · ·+an2

= n, where 1 < k ≤ 1+

s

n , n−1

1 1 1 n + + ··· + ≥ . k − a1 k − a2 k − an k−1 (Vasile Cirtoaje, 2010)

Solution. Replacing a1 , a2 , . . . , an by

p

a1 ,

p

a2 , . . . ,

f (a1 ) + f (a2 ) + · · · + f (an ) ≥ n f (s), where f (u) =

1 p , k− u

p

s=

an , we have to prove that a1 + a2 + · · · + an = 1, n

u ∈ [0, k2 ).

54

Vasile Cîrtoaje

From

p 3 u−k f (u) = p , p 4u u(k − u)3 00

it follows that f is convex for u ≥ 1, since p 3 u−k ≥3−k ≥2−

s

n > 0. n−1

According to HCF Theorem and Remark 1, it suffices to show that h(x, y) ≥ 0 for all 0 ≤ x, y < k2 such that x + (n − 1) y = n. Since g(u) =

f (u) − f (1) 1 = p p u−1 (k − 1)(k − u)(1 + u)

and p p x + y +1−k g(x) − g( y) h(x, y) = = p p p p p p , x−y (k − 1)( x + y)(1 + x)(1 + y)(k − x)(k − y) we need to show that

p

x+

p

y ≥ k − 1.

Indeed, p

x+

p

y≥

p

x+y≥

s

x +y= n−1

s

n ≥ k − 1. n−1

The proof is completed. In accord with Remark 3, sthe equality holds for a1 = a2 = · · · = n an = 1, and also for a1 = 0 and a2 = · · · = an = (or any cyclic permutation). n−1

P 1.36. If a, b, c are nonnegative real numbers, no two of which are zero, then v v v t 48a t 48b t 48c 1+ + 1+ + 1+ ≥ 15. b+c c+a a+b (Vasile Cirtoaje, 2005) Solution. Due to homogeneity, we may assume that a + b + c = 1. Thus, we need to show that a+b+c 1 f (a) + f (b) + f (c) ≥ 3 f (s), s = = , 3 3 where v t 1 + 47u f (u) = , u ∈ [0, 1). 1−u

Half Convex Function Method

55

From f 00 (u) = p

48(47u − 11) (1 − u)5 (1 + 47u)3

,

it follows that f is convex on [1/3, 1). By HCF Theorem, it suffices to show that f (x) + 2 f ( y) ≥ 3 f (1/3) for x, y ≥ 0 such that x + 2 y = 1; that is, v v t 1 + 47x t 49 − 47x +2 ≥ 15. 1− x 1+ x Setting t=

v t 49 − 47x 1+ x

, 1 < t ≤ 7,

the inequality turns into v t 1175 − 23t 2 t2 − 1

≥ 15 − 2t.

By squaring, this inequality becomes 350 − 15t − 61t 2 + 15t 3 − t 4 ≥ 0, (5 − t)2 (2 + t)(7 − t) ≥ 0. The last inequality is clearly true. From x + 2 y = 1 and f (x) + 2 f ( y) = 3 f (1/3), we get x = y = 1/3 and x = 0, y = 1/2. Therefore, in accordance with Remark 3, the equality in the case a + b + c = 1 holds for a = b = c = 1/3, and also for a = 0 and b = c = 1/2 (or any cyclic permutation). For the original inequality, the equality holds for a = b = c, and also for a = 0 and b = c (or any cyclic permutation).

P 1.37. If a, b, c are nonnegative real numbers, then v v v t t t 3a2 3b2 3c 2 + + ≤ 1. 7a2 + 5(b + c)2 7b2 + 5(c + a)2 7c 2 + 5(a + b)2 (Vasile Cirtoaje, 2008) Solution. Due to homogeneity, we may assume that a + b + c = 3. Thus, we need to show that a+b+c f (a) + f (b) + f (c) ≥ 3 f (s), s = = 1, 3 where v t 3u2 −u =p , u ∈ [0, 3]. f (u) = − 2 2 7u + 5(3 − u) 4u2 − 10u + 15

56

Vasile Cîrtoaje

From f 00 (u) =

10(u − 1)(15 − 4u) 5(−8u2 + 41u − 30) 5(−8u2 + 38u − 30) ≥ = , 2 5/2 2 5/2 (4u − 10u + 15) (4u − 10u + 15) (4u2 − 10u + 15)5/2

it follows that f is convex on [1, 3]. By HCF Theorem, it suffices to prove the original homogeneous inequality for b = c = 1; that is v v t 3a2 t 3 + 2 ≤ 1. 7a2 + 20 5a2 + 10a + 12 By squaring two times, the inequality becomes Æ a(5a3 + 10a2 + 16a + 50) ≥ 3a (7a2 + 20)(5a2 + 10a + 12), a2 (5a6 + 20a5 − 11a4 + 38a3 − 80a2 − 40a + 68) ≥ 0, a2 (a − 1)2 (5a4 + 30a3 + 44a2 + 96a + 68) ≥ 0. The last inequality is clearly true. The equality holds for a = b = c, and also for a = 0 and b = c (or any cyclic permutation).

P 1.38. If a, b, c are nonnegative real numbers, then v v v t t t a2 b2 c2 + + ≥ 1. a2 + 2(b + c)2 b2 + 2(c + a)2 c 2 + 2(a + b)2 (Vasile Cirtoaje, 2008) Solution. Due to homogeneity, we may assume that a + b + c = 3. Thus, we need to show that a+b+c f (a) + f (b) + f (c) ≥ 3 f (s), s = = 1, 3 where v t 3u2 u =p , u ∈ [0, 3]. f (u) = 2 2 u + 2(3 − u) u2 − 4u + 6 From f 00 (u) =

2(2u2 − 11u + 12) 2(−11u + 12) ≥ 2 , 2 5/2 (u − 4u + 6) (u − 4u + 6)5/2

it follows that f is convex on [0, 1]. By HCF Theorem, it suffices to prove the original homogeneous inequality for b = c = 1; that is p

2 +p ≥ 1. 2a2 + 4a + 3 a2 + 8 a

Half Convex Function Method

57

By squaring, the inequality becomes Æ a (a2 + 8)(2a2 + 4a + 3) ≥ 3a2 + 8a − 2. For the nontrivial case 3a2 + 8a − 2 > 0, squaring both sides, we get a6 + 2a5 + 5a4 − 8a3 − 14a2 + 16a − 2 ≥ 0, (a − 1)2 [a4 + 4a3 + 9a2 + 4a + (3a2 + 8a − 2)] ≥ 0. The last inequality is clearly true. The equality holds for a = b = c.

P 1.39. Let a, b, c be nonnegative real numbers, no two of which are zero. If k0 =

k ≥ k0 , then 

2a b+c

‹k

+



ln 3 − 1 ≈ 0.585, ln 2

2b c+a

‹k

+



2c a+b

‹k

≥ 3. (Vasile Cirtoaje, 2005)

Solution. For k = 1, the inequality is just the well known Nesbitt’s inequality 2b 2c 2a + + ≥ 3, b+c c+a a+b while for k ≥ 1, the inequality follows from Jensens’s inequality applied to the convex function f (u) = uk : 

2a b+c

‹k

2b + c+a 

‹k

2c + a+b 

‹k

‚ ≥3

2a b+c

+

2b c+a

+

2c a+b

Œk

3

≥ 3.

Consider now that k0 ≤ k < 1. Due to homogeneity, we may assume that a + b + c = 1. Thus, we need to show that f (a) + f (b) + f (c) ≥ 3 f (s), where 2u f (u) = 1−u 

‹k

,

From f 00 (u) =

4k (1 − u)4



2u 1−u

s=

a+b+c 1 = , 3 3

u ∈ [0, 1). ‹k−2

(2u + k − 1),

58

Vasile Cîrtoaje

it follows that f is convex on [1/3, 1) (since u ≥ 1/3 involves 2u + k − 1 ≥ 2/3 + k − 1 = k −1/3 > 0). By HCF Theorem, it suffices to prove the original homogeneous inequality for b = c = 1; that is, to show that h(a) ≥ 3 for all a ≥ 0, where  ‹k 2 k h(a) = a + 2 . a+1 For a > 0, the derivative 0

h (a) = ka

k−1

2 −k a+1 

‹k+1

has the same sign as g(a) = (k − 1) ln a − (k + 1) ln From g 0 (a) =

2 . a+1

2ka + k − 1 , a(a + 1)

it follows that g 0 (a) = 0 for a0 = (1 − k)/(2k) < 1, g 0 (a) < 0 for a ∈ (0, a0 ) and g 0 (a) > 0 for a ∈ (a0 , ∞). Then, g is strictly decreasing on (0, a0 ] and strictly increasing on (a0 , ∞). Since lima→0 g(a) = ∞ and g(1) = 0, there exists a1 ∈ (0, a0 ) such that g(a1 ) = 0, g(a) > 0 for a ∈ (0, a1 ) ∪ (1, ∞) and g(a) < 0 for a ∈ (a1 , 1); hence, h(a) is strictly increasing on [0, a1 ] ∪ [1, ∞) and strictly decreasing on [a1 , 1]. Consequently, h(a) ≥ min{h(0), h(1)}. Since h(0) = 2k+1 ≥ 3 and h(1) = 3, we get h(a) ≥ 3. The proof is completed. The equality holds for a = b = c. If k = k0 , then the equality holds also for a = 0 and b = c (or any cyclic permutation). Remark. For k = 2/3, we can give the following solution (based on the AM-GM inequality): X  2a ‹2/3 X 2a = p 3 b+c 2a · (b + c) · (b + c) X 6a ≥ = 3. 2a + (b + c) + (b + c)

p P 1.40. If a, b, c ∈ [1, 7 + 4 3], then v v v t 2b t 2a t 2c + + ≥ 3. b+c c+a a+b (Vasile Cirtoaje, 2007)

Half Convex Function Method

59

Solution. Denoting s=

a+b+c , 3

we need to show that f (a) + f (b) + f (c) ≥ 3 f (s), where

v t 2u f (u) = , 1 ≤ u < 3s. 3s − u

For u ≥ s, we have 3s − u f (u) = 3s 2u 00



‹3/2

4u − 3s > 0. (3s − u)4

Therefore, f is convex for u ≥ s. By HCF Theorem, it suffices to prove the original inequality for b = c; that is, v s t 2b a +2 ≥ 3. b a+b v tb p Putting t = , the condition a, b ∈ [1, 7 + 4 3] involves a p p 2 − 3 ≤ t ≤ 2 + 3. We need to show that

v t 2t 2 1 2 ≥3− . t2 + 1 t

This is true if

 ‹ 8t 2 1 2 ≥ 3− . t2 + 1 t This is equivalent to the obvious inequality (t − 1)2 (t 2 − 4t + 1) ≥ 0.

The proof is completed. In accord p with Remark 3, the equality holds for a = b = c, and also for a = 1 and b = c = 7 + 4 3 (or any cyclic permutation).

P 1.41. Let a, b, c be nonnegative real numbers such that a + b + c = 3. If 0 < k ≤ k0 ,

k0 =

ln 2 ≈ 1.71, ln 3 − ln 2

then a k (b + c) + b k (c + a) + c k (a + b) ≤ 6.

60

Vasile Cîrtoaje

Solution. For 0 < k ≤ 1, the inequality follows from Jensens’s inequality applied to the convex function f (u) = uk : ˜ • (b + c)a + (c + a)b + (a + b)c k (b + c)a k + (c + a)b k + (a + b)c k ≤ 2(a + b + c) 2(a + b + c)  ‹  ‹ a b + bc + ca k a + b + c 2k =6 ≤6 = 6. 3 3 Consider now that 1 < k ≤ k0 , and write the inequality as f (a) + f (b) + f (c) ≥ 3 f (s), s =

a+b+c = 1, 3

where f (u) = uk (u − 3), u ∈ [0, 3]. For u ≥ 1, we have f 00 (u) = kuk−2 [(k + 1)u − 3k + 3] ≥ kuk−2 [(k + 1) − 3k + 3] = 2k(2 − k)uk−2 > 0; therefore, f is convex for u ≥ 1. By HCF Theorem, it suffices to consider the case where two of a, b, c are equal. Write the desired inequality in the homogeneous form  ‹ a + b + c k+1 k k k a (b + c) + b (c + a) + c (a + b) ≤ 6 . 3 Since this inequality is trivial for b = c = 0, we may consider that b = c = 1. So, we need to show that g(a) ≥ 0 for a ≥ 0, where  ‹ a + 2 k+1 g(a) = 3 − a k − a − 1. 3 We have a+2 g (a) = (k + 1) 3 0



‹k

− ka

k−1

− 1,

1 00 k+1 g (a) = k 3



a+2 3

‹k−1



k−1 . a2−k

Since g 00 is strictly increasing, lima→0 g(a) = −∞ and g 00 (1) = 2k(2 − k)/3 > 0, there exists a1 ∈ (0, 1) such that g 00 (a1 ) = 0, g 00 (a) < 0 for a ∈ (0, a1 ), and g 00 (a) > 0 for a > 1. Therefore, g 0 is strictly decreasing on [0, a1 ] and strictly increasing on [a1 , ∞). Since g 0 (0) = (k + 1)(2/3)k − 1 ≥ (k + 1)(2/3)k0 − 1 =

k+1 k−1 −1= > 0, 2 2

g 0 (1) = 0,

there exists a2 ∈ (0, a1 ) such that g 0 (a2 ) = 0, g 0 (a) > 0 for a ∈ [0, a2 ) ∪ (1, ∞), and g 0 (a) < 0 for a ∈ (a2 , 1). Thus, g is strictly increasing on [0, a2 ] ∪ [1, ∞) and strictly decreasing on [a2 , 1]. Consequently, g(a) ≥ min{g(0), g(1)},

Half Convex Function Method

61

and from g(0) = 3(2/3)k+1 − 1 ≥ 3(2/3)k0 +1 − 1 = 1 − 1 = 0,

g(1) = 0,

we get g(a) ≥ 0. This completes the proof. The equality holds for a = b = c = 1. If k = k0 , then the equality holds also for a = 0 and b = c = 3/2 (or any cyclic permutation). Remark 1. Using the Cauchy-Schwarz inequality, we get X

(a + b + c)2 9 3 a P ≥ =P ≥ . k k k k k b +c a(b + c ) a (b + c) 2

Thus, the following statement holds. • Let a, b, c be nonnegative real numbers such that a + b + c = 3. If 0 < k ≤ k0 , then

k0 =

ln 2 ≈ 1.71, ln 3 − ln 2

b c 3 a + + ≥ , bk + c k c k + ak ak + bk 2

with equality for a = b = c = 1. If k = k0 , then the equality holds also for a = 0 and b = c = 3/2 (or any cyclic permutation). Remark 2. Also, the following statement holds. • Let a, b, c be nonnegative real numbers such that a + b + c = 3. If k ≥ k1 ,

k1 =

ln 9 − ln 8 ≈ 0.2905, ln 3 − ln 2

then ak bk ck 3 + + ≥ , b+c c+a a+b 2 with equality for a = b = c = 1. If k = k1 , then the equality holds also for a = 0 and b = c = 3/2 (or any cyclic permutation). For k1 ≤ k ≤ 2, the inequality can be proved using the Cauchy-Schwarz inequality, as follows: X ak (a + b + c)2 9 3 ≥P =P = . 2−k 2−k b+c a (b + c) a (b + c) 2

62

Vasile Cîrtoaje

For k > 2, the inequality can be deduced from the Cauchy-Schwarz inequality and Bernoulli’s inequality, as follows: P k/2 2 P k/2 2 X ak a a ≥ P = , b+c 6 (b + c) ˜ X X• k k/2 a ≥ 1 + (a − 1) = 3. 2

P 1.42. Let a1 , a2 , . . . , an be nonnegative real numbers such that a1 + a2 + · · · + an = n. If n2 , then k≥ 4(n − 1) (a12 + k)(a22 + k) · · · (an2 + k) ≥ (1 + k)n . Solution. Write the inequality as f (a1 ) + f (a2 ) + · · · + f (an ) ≥ n f (s),

s=

a1 + a2 + · · · + an = 1, n

where f (u) = ln(u2 + k),

u ∈ [0, n].

We have f 0 (u) =

2u , u2 + k

f 00 (u) =

2(k − u2 ) . (u2 + k)2

For u ∈ [0, 1], we have f 00 (u) > 0, since k − u2 ≥ k − 1 ≥

n2 − 1 ≥ 0. 4(n − 1)

Therefore, f is convex on [0, 1]. By HCF Theorem and Remark 2, it suffices to prove that H(x, y) ≥ 0 for x, y ≥ 0 such that x + (n − 1) y = n, where H(x, y) =

f 0 (x) − f 0 ( y) . x−y

Since

2(k − x y) 2[n2 − 4(n − 1)x y] ≥ , (x 2 + k)( y 2 + k) 4(n − 1)(x 2 + k)( y 2 + k) we only need to show that n2 ≥ 4(n − 1)x y. H(x, y) =

Indeed, we have n2 = [x + (n − 1) y]2 ≥ 4(n − 1)x y. The equality holds for a1 = a2 = · · · = an = 1.

Half Convex Function Method

63

P 1.43. Let a, b, c be nonnegative real numbers such a + b + c = 3. If k ≥ k0 , where p p p 6−2 = (2 + 2)(2 + 3) ≈ 12.74 , k0 = p p 6− 2−1 then p

a+

p

b+

p

c−3≥ k

‚v ta + b 2

+

v tb+c 2

+

s

Œ c+a −3 . 2 (Vasile Cirtoaje, 2008)

Solution. By the Cauchy-Schwarz inequality (1 + 1 + 1)[(a + b) + (b + c) + (c + a)] ≥

€p

a+b+

p

b+c+

p

c+a

Š2

,

we get v ta + b 2

+

v tb+c 2

+

s

c+a ≤ 3. 2

Therefore, it suffices to consider the case k = k0 . Write the inequality as f (a) + f (b) + f (c) ≥ 3 f (s), where

s=

v t3 − u p f (u) = u − k0 , 2

a+b+c = 1, 3

u ∈ [0, 3).

For u ≥ 1, we have 00

4 f (u) = −u

−3/2

k0 + 4



3−u 2

‹−3/2

≥ −1 +

k0 > 0. 4

Therefore, f is convex on [1, 3]. By HCF Theorem and Remark 5, it suffices to consider only the case a ≤ 1 ≤ b = c. Write the original inequality in the homogeneous form v v v ‚v Œ ta + b t b + c sc + a ta + b + c ta + b + c p p p a+ b+ c−3 ≥ k0 + + −3 . 3 2 2 2 3 Due to homogeneity, we may assume that b = c = 1. Moreover, it is convenient to use p the substitution x = a. Thus, we need to show that g(x) ≥ 0 for x ∈ [0, 1], where v v t x2 + 1 t x2 + 2 g(x) = x + 2 − k0 + 3(k0 − 1) − 2k0 . 3 2 We have 0

g (x) = 1 + (k0 − 1)x

v t

v t 2 3 − k x , 0 x2 + 2 x2 + 1

64

Vasile Cîrtoaje k0 g 00 (x) = 2



2 2 x +1

‹3/2 –

x2 + 1 m· 2 x +2

3/2

™ −1 ,

where v u  ‹ t 1 2 3 m= 6 1− ≈ 1.72. k0 Clearly, g 00 (x) has the same sign as h(x), where x2 + 1 − 1. x2 + 2

h(x) = m · Since h(0) =

m − 1 < 0, 2

h(1) =

2m − 1 > 0, 3

there is x 1 ∈ (0, 1) such that h(x) < 0 for x ∈ [0, x 1 ), h(x 1 ) = 0 and h(x) > 0 for x ∈ (x 1 , 1]. Therefore, g 0 is strictly decreasing on [0, x 1 ] and strictly increasing on [x 1 , 1]. Since g 0 (0) = 1 and g 0 (1) = 0, there is x 2 ∈ (0, x 1 ) such that g 0 (x) > 0 for x ∈ [0, x 2 ), g 0 (x 2 ) = 0 and g 0 (x) < 0 for x ∈ (x 2 , 1). Thus, g(x) is strictly p increasing p on [0, x 2 ] and strictly decreasing on [x 2 , 1]. From g(0) = 2 − k0 + (k0 − 1) 6 − k0 2 = 0 and g(1) = 0, it follows that g(x) ≥ 0 for x ∈ [0, 1]. This completes the proof. The equality holds for a = b = c. If k = k0 , then the equality holds also for a = 0 and b = c = 3/2 (or any cyclic permutation).

P 1.44. Let a, b, c be nonnegative real numbers such a + b + c = 3. If k ≤ k1 , where p p p k1 = ( 3 − 1)( 3 + 2) ≈ 2.303 , then p

a+

p

b+

p

c−3≤ k

‚v ta + b 2

+

v tb+c 2

+

s

Œ c+a −3 . 2 (Vasile Cirtoaje, 2008)

Solution. By the Cauchy-Schwarz inequality (1 + 1 + 1)[(a + b) + (b + c) + (c + a)] ≥

€p

a+b+

p

we get v ta + b 2

+

v tb+c 2

+

s

c+a ≤ 3. 2

b+c+

p

c+a

Š2

,

Half Convex Function Method

65

Therefore, it suffices to consider the case k = k1 . Write the inequality as f (a) + f (b) + f (c) ≥ 3 f (s), where

s=

v t3 − u p f (u) = − u + k1 , 2

a+b+c = 1, 3

u ∈ [0, 3).

For 0 ≤ u ≤ 1, we have 4 f 00 (u) = u−3/2 −

k1 4



‹−3/2

3−u 2

≥1−

k1 > 0. 4

Therefore, f is convex on [0, 1]. By HCF Theorem and Remark 5, it suffices to consider only the case a ≥ 1 ≥ b = c. Write the original inequality in the homogeneous form p

a+

p

b+

p

c−3

v ta + b + c 3

≤ k1

‚v ta + b 2

+

v tb+c 2

+

s

v Œ ta + b + c c+a −3 . 2 3

Due to homogeneity, we may assume that b = c = 1. Moreover, it is convenient to use p the substitution x = a. Thus, we need to show that g(x) ≤ 0 for x ≥ 1, where v v t x2 + 1 t x2 + 2 − 2k1 . g(x) = x + 2 − k1 + 3(k1 − 1) 3 2 We have

v t

v t 2 3 g (x) = 1 + (k1 − 1)x − k x , 1 x2 + 2 x2 + 1 ™ 3/2  ‹3/2 – 2 k 2 x + 1 1 g 00 (x) = m· 2 −1 , 2 x2 + 1 x +2 0

where

v u  ‹ t 1 2 3 m= 6 1− ≈ 1.2431 . k1

Clearly, g 00 (x) has the same sign as h(x), where h(x) = m ·

x2 + 1 − 1. x2 + 2

Since h is strictly increasing on [1, ∞) and h(1) =

2m − 1 < 0, 3

lim h1 (x) = m − 1 > 0,

x→∞

66

Vasile Cîrtoaje

there is x 1 ∈ (1, ∞) such that h(x) < 0 for x ∈ [1, x 1 ), h(x 1 ) = 0 and h(x) > 0 for x ∈ (x 1 , ∞). Therefore, g 0 is strictly decreasing on [1, x 1 ] and strictly increasing on [x 1 , ∞). Since g 0 (1) = 0 and lim x→∞ g 0 (x) = 0, it follows that g 0 (x) < 0 for x ∈ (1, ∞). Thus, g(x) is strictly decreasing on [1, ∞), hence g(x) ≤ g(1) = 0. This completes the proof. The equality holds for a = b = c = 1. If k = k0 , then the equality holds also for a = 0 and b = c = 3/2 (or any cyclic permutation).

P 1.45. If a, b, c are positive real numbers such that a bc = 1, then a2 + b2 + c 2 − 3 ≥ 18(a + b + c − a b − bc − ca). Solution. Using the substitution a = ex ,

b = ey,

c = ez ,

we need to show that f (x) + f ( y) + f (z) ≥ 3 f (s),

s=

x + y +z = 0, 3

where f (u) = e2u − 1 − 18(eu − e−u ), u ∈ R. For u ≤ 0, we have

f 00 (u) = 4e2u + 18(e−u − eu ) > 0,

hence f is convex on (−∞, 0]. By HCF Theorem, it suffices to prove the original inequality for b = c := t and a = 1/t 2 , where t > 0. Since a2 + b2 + c 2 − 3 = and a + b + c − a b − bc − ca =

1 (t 2 − 1)2 (2t 2 + 1) 2 + 2t − 3 = t4 t4 −(t 4 − 2t 3 + 2t − 1) −(t − 1)3 (t + 1) = , t2 t2

we get a2 + b2 + c 2 − 3 − 18(a + b + c − a b − bc − ca) =

(t − 1)2 (2t − 1)2 (t + 1)(5t + 1) ≥ 0. t4

The equality holds for a = b = c = 1, and also for a = 4 and b = c = 1/2 (or any cyclic permutation).

Half Convex Function Method

67

P 1.46. If a, b, c are positive real numbers such that a bc = 1, then p p p a2 − a + 1 + b2 − b + 1 + c 2 − c + 1 ≥ a + b + c. Solution. Using the substitution a = ex ,

b = ey,

c = ez ,

we need to show that f (x) + f ( y) + f (z) ≥ 3 f (s), where f (u) =

p

s=

x + y +z = 0, 3

e2u − eu + 1 − eu , u ∈ R.

We claim that f is convex for u ≥ 0. Since e−u f 00 (u) =

4e3u − 6e2u + 9eu − 2 − 1, 4(e2u − eu + 1)3/2

we need to show that (4x 3 − 6x 2 + 9x − 2)2 ≥ 16(x 2 − x + 1)3 , where x = eu ≥ 1. Indeed, (4x 3 − 6x 2 + 9x − 2)2 − 16(x 2 − x + 1)3 = 12x 3 (x − 1) + 9x 2 + 12(x − 1) > 0. By HCF Theorem, it suffices to prove the original inequality for b = c := t and a = 1/t 2 , where t > 0; that is, p p t4 − t2 + 1 1 + 2 t 2 − t + 1 ≥ 2 + 2t, t2 t p

t2 − 1 t4 − t2 + 1 + 1

Since p it suffices to show that

+p

t2 − 1 t4 − t2 + 1

2(1 − t) t2 − t + 1 + t ≥

≥ 0.

t2 − 1 , t2 + 1

t2 − 1 2(1 − t) +p ≥ 0, 2 t +1 t2 − t + 1 + t

which is equivalent to ˜ t +1 2 (t − 1) 2 −p ≥ 0, t +1 t2 − t + 1 + t •

68

Vasile Cîrtoaje ” — p (t − 1) (t + 1) t 2 − t + 1 − t 2 + t − 2 ≥ 0, (t − 1)2 (3t 2 − 2t + 3) ≥ 0. p (t + 1) t 2 − t + 1 + t 2 − t + 2

Clearly, the last inequality is true. The equality holds for a = b = c = 1.

P 1.47. If a, b, c, d ≥

1 p such that a bcd = 1, then 1+ 6 1 1 1 1 4 + + + ≤ . a+2 b+2 c+2 d +2 3 (Vasile Cirtoaje, 2005)

Solution. Using the substitution a = ex ,

b = ey,

c = ez ,

d = ew,

we need to show that f (x) + f ( y) + f (z) + f (w) ≥ 4 f (s), where f (u) = For u ≤ 0, we have f 00 (u) =

s=

x + y +z+w = 0, 4

−1 , u ∈ R. +2

eu

eu (2 − eu ) > 0, (eu + 2)3

hence f is convex on (−∞, 0]. By HCF Theorem, it suffices to prove the original in1 equality for b = c = d := t and a = 1/t 3 , where t ≥ p ; that is, 1+ 6 t3 3 4 + ≤ , 3 2t + 1 t + 2 3 which is equivalent to the obvious inequality (t − 1)2 (5t 2 + 2t − 1) ≥ 0. In accord with Remark 3, the equality holds for a = b = c = d = 1, and also for p 1 a = 19 + 9 6 and b = c = d = p (or any cyclic permutation). 1+ 6

Half Convex Function Method

69

P 1.48. If a1 , a2 , . . . , an are positive real numbers such that a1 a2 · · · an = 1, then  ‹ p 1 1 1 2 2 2 . − − ··· − a1 + a2 + · · · + an − n ≥ 6 3 a1 + a2 + · · · + an − a1 a2 an Solution. Using the substitutions ai = e x i for i = 1, 2, . . . , n, we need to show that f (x 1 ) + f (x 2 ) + · · · + f (x n ) ≥ n f (s), where

s=

x1 + x2 + · · · + x n = 0, n

p f (u) = e2u − 1 − 6 3 (eu − e−u ), u ∈ R.

For u ≤ 0, we have

p f 00 (u) = 4e2u + 6 3(e−u − eu ) > 0,

hence f is convex on (−∞, 0]. By HCF Theorem and Remark 2, it suffices to show that H(x, y) ≥ 0 for x, y ∈ R such that x + (n − 1) y = 0, where H(x, y) = From

f 0 (x) − f 0 ( y) . x−y

p f 0 (u) = 2e2u − 6 3 (eu + e−u ),

we get H(x, y) =

 p p 2(e x − e y ) x e + e y − 3 3 + 3 3 e−x e− y . x−y

Since (e x − e y )/(x − y) > 0, we need to prove that p p e x + e y + 3 3 e−x e− y ≥ 3 3. Indeed, by the AM-GM inequality, we have Æ p p p 3 e x + e y + 3 3 e−x e− y ≥ 3 e x · e y · 3 3 e−x e− y = 3 3. The proof is completed. The equality holds for a1 = a2 = · · · = an = 1.

P 1.49. If a1 , a2 , . . . , an (n ≥ 4) are positive real numbers such that a1 a2 · · · an = 1, then (n − 1)(a12 + a22 + · · · + an2 ) + n(n + 3) ≥ (2n + 2)(a1 + a2 + · · · + an ).

70

Vasile Cîrtoaje

Solution. Using the substitutions ai = e x i for i = 1, 2, . . . , n, we need to show that f (x 1 ) + f (x 2 ) + · · · + f (x n ) ≥ n f (s),

s=

x1 + x2 + · · · + x n = 0, n

where f (u) = (n − 1)e2u − (2n + 2)eu , u ∈ R. For u ≥ 0, we have f 00 (u) = 4(n − 1)e2u − (2n + 2)eu = 2eu [2(n − 1)eu − n − 1] ≥ 2eu [2(n − 1) − n − 1] = 2(n − 3)eu > 0. Therefore, f is convex on [0, ∞). By HCF Theorem and Remark 2, it suffices to show that H(x, y) ≥ 0 for x, y ∈ R such that x + (n − 1) y = 0, where H(x, y) =

f 0 (x) − f 0 ( y) . x−y

From f 0 (u) = 2(n − 1)e2u − (2n + 2)eu , we get H(x, y) =

2(e x − e y ) [(n − 1)(e x + e y ) − (n + 1)] . x−y

Since (e x − e y )/(x − y) > 0, we need to prove that (n − 1)(e x + e y ) ≥ n + 1. Using the AM-GM inequality, we have Æ n (n − 1)(e x + e y ) = (n − 1)e x + e y + e y + · · · + e y ≥ n (n − 1)e x · e y · e y · · · e y Æ p n n = n (n − 1)e x+(n−1) y = n n − 1. Thus, it suffices to show that which is equivalent to

p n n n − 1 ≥ n + 1,  ‹ 1 n n−1≥ 1+ . n

This is true for n ≥ 4, since

 ‹ 1 n n−1≥3> 1+ . n The proof is completed. The equality holds for a1 = a2 = · · · = an = 1. Remark. From the proof above, it follows that the following sharper inequality holds in the same conditions (Gabriel Dospinescu and Calin Popa): p n 2n n − 1 2 2 2 a1 + a2 + · · · + an − n ≥ (a1 + a2 + · · · + an − n). n−1

Half Convex Function Method

71

P 1.50. Let a, b, c, d be positive real numbers such that a bcd = 1. If p and q are nonnegative real numbers such that p + q = 3, then then 1 1 1 1 + + + ≥ 1. 1 + pa + qa3 1 + p b + q b3 1 + pc + qc 3 1 + pd + qd 3 (Vasile Cirtoaje, 2007) Solution. Using the substitutions a = ex ,

b = ey,

c = ez ,

d = ew,

we need to show that f (x) + f ( y) + f (z) + f (w) ≥ 4 f (s), where f (u) =

1 1+

peu

+ qe3u

s=

x + y +z+w = 0, 4

, u ∈ R.

We will show that f 00 (u) > 0 for u ≥ 0, hence f is convex on [0, ∞). Since f 00 (u) =

th(t) , (1 + pt + qt 3 )3

where h(t) = 9q2 t 5 + 2pqt 3 − 9qt 2 + p2 t − p,

t = eu , t ≥ 1,

we need to show that h(t) > 0 for t ≥ 1. Indeed, we have h(t) ≥ 9q2 t 3 + 2pqt 3 − 9qt 2 + p2 t − pt = t[(9q2 + 2pq)t 2 − 9qt + p2 − p] and (9q2 + 2pq)t 2 − 9qt + p2 − p ≥ (9q2 + 2pq)(2t − 1) − 9qt + p2 − p = q(18q + 4p − 9)t − 9q2 − 2pq + p2 − p ≥ q(18q + 4p − 9) − 9q2 − 2pq + p2 − p = p2 + 2pq + 9q2 − p − 9q = p2 + 2pq + 9q2 − =

(p + 9q)(p + q) 3

2(p − q)2 + 16q2 > 0. 3

By HCF Theorem, it suffices to prove the original inequality for b = c = d = t and a = 1/t 3 , where t > 0; that is, 3 t9 + ≥ 1, 9 6 t + pt + q 1 + pt + qt 3

72

Vasile Cîrtoaje pt 6 + q 3 ≥ , 1 + pt + qt 3 t 9 + pt 6 + q (3 − pq)t 9 − p2 t 7 + 2pt 6 − q2 t 3 − pqt + 2q ≥ 0, [(p + q)2 − 3pq]t 9 − 3p2 t 7 + 2p(p + q)t 6 − 3q2 t 3 − 3pqt + 2q(p + q) ≥ 0, Ap2 + Bq2 ≥ C pq,

where A = t 9 − 3t 7 + 2t 6 = t 6 (t − 1)2 (t + 2) ≥ 0, B = t 9 − 3t 3 + 2 = (t 3 − 1)2 (t 3 + 2) ≥ 0, C = t 9 − 2t 6 + 3t − 2. Since A ≥ 0 and B ≥ 0, it suffices to consider that C ≥ 0 and to show that 4AB ≥ C 2 . From t 3 − 3t + 2 = (t − 1)2 (t + 2) ≥ 0, we get 3t − 2 ≤ t 3 . Therefore C ≤ t 9 − 2t 6 + t 3 = t 3 (t 3 − 1)2 , hence 4AB − C 2 ≥ 4AB − t 6 (t 3 − 1)4 = t 6 (t − 1)2 (t 3 − 1)2 [4(t + 2)(t 3 + 2) − (t 2 + t + 1)2 ] = t 6 (t − 1)2 (t 3 − 1)2 (3t 4 + 6t 3 − 3t 2 + 6t + 15) ≥ 0. The proof is completed. The inequality holds for a = b = c = d = 1. Remark. Similarly, we can prove the following generalization. • Let a, b, c, d be positive real numbers such that a bcd = 1. If p and q are nonnegative real numbers such that p + q ≥ 3, then then 1 1 1 1 4 + + + ≥ . 3 3 3 3 1 + pa + qa 1 + pb + qb 1 + pc + qc 1 + pd + qd 1+p+q

P 1.51. Let a1 , a2 , . . . , an (n ≥ 3) be positive real numbers such that a1 a2 . . . an = 1. If p, q ≥ 0 such that p + q ≥ n − 1, then 1 1+

pa1 + qa12

+

1 1+

pa2 + qa22

+ ··· +

1 n ≥ . 2 1 + pan + qan 1+p+q (Vasile Cirtoaje, 2007)

Half Convex Function Method

73

Solution. Using the substitutions ai = e x i for i = 1, 2, . . . , n, we need to show that f (x 1 ) + f (x 2 ) + · · · + f (x n ) ≥ n f (s), where f (u) =

1 1+

peu

+ qe2u

s=

x1 + x2 + · · · + x n = 0, n

, u ∈ R.

For u ≥ 0, we have eu [4q2 e3u + 3pqe2u + (p2 − 4q)eu − p] (1 + peu + qe2u )3 2u 2 e [4q + 3pq + (p2 − 4q) − p] ≥ (1 + peu + qe2u )3 e2u [(p + 2q)(p + q − 2) + 2q2 + p] = > 0, (1 + peu + qe2u )3

f 00 (u) =

therefore f (u) is convex for u ≥ 0. By HCF Theorem, it suffices to prove the original inequality for a2 = · · · = an := t and a1 = 1/t n−1 , where t > 0. Write this inequality as t 2n−2 n−1 n + ≥ . 2n−2 n−1 2 t + pt + q 1 + pt + qt 1+p+q Applying the Cauchy-Schwarz inequality, it suffices to prove that n (t n−1 + n − 1)2 ≥ , (t 2n−2 + pt n−1 + q) + (n − 1)(1 + pt + qt 2 ) 1 + p + q which is equivalent to pB + qC ≥ A, where A = (n − 1)(t n−1 − 1)2 ≥ 0, A + nE, E = t n−1 + n − 2 − (n − 1)t, n−1 A C = (t n−1 − 1)2 + nF = + nF, F = 2t n−1 + n − 3 − (n − 1)t 2 . n−1 By the AM-GM inequality applied to n − 1 positive numbers, we have E ≥ 0 and F ≥ 0 for n ≥ 3. Since A ≥ 0 and p + q ≥ n − 1, we have B = (t n−1 − 1)2 + nE =

pB + qC − A ≥ pB + qC −

(p + q)A = n(pE + qF ) ≥ 0. n−1

The equality holds for a1 = a2 = · · · = an = 1.

74

Vasile Cîrtoaje

Remark 1. For p = 2k and q = k2 , we get the following result. • Let a1 , a2 , · · · , an (n ≥ 3) be positive real numbers such that a1 a2 · · · an = 1. If p k ≥ n − 1, then 1 1 n 1 + + ··· + ≥ , (1 + ka1 )2 (1 + ka2 )2 (1 + kan )2 (1 + k)2 with equality for a1 = a2 = · · · = an = 1. In addition, for n = 4 and k = 1, we get the known inequality (Vasile Cirtoaje, 1999): 1 1 1 1 + + + ≥ 1, 2 2 2 (1 + a) (1 + b) (1 + c) (1 + d)2 where a, b, c, d > 0 such that a bcd = 1. Remark 2. For p + q = n − 1, we get the beautiful inequality 1 1+

pa1 + qa12

+

1 1+

pa2 + qa22

+ ··· +

1 ≥ 1, 1 + pan + qan2

n ≥ 3,

which is a generalization of the following inequalities: 1 1 1 + + ··· + ≥ 1, 1 + (n − 1)a1 1 + (n − 1)a2 1 + (n − 1)an 1 1 1 + + · · · + ≥ 1, p p p [1 + ( n − 1)a1 ]2 [1 + ( n − 1)a1 ]2 [1 + ( n − 1)a1 ]2 1 2 + (n − 1)(a1 + a12 )

+

1 2 + (n − 1)(a2 + a22 )

+ ··· +

1 1 ≥ . 2 2 + (n − 1)(an + an ) 2

Remark 3. Similarly, we can prove the following statement: • Let a1 , a2 , . . . , an (n ≥ 4) be positive real numbers such that a1 a2 . . . an = 1. If p, q, r ≥ 0 such that p + q + r ≥ n − 1, then n X i=1

1 1+

pai + qai2

+

r ai3



n . 1+p+q+r

For n = 4 and p + q + r = 3, we get the beautiful inequality 4 X

1

i=1

1 + pai + qai2 + r ai3

≥ 1.

Half Convex Function Method

75

If p = q = r = 1, then we get the known inequality (Vasile Cirtoaje, 1999): 4 X

1

i=1

1 + ai + ai2 + ai3

≥ 1.

Since 2ai2 ≤ ai + ai3 , the best inequality with respect to q if for q = 0; that is, 4 X

1

i=1

1 + pai + r ai3

p + r = 3.

≥ 1,

For p = 1 and p = 2, we get the following strong inequalities: 4 X

1

i=1

1 + ai + 2ai3

4 X

1

i=1

1 + 2ai + ai3

≥ 1,

≥ 1.

Actually, the following generalization holds. • Let a1 , a2 , . . . , an be positive real numbers such that a1 a2 . . . an = 1, and let k1 , k2 , . . . , km ≥ 0 such that k1 + k2 + · · · + km ≥ n − 1. If m ≤ n − 1, then n X

1

1 + k1 ai + k2 ai2 i=1

+ · · · + km aim



n 1 + k1 + k2 + · · · + km

.

(*)

For m = n − 1 and k1 = k2 = · · · = km = 1, (*) turns into the known beautiful inequality n X

1

1 + ai + ai2 i=1

+ · · · + ain−1

≥ 1.

Since (m − 1)aik ≤ (m − k)ai + (k − 1)aim ,

k = 2, 3, . . . , m − 1

(by the AM-GM inequality), the best inequality (*) with respect to k2 , · · · , km−1 is for k2 , · · · , km−1 = 0; that is, n X i=1

n 1 , m ≥ 1 + k1 ai + km ai 1 + k1 + km

k1 + km ≥ n − 1, 1 ≤ m ≤ n − 1.

If k1 + km = n − 1 and m = n − 1, then n X

1

i=1

1 + k1 ai + kn−1 ain−1

≥ 1.

76

Vasile Cîrtoaje

For k1 = 1 and k1 = n − 2, we get the following strong inequalities: n X

1

i=1

1 + ai + (n − 2)ain−1

n X

1

i=1

1 + (n − 2)ai + ain−1

≥ 1,

≥ 1.

P 1.52. Let a1 , a2 , . . . , an be positive real numbers such that a1 a2 . . . an = 1. If k ≥ n2 − 1, then 1 1 1 n . +p + ··· + p ≥p p 1+k 1 + kan 1 + ka1 1 + ka2 Solution. Using the substitutions ai = e x i for i = 1, 2, . . . , n, we need to show that f (x 1 ) + f (x 2 ) + · · · + f (x n ) ≥ n f (s), where

s=

x1 + x2 + · · · + x n = 0, n

1 , u ∈ R. f (u) = p 1 + keu

For u ≥ 0, we have f 00 (u) =

keu (keu − 2) keu (k − 2) ≥ > 0. 4(1 + keu )5/2 4(1 + keu )5/2

Therefore, f is convex on [0, ∞). By HCF Theorem and Remark 5, it suffices to prove the original inequality for a2 = · · · = an := t and a1 = 1/t n−1 , where t ≥ 1. Write this inequality as h(t) ≥ 0, where v t t n−1 n−1 n h(t) = +p −p . n−1 t +k 1 + kt 1+k The derivative h0 (t) =

(n − 1)kt (n−3)/2 (n − 1)k − n−1 3/2 2(t + k) 2(kt + 1)3/2

has the same sign as h1 (t) = t n/3−1 (kt + 1) − t n−1 − k. Denoting m = n/3, m ≥ 2/3, we see that h1 (t) = kt m + t m−1 − t 3m−1 − k = k(t m − 1) − t m−1 (t 2m − 1) = (t m − 1)h2 (t),

Half Convex Function Method

77

where h2 (t) = k − t m−1 − t 2m−1 . For t > 1, we have h02 (t) = t m−2 [−m + 1 − (2m − 1)t m ] < t m−2 [−m + 1 − (2m − 1)] = −(3m − 2)t m−2 ≤ 0, hence h2 (t) is strictly decreasing for t ≥ 1. Since h2 (1) = k − 2 > 0 and lim t→∞ h2 (t) = −∞, there exists t 1 > 1 such that h2 (t 1 ) = 0, h2 (t) > 0 for t ∈ (1, t 1 ), and h2 (t) < 0 for t ∈ (t 1 , ∞). Since h2 , h1 and h0 has the same sign for t > 1, h(t) is strictly increasing for t ∈ [1, t 1 ] and strictly decreasing for t ∈ [t 1 , ∞); this yields h(t) ≥ min{h(1), h(∞)}. n ≥ 0, it follows that h(t) ≥ 0 for all t ≥ 1. The From h(1) = 0 and h(∞) = 1 − p 1+k proof is completed. The equality holds for a1 = a2 = · · · = an = 1. Remark. The following generalization holds (Vasile Cirtoaje, 2005): • Let a1 , a2 , · · · , an be positive real numbers such that a1 a2 · · · an = 1. If k and m are positive numbers such that m ≤ n − 1,

k ≥ n1/m − 1,

then 1 1 1 n + + ··· + ≥ , m m m (1 + ka1 ) (1 + ka2 ) (1 + kan ) (1 + k)m with equality for a1 = a2 = · · · = an = 1. For 0 < m ≤ n − 1 and k = n1/m − 1, we get the beautiful inequality 1 1 1 + + ··· + ≥ 1. m m (1 + ka1 ) (1 + ka2 ) (1 + kan )m

P 1.53. If a1 , a2 , . . . , an are positive real numbers such that a1 a2 . . . an = 1, then 1 1 + a1 + · · · + a1n−1

+

1 1 + a2 + · · · + a2n−1

+ ··· +

1 ≥ 1. 1 + an + · · · + ann−1 (Vasile Cirtoaje, 2007)

78

Vasile Cîrtoaje

Solution. Using the substitutions ai = e x i for i = 1, 2, . . . , n, we need to show that f (x 1 ) + f (x 2 ) + · · · + f (x n ) ≥ n f (s), where f (u) =

s=

x1 + x2 + · · · + x n = 0, n

1 , u ∈ R. 1 + eu + · · · + e(n−1)u

We will show that f is convex for u ≥ 0. Setting t = eu , t ≥ 1, the necessary and sufficient condition f 00 (u) ≥ 0 for u ≥ 0 is equivalent to 2A2 ≥ B(1 + C), where A = t + 2t 2 + · · · + (n − 1)t n−1 , B = t + 4t 2 + · · · + (n − 1)2 t n−1 , C = t + t 2 + · · · + t n−1 . We will prove this inequality by induction on n. For n = 2, the inequality becomes t(t − 1) ≥ 0, which is clearly true for t ≥ 1. Assume now that the inequality is true for n and prove it for n + 1, n ≥ 2. So, we need to show that 2A2 ≥ B(1 + C) involves 2(A + nt n )2 ≥ (B + n2 t n )(1 + C + t n ), which is equivalent to 2A2 − B(1 + C) + t n [n2 (t n − 1) + D] ≥ 0, where 2

D = 2nA − B − n C =

n−1 X

bi t i ,

bi = 3n2 − (2n − i)2 .

i=1

Since 2A − B(1 + C) ≥ 0, it suffices to show that D ≥ 0. Since 2

b1 < b2 < · · · < bn−1 ,

t ≤ t 2 ≤ · · · ≤ t n−1 ,

we may apply Chebyshev’s inequality to get D≥

1 (b1 + b2 + · · · + bn−1 )(t + t 2 + · · · + t n−1 ). n

Thus, it suffices to show that b1 + b2 + · · · + bn−1 ≥ 0. Indeed, b1 + b2 + · · · + bn−1 =

n−1 X n(n − 1)(4n + 1) [3n2 − (2n − i)2 ] = > 0. 6 i=1

Half Convex Function Method

79

By HCF Theorem and Remark 5, it suffices to prove the original inequality for a2 = · · · = an := t and a1 = 1/t n−1 , where t ≥ 1. Setting k = n − 1, k ≥ 1, we need to show that 2

k tk + ≥ 1. 2 1 + t + · · · + tk 1 + tk + · · · + tk For the nontrivial case t > 1, this inequality is equivalent to the following sequence of inequalities: k 1 + t k + · · · + t (k−1)k , ≥ 1 + t + · · · + tk 1 + t k + · · · + t k2 2

k(t − 1) tk − 1 tk − 1 ≥ · , t k+1 − 1 t k − 1 t (k+1)k − 1 2

k(t − 1) tk − 1 ≥ , t k+1 − 1 t (k+1)k − 1 2

t k(k+1) − 1 tk − 1 ≥ , t k+1 − 1 t −1   k 1 + t k+1 + t 2(k+1) + · · · + t (k−1)(k+1) ≥ 1 + t + t 2 + · · · + t (k−1)(k+1) ,     k 1 · 1 + t · t k + · · · + t k−1 · t (k−1)k ≥ 1 + t + · · · + t k−1 1 + t k + · · · + t (k−1)k . k

Since 1 < t < · · · < t k−1 and 1 < t k < · · · < t (k−1)k , the last inequality follows from Chebyshev’s inequality. This completes the proof. The equality holds for a1 = a2 = · · · = an = 1.

P 1.54. Let a1 , a2 , . . . , an be positive real numbers such that a1 a2 . . . an = 1. If p, q ≥ 0 1 such that 0 < p + q ≤ , then n−1 1 1+

pa1 + qa12

+

1 1+

pa2 + qa22

+ ··· +

1 n ≤ . 1 + pan + qan2 1+p+q (Vasile Cirtoaje, 2007)

Solution. Using the substitutions ai = e x i for i = 1, 2, . . . , n, we need to show that f (x 1 ) + f (x 2 ) + · · · + f (x n ) ≥ n f (s), where f (u) =

s=

x1 + x2 + · · · + x n = 0, n

−1 , u ∈ R. 1 + peu + qe2u

80

Vasile Cîrtoaje

For u ≤ 0, we have eu [−4q2 e3u − 3pqe2u + (4q − p2 )eu + p] (1 + peu + qe2u )3 e2u [−4q2 − 3pq + (4q − p2 ) + p] ≥ (1 + peu + qe2u )3 e2u [(p + 4q)(1 − p − q) + 2pq] = ≥ 0, (1 + peu + qe2u )3

f 00 (u) =

therefore f (u) is convex for u ≤ 0. By HCF Theorem, it suffices to prove the original inequality for a2 = · · · = an := t and a1 = 1/t n−1 , where t > 0. Write this inequality as n−1 n t 2n−2 + ≤ , t 2n−2 + pt n−1 + q 1 + pt + qt 2 1+p+q p2 A + q2 B + pqC ≤ pD + qE, where A = t n−1 (t n − nt + n − 1),

B = t 2n − nt 2 + n − 1,

C = t 2n−1 + t 2n − nt n+1 + (n − 1)t n−1 − nt + n − 1, D = t n−1 [(n − 1)t n − nt n−1 + 1],

E = (n − 1)t 2n − nt 2n−2 + 1.

Applying the AM-GM inequality to n positive numbers yields D ≥ 0 and E ≥ 0. Then, since p + q ≤ 1/(n − 1) involves pD + qE ≥ (n − 1)(p + q)(pD + qE), it suffices to show that p2 A + q2 B + pqC ≤ (n − 1)(p + q)(pD + qE). Write this inequality as p2 A1 + q2 B1 + pqC1 ≥ 0, where A1 = (n − 1)D − A = nt n [(n − 2)t n−1 − (n − 1)t n−2 + 1], B1 = (n − 1)E − B = nt 2 [(n − 2)t 2n−2 − (n − 1)t 2n−4 + 1], C1 = (n − 1)(D + E) − C = n y[(n − 2)(t 2n−1 + t 2n−2 ) − 2(n − 1)t 2n−3 + t n + 1]. Applying the AM-GM inequality to n−1 nonnegative numbers yields A1 ≥ 0 and B1 ≥ 0. So, it suffices to show that C1 ≥ 0. Indeed, we have (n − 2)(t 2n−1 + t 2n−2 ) − 2(n − 1)t 2n−3 + t n + 1 = A2 + B2 + C2 , where A2 = (n − 2)t 2n−1 − (n − 1)t 2n−3 + t ≥ 0, B2 = (n − 2)t 2n−2 − (n − 1)t 2n−3 + t n−1 ≥ 0,

Half Convex Function Method

81

C2 = t n − t n−1 − t + 1 = (t − 1)(t n−1 − 1) ≥ 0. The inequalities A2 ≥ 0 and B2 ≥ 0 follows by the AM-GM inequality applied to n − 1 nonnegative numbers. This completes the proof. The equality holds for a1 = a2 = · · · = an = 1. Remark 1. For p + q =

1 , we get the inequality n−1

1 1+

pa1 + qa12

+

1 1+

pa2 + qa22

+ ··· +

1 ≤ n − 1, 1 + pan + qan2

which is a generalization of the following inequalities: 1 1 1 + + ··· + ≤ 1, n − 1 + a1 n − 1 + a2 n − 1 + an 1 2n − 2 + a1 + a12

+

1 2n − 2 + a2 + a22

+ ··· +

1 1 ≤ . 2 2n − 2 + an + an 2

Remark 2. For p = 2k and q = k2 , we get the following result: • Let a1 , a2 , · · · , an be positive real numbers such that a1 a2 · · · an = 1. If 0 0 for t ∈ (0, 1]. Also, we have h1 (t) > 0 and h0 (t) > 0 for t ∈ (0, 1). Therefore, h is strictly increasing on [0, 1], hence h(t) ≤ h(1) = 0. Case 2: h2 (0) < 0. This case is possible for either m = 1 (n = 3) and 1 < k ≤ 5/4, or m > 1 (n ≥ 4). Since h2 (1) = 2 − k > 0, there exists t 1 ∈ (0, 1) such that h2 (t 1 ) = 0, h2 (t) < 0 for t ∈ (0, t 1 ), and h2 (t) > 0 for t ∈ (t 1 , 1). Since h0 has the same sign as h2 on (0, 1), it follows that h is strictly decreasing on [0, t 1 ], and strictly increasing on [t 1 , 1]. n Therefore, h(t) ≤ min{h(0), h(1)}. Since h(0) = n − 1 − p ≤ 0 and h(1) = 0, 1+k

Half Convex Function Method

83

we have h(t) ≤ 0 for all t ∈ (0, 1]. This completes the proof. The equality holds for a1 = a2 = · · · = an = 1. Remark. The following generalization holds (Vasile Cirtoaje, 2005): • Let a1 , a2 , · · · , an (n ≥ 3) be positive real numbers such that a1 a2 · · · an = 1. If k and m are positive numbers such that 1 m≥ , n−1 then

 n 1/m k≤ − 1, n−1

1 1 1 n + + ··· + ≤ , m m m (1 + ka1 ) (1 + ka2 ) (1 + kan ) (1 + k)m

with equality for a1 = a2 = · · · = an = 1. For n ≥ 3, m ≥

 n 1/m 1 and k = − 1, we get the beautiful inequality n−1 n−1 1 1 1 + + ··· + ≤ n − 1. m m (1 + ka1 ) (1 + ka2 ) (1 + kan )m

P 1.56. If a1 , a2 , . . . , an are positive real numbers such that a1 a2 . . . an = 1, then  ‹ 1 1 1 n−1 n−1 n−1 . a1 + a2 + · · · + an + n(n − 2) ≥ (n − 1) + + ··· + a1 a2 an Solution. Using the substitutions ai = e x i for i = 1, 2, . . . , n, we need to show that f (x 1 ) + f (x 2 ) + · · · + f (x n ) ≥ n f (s), where

s=

x1 + x2 + · · · + x n = 0, n

f (u) = e(n−1)u − (n − 1)e−u , u ∈ R.

For u ≥ 0, we have f 00 (u) = (n − 1)2 e(n−1)u − (n − 1)e−u = (n − 1)e−u [(n − 1)e nu − 1] ≥ 0; therefore, f (u) is convex on [0, ∞). By HCF Theorem and Remark 2, it suffices to show that H(x, y) ≥ 0 for x, y ∈ R such that x + (n − 1) y = 0, where H(x, y) =

f 0 (x) − f 0 ( y) . x−y

84

Vasile Cîrtoaje

From

f 0 (u) = (n − 1)[e(n−1)u + e−u ],

we get  (n − 1)(e x − e y )  (n−2)x e + e(n−3)x+ y + · · · + e x+(n−3) y + e(n−2) y − e−x− y x−y  (n − 1)(e x − e y )  (n−2)x = e + e(n−3)x+ y + · · · + e x+(n−3) y ) . x−y

H(x, y) =

Since (e x − e y )/(x − y) > 0, we have H(x, y) > 0. The equality holds for a1 = a2 = · · · = an = 1.

P 1.57. Let a1 , a2 , . . . , an be positive real numbers such that a1 a2 . . . an = 1. If k ≥ n, then  ‹ 1 1 1 k k k a1 + a2 + · · · + an + kn ≥ (k + 1) + + ··· + . a1 a2 an (Vasile Cirtoaje, 2006) Solution. Using the substitutions ai = e x i for i = 1, 2, . . . , n, we need to show that f (x 1 ) + f (x 2 ) + · · · + f (x n ) ≥ n f (s),

s=

x1 + x2 + · · · + x n = 0, n

where f (u) = e ku − (k + 1)e−u , u ∈ R. For u ≥ 0, we have   f 00 (u) = k2 e ku − (k + 1)e−u = e−u k2 e(k+1)u − k − 1 ≥ e−u (k2 − k − 1) > 0; therefore, f is convex on [0, ∞). By HCF Theorem and Remark 5, it suffices to to prove the original inequality for a2 = · · · = an := b ≥ 1 and a1 := a ≤ 1, a b n−1 = 1; that is a k + (n − 1)b k −

k + 1 (k + 1)(n − 1) − + kn ≥ 0. a b

By the weighted AM-GM inequality, we have 1

a k + (kn − k − 1) ≥ [1 + (kn − k − 1)](a k ) 1+(kn−k−1) = Then, we still have to show that  ‹  ‹ 1 1 (n − 1) b k − − (k + 1) − 1 ≥ 0, b a

k(n − 1) . b

Half Convex Function Method

85

which is equivalent to h(b) ≥ 0 for b ≥ 1, where h(b) = (n − 1)(b k+1 − 1) − (k + 1)(b n − b). Since h0 (b) = (n − 1)b k − nb n−1 + 1 ≥ (n − 1)b n − nb n−1 + 1 k+1 = nb n−1 (b − 1) − (b n − 1)   = (b − 1) (b n−1 − b n−2 ) + (b n−1 − b n−3 ) + · · · + (b n−1 − 1) ≥ 0, h is increasing on [1, ∞), hence h(b) ≥ h(1) = 0. The proof is completed. The equality holds for a1 = a2 = · · · = an = 1.

P 1.58. If a1 , a2 , . . . , an are positive real numbers such that a1 a2 . . . an = 1, then ‹  ‹  ‹  1 a2 1 an 1 a1 + 1− + ··· + 1 − ≤ n − 1. 1− n n n (Vasile Cîrtoaje, 2006) Solution. Let k=

n , n−1

k > 1,

and m = ln k, 0 < m ≤ ln 2 < 1. Using the substitutions ai = e x i for i = 1, 2, . . . , n, we need to show that f (x 1 ) + f (x 2 ) + · · · + f (x n ) ≥ n f (s), where

s=

x1 + x2 + · · · + x n = 0, n

u

f (u) = −k−e , u ∈ R. From

u

f 00 (u) = meu k−e (1 − meu ), it follows that f 00 (u) > 0 for u ≤ 0, since 1 − meu ≥ 1 − m ≥ 1 − ln 2 > 0. Therefore, f is convex on (−∞, 0]. By HCF Theorem and Remark 5, it suffices to prove the original inequality for a2 = · · · = an := t and a1 = t −n+1 , where 0 < t ≤ 1. Write this inequality as h(t) ≤ n − 1,

86

Vasile Cîrtoaje

where h(t) = k−t

−n+1

We have h0 (t) = (n − 1)mt −n k−t h01 (t) = k−t

−n+1

−t

+ (n − 1)k−t , −n+1

t ∈ (0, 1].

h1 (t), h1 (t) = 1 − t n k−t

−n+1

−t

,

h2 (t), h2 (t) = [m(n − 1 + t n ) − nt n−1 ,

h02 (t) = nt n−2 (mt − n + 1) ≤ nt n−2 (m − n + 1) ≤ nt n−2 (m − 1) < 0, hence h2 is strictly decreasing on [0, 1]. Since h2 (0) = (n − 1)m > 0 and h2 (1) = n(m − 1) < 0, there is t 1 ∈ (0, 1) such that h2 (t 1 ) = 0, h2 (t) > 0 for t ∈ [0, t 1 ) and h2 (t) < 0 for t ∈ (t 1 , 1]. Therefore, h1 is strictly increasing on (0, t 1 ] and is strictly decreasing on [t 1 , 1]. Since lim t→0 h1 (u) = −∞ and h1 (1) = 0, there is t 2 ∈ (0, t 1 ) such that h1 (t 2 ) = 0, h1 (t) < 0 for t ∈ (0, t 2 ) and h1 (t) > 0 for t ∈ (t 2 , 1). Thus, h is strictly decreasing on (0, t 2 ] and is strictly increasing on [t 2 , 1]. Since lim t→0 h(u) = n − 1 and h(1) = n − 1, we have h(t) ≤ n − 1 for all t ∈ (0, 1]. This completes the proof. The equality holds for a1 = a2 = · · · = an = 1.

P 1.59. If a, b, c are positive real numbers such that a bc = 1, then 1 1 1 + + ≤ 1. p p p 1 + 1 + 3a 1 + 1 + 3b 1 + 1 + 3c (Vasile Cîrtoaje, 2008) Solution. Using the substitution a = e−x ,

b = e− y ,

c = e−z ,

we need to show that f (x) + f ( y) + f (z) ≥ 3 f (s), where f (u) =

s=

x + y +z = 0, 3

p −3 = eu − e2u + 3eu , u ∈ R. p 1 + 1 + 3e−u

For u ≥ 0, which involves t = eu ≥ 1, we have   4t 2 + 18t + 9 00 f (u) = t 1 − >0 p 4(t + 3) t(t + 3) since 16t(t + 3)3 − (4t 2 + 18t + 9)2 = 9(4t 2 + 12t − 9) > 0.

Half Convex Function Method

87

Therefore, f is half convex for u ≥ 0. By HCF Theorem, it suffices to prove that f (x) + 2 f ( y) ≥ 3 f (0), where x, y ∈ R such that x + 2 y = 0. Substituting a = e x and b = e y , we need to show that p p a − a2 + 3a + 2(b − b2 + 3b ) ≥ −3, where a, b > 0 such that a b2 = 1. Write this inequality as 2b3 + 3b2 + 1 ≥

p

3b2 + 1 + 2b2

p

b2 + 3b.

Squaring and dividing by b2 , the inequality becomes 9b2 + 4b + 3 ≥ 4

Æ

(b2 + 3b)(3b2 + 1).

Since 2

Æ

(b2 + 3b)(3b2 + 1) ≤ (b2 + 3b) + (3b2 + 1) = 4b2 + 3b + 1,

we get 9b2 + 4b + 3 − 4

Æ

(b2 + 3b)(3b2 + 1) ≥ 9b2 + 4b + 3 − 2(4b2 + 3b + 1) = (b − 1)2 ≥ 0.

The equality holds for a = b = c = 1. Remark. Similarly, we can prove the following generalization. • Let a1 , a2 , . . . , an be positive real numbers such that a1 a2 · · · an = 1. If 00 f (u) = t p 4(t + k) t(t + k) f (u) =

since (4t 2 + 6kt + k2 )2 − 16t(t + k)3 = k2 (k2 − 4kt − 4t 2 ) ≥ k2 (k2 − 4k − 4) > 0. Therefore, f is convex on (−∞, 0]. By HCF Theorem, it suffices to prove that f (x) + (n − 1) f ( y) ≥ n f (0), where x, y ∈ R such that x + (n − 1) y = 0. Substituting a = e x and b = e y , we need to show that p p p a2 + ka − a + (n − 1)( b2 + k b − b) ≥ n( 1 + k − 1), where a, b > 0 such that a b n−1 = 1. Write this inequality as Æ Æ 4n(n − 1)b n−1 + 1 + (n − 1) 4n(n − 1)b2n−1 + b2n ≥ (n − 1)b n + 2n(n − 1)b n−1 + 1. By Minkowski’s inequality, we have Æ Æ 4n(n − 1)b n−1 + 1 + (n − 1) 4n(n − 1)b2n−1 + b2n ≥ Æ ≥ 4n(n − 1)b n−1 [1 + (n − 1)b n/2 ]2 + [1 + (n − 1)b n ]2 . Thus, it suffices to show that 4n(n − 1)b n−1 [1 + (n − 1)b n/2 ]2 + [1 + (n − 1)b n ]2 ≥ [(n − 1)b n + 2n(n − 1)b n−1 + 1]2 , which is equivalent to 4n(n − 1)2 b

3n−2 2

”

n

2 + (n − 2)b 2 − nb

n−2 2

—

≥ 0.

This inequality follows immediately by the AM-GM inequality. Thus, the proof is completed. The equality holds for a1 = a2 = · · · = an = 1.

Half Convex Function Method

89

P 1.61. If a, b, c are positive real numbers such that a bc = 1, then b6 c6 a6 + + ≥ 1. 1 + 2a5 1 + 2b5 1 + 2c 5 (Vasile Cîrtoaje, 2008) Solution. Using the substitution a = ex ,

b = ey,

c = ez ,

we need to show that f (x) + f ( y) + f (z) ≥ 3 f (s),

s=

x + y +z = 0, 3

where

e6u , u ∈ R. 1 + 2e5u For u ≤ 0, which involves w = eu ∈ (0, 1], we have f (u) =

f 00 (u) =

2w6 (2 − w5 )(9 − 2w5 ) > 0. (1 + 2w5 )3

Therefore, f is convex for u ≤ 0. By HCF Theorem, it suffices to prove the original inequality for b = c := t and a = 1/t 2 , where t > 0; that is, 1 t 2 (t 10 + 2)

+

2t 6 ≥ 1. 1 + 2t 5

Since 1 + 2t 5 ≤ 1 + t 4 + t 6 , it suffices to show that 1 2x 3 + ≥ 1, x(x 5 + 2) 1 + x 2 + x 3

x=

p

t.

This inequality can be written as follows: x 3 (x 6 − x 5 − x 3 + 2x − 1) + (x − 1)2 ≥ 0, x 3 (x − 1)2 (x 4 + x 3 + x 2 − 1) + (x − 1)2 ≥ 0, (x − 1)2 [x 7 + x 5 + (x 6 − x 3 + 1)] ≥ 0. Clearly, the last inequality is true. The equality holds for a = b = c = 1.

90

Vasile Cîrtoaje

P 1.62. If a, b, c are positive real numbers such that a bc = 1, then p p p 25a2 + 144 + 25b2 + 144 + 25c 2 + 144 ≤ 5(a + b + c) + 24. (Vasile Cîrtoaje, 2008) Solution. Using the substitution a = ex ,

b = ey,

c = ez ,

we need to show that f (x) + f ( y) + f (z) ≥ 3 f (s), where f (u) = 5eu −

p

s=

x + y +z = 0, 3

25e2u + 144, u ∈ R.

We will show that f is convex for u ≤ 0. From   5w(25w2 + 288) 00 f (u) = 5w 1 − , (25w2 + 144)3/2

w = eu ∈ (0, 1],

we need to show that (25w2 + 144)3 ≥ 25w2 (25w2 + 288)2 . Setting 25w2 = 144z, z ∈ (0, 25/144], we have (25w2 +144)3 −25w2 (25w2 +288)2 = 1443 (z+1)3 −1443 z(z+2)2 = 1443 (1−z−z 2 ) > 0. By HCF Theorem, it suffices to prove the original inequality for a = t 2 and b = c = 1/t, where t > 0; that is, p p 5t 3 + 24t + 10 ≥ 25t 6 + 144t 2 + 2 25 + 144t 2 . Squaring and dividing by 4t give 60t 3 + 25t 2 − 36t + 120 ≥

Æ

(25t 4 + 144)(144t 2 + 25).

Squaring again and dividing by 120, the inequality becomes 25t 5 − 36t 4 + 105t 3 − 112t 2 − 72t + 90 ≥ 0, (t − 1)2 (25t 3 + 14t 2 + 108t + 90) ≥ 0. Since the last inequality is true, the proof is completed. The equality holds for a = b = c = 1.

Half Convex Function Method

91

P 1.63. If a, b, c are positive real numbers such that a bc = 1, then p p p 16a2 + 9 + 16b2 + 9 + 16c 2 + 9 ≥ 4(a + b + c) + 3. (Vasile Cîrtoaje, 2008) Solution. Using the substitution a = ex ,

b = ey,

c = ez ,

we need to show that f (x) + f ( y) + f (z) ≥ 3 f (s), where f (u) =

p

s=

x + y +z = 0, 3

16e2u + 9 − 4eu , u ∈ R.

We will show that f is convex for u ≥ 0. From   4w(16w2 + 18) 00 f (u) = 4w −1 , (16w2 + 9)3/2

w = eu ≥ 1,

we need to show that 16w2 (16w2 + 18)2 ≥ (16w2 + 9)3 . Setting 16w2 = 9z, z ≥ 16/9, we have Indeed, 16w2 (16w2 + 18)2 − (16w2 + 9)3 = 729z(z + 2)2 − 729(z + 1)3 = 729(z 2 + z − 1) > 0. By HCF Theorem, it suffices to prove the original inequality for a = t 2 and b = c = 1/t, where t > 0; that is, p p 16a6 + 9a2 + 2 16 + 9a2 ≥ 4t 3 + 3t + 8. Squaring and dividing by 4a give Æ (16a4 + 9)(9a2 + 16) ≥ 6a3 + 16a2 − 9a + 12. Squaring again and dividing by 12a, the inequality becomes 9a5 − 16a4 + 9a3 + 12a2 − 32a + 18 ≥ 0, (a − 1)2 (9a3 + 2a2 + 4a + 18) ≥ 0. Since the last inequality is true, the proof is completed. The equality holds for a = b = c = 1.

92

Vasile Cîrtoaje

P 1.64. If a, b, c, d are real numbers such that a + b + c + d = 4, then a2

b c d a + 2 + 2 + 2 ≤ 1. −a+4 b − b+4 c −c+4 d −d +4 (Sqing, 2015)

Solution. Write the inequality as f (a) + f (b) + f (c) + f (d) ≥ 4 f (s), where f (u) = From f 0 (u) =

−u , u2 − u + 4

u2 − 4 , (u2 − u + 4)2

f 00 (u) =

s=

a+b+c+d = 1, 4

u ∈ R. 2(−u3 + 12u − 4) , (u2 − u + 4)3

it follows that f is increasing on (−∞, −2] ∪ [2, ∞), decreasing on [−2, 2] and convex on [1, 2]. Define the function   f (u), u ≤ 2 f0 (u) = .  f (2), u > 2 Since f0 (u) ≤ f (u) for u ∈ R and f0 (1) = f (1), it suffices to show that f0 (a) + f0 (b) + f0 (c) + f0 (d) ≥ 4 f0 (s),

s=

a+b+c+d = 1. 4

It is easy to check that f0 is convex on [1, ∞). Therefore, by HCF Theorem and Remark 5, we only need to show that f0 (x)+3 f0 ( y) ≥ 4 f0 (1) for all x, y ∈ R such that x ≤ 1 ≤ y and x + 3 y = 4. There are two cases to consider: y ≤ 2 and y > 2. Case 1: y ≤ 2. We need to show that f (x) + 3 f ( y) ≥ 3 f (1) for all x, y ∈ R such that x + 3 y = 4. According to Remark 1, this is true if h(x, y) ≥ 0 for x + 3 y = 4. We have g(u) = h(x, y) =

f (u) − f (1) u−4 = , 2 u−1 4(u − u + 4)

g(x) − g( y) 4(x + y) − x y 3( y − 2)2 + 4 = = > 0. x−y 4(x 2 − x + 4)( y 2 − y + 4) 4(x 2 − x + 4)( y 2 − y + 4)

Case 2: y > 2. From y > 2 and x + 3 y = 4, we get x < −2 and f0 (x) + 3 f0 ( y) − 4 f0 (1) = f (x) + 3 f (2) − 4 f (1) = The equality holds for a = b = c = d = 1.

x2

−x > 0. − x +4

Half Convex Function Method

93

P 1.65. If a, b, c are nonnegative real numbers such that a + b + c = 12, then (a2 + 10)(b2 + 10)(c 2 + 10) ≥ 13310. (Vasile Cîrtoaje, 2006) Solution. Let f (u) = ln(u2 + 10), From f 00 (u) =

u ∈ [0, ∞).

2(10 − u2 ) , (u2 + 10)2

p p it follows that f is convex on [0, 10] and concave on [ 10, ∞). According to LCRCFTheorem, the sum f (a) + f (b) + f (c) is minimum when two of a, b, c are equal. Therefore, it suffices to prove the original inequality for b = c. So, we need to show that (a2 + 10)(b2 + 10)2 ≥ 13310 for a + 2b = 12. Write this inequality as follows: [(12 − 2b)2 + 10](b2 + 10)2 ≥ 13310, (2b2 − 24b + 77)(b4 + 20b2 + 100) ≥ 6655, 2b6 − 24b5 + 117b4 − 480b3 + 1740b2 − 2400b + 1045 ≥ 0, (b − 1)2 (2b4 − 20b3 + 75b2 − 310b + 1045) ≥ 0, (b − 1)2 [2b2 (b − 5)2 + 5(5b2 − 62b + 209)] ≥ 0. The last inequality is true since 31 5b − 62b + 209 = 5 b − 5 2



‹2

+

84 > 0. 5

The proof is completed. The equality holds for a = 10 and b = c = 1 (or any cyclic permutation). Remark. Similarly, we can prove the following generalization. • If a1 , a2 , . . . , an are nonnegative real numbers such that a1 + a2 + · · · + an = 2n(n − 1), then (a12 + k)(a22 + k) · · · (an2 + k) ≥ k(k + 1)n , k = (n − 1)(2n − 1). The equality holds for a1 = k and a2 = · · · = an = 1 (or any cyclic permutation).

94

Vasile Cîrtoaje

P 1.66. Let a, b, c be nonnegative real numbers. If k0 ≤ k ≤ 3, then

k0 =

ln 2 ≈ 1.71, ln 3 − ln 2

a+b+c a (b + c) + b (c + a) + c (a + b) ≤ 2 2 k

k



k

‹k+1

.

Solution. Due to homogeneity, we may assume that a + b + c = 2. Write the inequality as f (a) + f (b) + f (c) ≤ 2, where f (u) = uk (2 − u), u ∈ [0, ∞). From f 00 (u) = kuk−2 [2k − 2 − (k + 1)u], ˜ • ˜ • 2k − 2 2k − 2 and concave on , 2 . By it follows that therefore, f is convex on 0, k+1 k+1 LCRCF-Theorem, the sum f (a) + f (b) + f (c) is maximum when a = 0 or 0 < a ≤ b = c. Case 1: a = 0. We need to show that bc(b k−1 + c k−1 ) ≤ 2 for b + c = 2. Since 0 < (k − 1)/2 ≤ 1, Bernoulli’s inequality gives b k−1 + c k−1 = (b2 )(k−1)/2 + (c 2 )(k−1)/2 ≤ 1 + =3−k+

k−1 2 k−1 2 (b − 1) + 1 + (b − 1) 2 2

k−1 2 (b + c 2 ). 2

Thus, it suffices to show that (3 − k)bc +

k−1 bc(b2 + c 2 ) ≤ 2. 2

Since bc ≤



b+c 2

‹2

= 1,

we only need to show that bc(b2 + c 2 ) ≤ 2. Indeed, we have 8[2 − bc(b2 + c 2 )] = (b + c)4 − 8bc(b2 + c 2 ) = (b − c)4 ≥ 0.

Half Convex Function Method

95

Case 2: 0 < a ≤ b = c. We only need to prove the original homogeneous inequality for b = c = 1 and 0 < a ≤ 1; that is,  a k+1 − a k − a − 1 ≥ 0. 1+ 2  a k+1 Since 1 + is increasing and a k is decreasing when k increases, it suffices to prove 2 that g(a) ≥ 0 for 0 < a ≤ 1, where  a k0 +1 g(a) = 1 + − a k0 − a − 1. 2 We have

k0 + 1  a k0 − k0 a k0 −1 − 1, 1+ 2 2 k0 + 1  1 00 a k0 k0 − 1 − 2−k . g (a) = 1+ k0 4 2 a 0

g 0 (a) =

Since g 00 is increasing on (0, 1], lim x→0 g 00 (a) = −∞ and  ‹ k0 + 1 3 k0 k0 + 1 3 − k0 1 00 g (1) = − k0 + 1 = − k0 + 1 = > 0, k0 4 2 2 2 there exists a1 ∈ (0, 1) such that g 00 (a1 ) = 0, g 00 (a) < 0 for a ∈ (0, a1 ), and g 00 (a) > 0 for a ∈ (a1 , 1]. Therefore, g 0 is strictly decreasing on [0, a1 ] and strictly increasing on  k0 − 1 k0 + 1  [a1 , 1]. Since g 0 (0) = > 0 and g 0 (1) = (3/2)k0 − 2 = 0, there exists 2 2 a2 ∈ (0, a1 ) such that g 0 (a2 ) = 0, g 0 (a) > 0 for a ∈ [0, a2 ), and g 0 (a) < 0 for a ∈ (a2 , 1). Thus, g is strictly increasing on [0, a2 ], and strictly decreasing on [a2 , 1]. Consequently, g(a) ≥ min{g(0), g(1)}, and from g(0) = 0,

g(1) = (3/2)k0 +1 − 3 = 0,

we get g(a) ≥ 0. This completes the proof. The equality holds for a = 0 and b = c (or any cyclic permutation). If k = k0 , then the equality holds also for a = b = c.

P 1.67. If a1 , a2 , . . . , an are positive real numbers such that a1 + a2 + · · · + an = n, then  ‹ 1 1 1 (n + 1)2 + + ··· + ≥ 4(n + 2)(a12 + a22 + · · · + an2 ) + n(n2 − 3n − 6). a1 a2 an (Vasile Cîrtoaje, 2006)

96

Vasile Cîrtoaje

Solution. Write the inequality as f (a1 ) + f (a2 ) + · · · + f (an ) ≥ n(n2 − 3n − 6), where f (u) =

(n + 1)2 − 4(n + 2)u2 , u

u ∈ (0, ∞).

From

2(n + 1)2 − 8(n + 2), u3 it follows that f is strictly convex on (0,c] and strictly concave on [c, ∞), where v t 2 3 (n + 1) c= . 4(n + 2) f 00 (u) =

According to LCRCF-Theorem and Remark 6, it suffices to consider the case a1 = a2 = · · · = an−1 = x, 0 < x ≤ 1,

an = n − (n − 1)x,

when the inequality becomes as follows:  ‹ 1 2 n−1 (n + 1) + ≥ 4(n + 2)[(n − 1)x 2 + an2 ) + n(n2 − 3n − 6), x an n(n − 1)(2x − 1)2 [(n + 2)(n − 1)x 2 − (n + 2)(2n − 1)x + (n + 1)2 ] ≥ 0. The last inequality is true since 2n − 1 (n+2)(n−1)x −(n+2)(2n−1)x +(n+1) = (n+2)(n−1) x − 2n − 2 2

2

The equality holds for a1 = a2 = · · · = an−1 = tation).



‹2

+

3(n − 2) ≥ 0. 4(n − 1)

n+1 1 and an = (or any cyclic permu2 2

Chapter 2

Half Convex Function Method for Ordered Variables 2.1

Theoretical Basis

Half Convex Function Theorem for Ordered Variables (Vasile Cirtoaje, 2007). Let f (u) be a function defined on a real interval I and convex on Iu≥s / Iu≤s , where s ∈ I. The inequality a + a + ··· + a  1 2 n f (a1 ) + f (a2 ) + · · · + f (an ) ≥ n f n holds for all a1 , a2 , . . . , an ∈ I such that a1 + a2 + · · · + an = ns and at least n − m of a1 , a2 , . . . , an are smaller/greater than or equal to s if and only if f (x) + m f ( y) ≥ (1 + m) f (s) for all x, y ∈ I such that x + m y = (1 + m)s. Proof (for the right convexity). The necessity is obvious. Without loss of generality, assume that a1 ≤ a2 ≤ · · · ≤ an , a1 ≤ · · · ≤ an−m ≤ s. Since at least n − m of a1 , a2 , . . . , an are smaller than or equal to s , there is an integer k ∈ {n − m, . . . , n − 1} such that a1 ≤ · · · ≤ ak ≤ s ≤ ak+1 ≤ · · · ≤ an . Since f is convex on Iu≥s , we can apply Jensen’s inequality to get f (ak+1 ) + · · · + f (an ) ≥ (n − k) f (z), 97

z=

ak+1 + · · · + an . n−k

98

Vasile Cîrtoaje

Thus, it suffices to show that f (a1 ) + · · · + f (ak ) + (n − k) f (z) ≥ n f (s). Let b1 , . . . , bk ∈ I defined by ai + mbi = (1 + m)s,

i = 1, . . . , k.

We claim that z ≥ b1 ≥ · · · ≥ bk ≥ s. Indeed, we have b1 ≥ · · · ≥ bk , bk − s =

s − ak ≥ 0, m

mb1 = (1 + m)s − a1 = (m − n + 1)s + (a2 + · · · + ak ) + (ak+1 + · · · + an ) ≤ (m − n + 1)s + (k − 1)s + ak+1 + · · · + an = (m − n + k)s + (ak+1 + · · · + an ) = (m − n + k)s + (n − k)z = (m − n + k)(s − z) + mz ≤ mz. By hypothesis, we have f (a1 ) + m f (b1 ) ≥ (1 + m) f (s), ··· f (ak ) + m f (bk ) ≥ (1 + m) f (s), hence f (a1 ) + · · · + f (ak ) + m[ f (b1 ) + · · · + f (bk )] ≥ k(1 + m) f (s). Consequently, it suffices to show that k(1 + m) f (s) − m[ f (b1 ) + · · · + f (bk )] + (n − k) f (z) ≥ n f (s), which is equivalent to p f (z) + (k − p) f (s) ≥ f (b1 ) + · · · + f (bk ),

p=

n−k ≤ 1. m

By Jensen’s inequality, we have p f (z) + (1 − p) f (s) ≥ f (w),

w = pz + (1 − p)s ≥ s.

Thus, we only need to show that f (w) + (k − 1) f (s) ≥ f (b1 ) + · · · + f (bk ).

HCF Method for Ordered Variables

99

Since the decreasingly ordered vector A~k = (w, s, . . . , s) majorizes the decreasingly ordered vector B~k = (b1 , b2 , . . . , bk ), this inequality follows from Karamata’s inequality for convex functions. The proof for the left convexity is similar. Remark 1. Let us denote g(u) =

f (u) − f (s) , u−s

h(x, y) =

g(x) − g( y) . x−y

In many applications, it is useful to replace the hypothesis f (x) + m f ( y) ≥ (1 + m) f (s) in Half Convex Function Theorem for Ordered Variables (HCF-OV Theorem) by the equivalent condition h(x, y) ≥ 0 for all x, y ∈ I such that x + m y = (1 + m)s. This equivalence is true since f (x) + m f ( y) − (1 + m) f (s) = [ f (x) − f (s)] + m[ f ( y) − f (s)] = (x − s)g(x) + m( y − s)g( y) m = (x − y)[g(x) − g( y)] 1+m m (x − y)2 h(x, y). = 1+m Remark 2. Assume that f is differentiable on I, and let H(x, y) =

f 0 (x) − f 0 ( y) . x−y

Then, the desired inequality of Jensen’s type in HCF-OV Theorem holds true by replacing the hypothesis f (x) + m f ( y) ≥ (1 + m) f (s) with the more restrictive condition H(x, y) ≥ 0 for all x, y ∈ I such that x + m y = (1 + m)s. To prove this claim, we will show that the new condition implies f (x) + m f ( y) ≥ (1 + m) f (s) for all x, y ∈ I such that x + m y = (1 + m)s. Write this inequality as f1 (x) ≥ (1 + m) f (s), where  ‹ (1 + m)s − x f1 (x) = f (x) + m f ( y) = f (x) + m f . m

100

Vasile Cîrtoaje

From f10 (x) = f 0 (x) − f 0



(1 + m)s − x m

‹

= f 0 (x) − f 0 ( y) =

1+m (x − s)H(x, y), m

it follows that f1 is decreasing for x ≤ s and increasing for x ≥ s; therefore, f1 (x) ≥ f1 (s) = (1 + m) f (s). Remark 3. HCF-OV Theorem is also valid in the case when I = [a, b] \ {u0 } or I = (a, b) \ {u0 }, where a, b, u0 are real numbers such that a < u0 < b. Clearly, two cases are possible: (1) u0 < s - when f is right convex, i.e. convex on Iu≥s ; (2) u0 > s - when f is left convex, i.e. convex on Iu≤s . Remark 4. In HCF-OV Theorem for the right convexity (when HCF-OV Theorem is called RHCF-OV Theorem), it suffices to consider x ≤ s ≤ y. This claim is true because in the proof of HCF-OV Theorem, the hypothesis f (x) + m f ( y) ≥ (1 + m) f (s) is used to get the inequalities f (ai ) + m f (bi ) ≥ (1 + m) f (s),

i = 1, 2, · · · , k,

where ai ≤ s ≤ bi . Similarly, in HCF-OV Theorem for the left convexity (when HCF-OV Theorem is called LHCF-OV Theorem), it suffices to consider x ≥ s ≥ y. Remark 5. HCF-OV Theorem is a generalization of HCF-Theorem, because the last theorem can be obtained from the first theorem for m = n − 1. Remark 6. If a1 ≥ · · · ≥ am ≥ s ≥ am+1 ≥ · · · ≥ an , then at least n − m of a1 , a2 , . . . , an are smaller than or equal to s. Also, if a1 ≥ · · · ≥ an−m ≥ s ≥ an−m+1 ≥ · · · ≥ an , then at least n − m of a1 , a2 , . . . , an are greater than or equal to s. Thus, from HCF-OV Theorem and Remark 4, we get the following corollaries.

HCF Method for Ordered Variables

101

RHCF-OV Corollary. Let f (u) be a function defined on a real interval I and convex on Iu≥s , where s ∈ I. The inequality a + a + ··· + a  1 2 n f (a1 ) + f (a2 ) + · · · + f (an ) ≥ n f n holds for all a1 , a2 , . . . , an ∈ I satisfying a1 ≥ · · · ≥ am ≥ s ≥ am+1 ≥ · · · ≥ an ,

a1 + a2 + · · · + an = ns,

if and only if f (x) + m f ( y) ≥ (1 + m) f (s) for all x, y ∈ I such that x ≤ s ≤ y and x + m y = (1 + m)s. LHCF-OV Corollary. Let f (u) be a function defined on a real interval I and convex on Iu≤s , where s ∈ I. The inequality a + a + ··· + a  1 2 n f (a1 ) + f (a2 ) + · · · + f (an ) ≥ n f n holds for all a1 , a2 , . . . , an ∈ I satisfying a1 ≤ · · · ≤ am ≤ s ≤ am+1 ≤ · · · ≤ an ,

a1 + a2 + · · · + an = ns,

if and only if f (x) + m f ( y) ≥ (1 + m) f (s) for all x, y ∈ I such that x ≥ s ≥ y and x + m y = (1 + m)s. Remark 7. The inequality in RHCF-OV Corollary becomes an equality for a1 = a2 = · · · = an = s and also for a1 = · · · = am = y,

am+1 = · · · = an−1 = s,

an = x (x < y),

where x, y ∈ I satisfy the equations x + m y = (1 + m)s,

f (x) + m f ( y) = (1 + m) f (s).

For x 6= y, these equations are equivalent to x + m y = (1 + m)s, h(x, y) = 0. Remark 8. The inequality in LHCF-OV Corollary becomes an equality for a1 = a2 = · · · = an = s

102

Vasile Cîrtoaje

and also for a1 = x,

a2 = · · · = an−m = s,

an−m+1 = · · · = an = y (x > y),

where x, y ∈ I satisfy the equations x + m y = (1 + m)s,

f (x) + m f ( y) = (1 + m) f (s).

For x 6= y, these equations are equivalent to x + m y = (1 + m)s, h(x, y) = 0.

HCF Method for Ordered Variables

2.2

103

Applications

2.1. Let a1 , a2 , . . . , an be nonnegative real numbers such that a1 + a2 + · · · + an = n. (a) If at least n − m of a1 , a2 , . . . , an are smaller than or equal to 1, then m(a13 + a23 + · · · + an3 − n) ≥ (2m + 1)(a12 + a22 + · · · + an2 − n); (b) If at least n − m of a1 , a2 , . . . , an are greater than or equal to 1, then a13 + a23 + · · · + an3 − n ≤ (m + 2)(a12 + a22 + · · · + an2 − n).

2.2. If a, b, c, d are real numbers such that a + b + c + d = 4,

a ≥ b ≥ 1 ≥ c ≥ d, then

(3a2 − 2)(a − 1)2 + (3b2 − 2)(b − 1)2 + (3c 2 − 2)(c − 1)2 + (3d 2 − 2)(d − 1)2 ≥ 0.

2.3. If a1 , a2 , . . . , a2n ≥

−2n − 1 such that n−1

a1 ≥ · · · ≥ an ≥ 1 ≥ an+1 ≥ · · · ≥ a2n ,

a1 + a2 + · · · + a2n = 2n,

then 3 ≥ 2n. a13 + a23 + · · · + a2n

2.4. Let a1 , a2 , . . . , an be real numbers such that a1 + a2 + · · · + an = n. (a) If a1 ≥ 1 ≥ a2 ≥ · · · ≥ an , then a14 + a24 + · · · + an4 − n ≥ 6(a12 + a22 + · · · + an2 − n); (b) If a1 ≥ a2 ≥ 1 ≥ a3 ≥ · · · ≥ an , then a14 + a24 + · · · + an4 − n ≥

14 2 (a + a22 + · · · + an2 − n); 3 1

(c) If a1 ≥ · · · ≥ an−2 ≥ 1 ≥ an−1 ≥ an , then a14 + a24 + · · · + an4 − n ≥

2(n2 − 3n + 3) 2 (a1 + a22 + · · · + an2 − n). n2 − 5n + 7

104

Vasile Cîrtoaje

2.5. Let a, b, c, d, e be nonnegative real numbers such that a + b + c + d + e = 5. (a) If a ≥ b ≥ 1 ≥ c ≥ d ≥ e, then 21(a2 + b2 + c 2 + d 2 + e2 ) ≥ a4 + b4 + c 4 + d 4 + e4 + 100; (b) If a ≥ b ≥ c ≥ 1 ≥ d ≥ e, then 13(a2 + b2 + c 2 + d 2 + e2 ) ≥ a4 + b4 + c 4 + d 4 + e4 + 60. 2.6. Let a1 , a2 , . . . , an be nonnegative real numbers such that a1 + a2 + · · · + an = n. (a) If a1 ≥ · · · ≥ an−1 ≥ 1 ≥ an , then 7(a13 + a23 + · · · + an3 ) ≥ 3(a14 + a24 + · · · + an4 ) + 4n; (b) If a1 ≥ · · · ≥ an−2 ≥ 1 ≥ an−1 ≥ an , then 13(a13 + a23 + · · · + an3 ) ≥ 4(a14 + a24 + · · · + an4 ) + 9n. 2.7. If a1 , a2 , . . . , an are positive real numbers such that a1 + a2 + · · · + an = n,

a1 ≥ · · · ≥ an−m ≥ 1 ≥ an−m+1 ≥ · · · ≥ an , then (m + 1)2



‹ 1 1 1 + + ··· + − n ≥ 4m(a12 + a22 + · · · + an2 − n). a1 a2 an

2.8. If a1 , a2 , . . . , an are positive real numbers such that 1 1 1 + + ··· + = n, a1 a2 an

a1 ≤ · · · ≤ an−m ≤ 1 ≤ an−m+1 ≤ · · · ≤ an , then a12

+ a22

+ · · · + an2

p ‹  m (a1 + a2 + · · · + an − n). −n≥2 1+ 1+m

2.9. Let a1 , a2 , . . . , an be nonnegative real numbers such that a1 + a2 + · · · + an = n. (a) If a1 ≥ 1 ≥ a2 ≥ · · · ≥ an , then 1 a12

+2

+

1 a22

+2

+ ··· +

an2

1 n ≥ ; +2 3

(b) If a1 ≥ a2 ≥ 1 ≥ a3 ≥ · · · ≥ an , then 1 2a12

+3

+

1 2a22

+3

+ ··· +

1 n ≥ . +3 5

2an2

HCF Method for Ordered Variables

105

2.10. If a1 , a2 , . . . , a2n are nonnegative real numbers such that a1 ≥ · · · ≥ an ≥ 1 ≥ an+1 ≥ · · · ≥ a2n ,

a1 + a2 + · · · + a2n = 2n,

then 1 na12

+ n2

+n+1

+

1 na22

+ n2

+n+1

+ ··· +

1 2 na2n

+ n2

+n+1



2n . (n + 1)2

2.11. If a, b, c, d, e, f are nonnegative real numbers such that a ≥ b ≥ c ≥ 1 ≥ d ≥ e ≥ f, then

a + b + c + d + e + f = 6,

3f + 4 3a + 4 3b + 4 3c + 4 3d + 4 3e + 4 + 2 + 2 + 2 + 2 + ≤ 6. 2 3a + 4 3b + 4 3c + 4 3d + 4 3e + 4 3 f 2 + 4

2.12. If a, b, c, d, e, f are nonnegative real numbers such that a ≥ b ≥ 1 ≥ c ≥ d ≥ e ≥ f,

a + b + c + d + e + f = 6,

then f2−1 b2 − 1 c2 − 1 d2 − 1 e2 − 1 a2 − 1 + + + + + ≥ 0. (2a + 7)2 (2b + 7)2 (2c + 7)2 (2d + 7)2 (2e + 7)2 (2 f + 7)2

2.13. If a, b, c, d, e, f are nonnegative real numbers such that a ≥ b ≥ c ≥ d ≥ 1 ≥ e ≥ f,

a + b + c + d + e + f = 6,

then f2−1 b2 − 1 c2 − 1 d2 − 1 e2 − 1 a2 − 1 + + + + + ≤ 0. (2a + 5)2 (2b + 5)2 (2c + 5)2 (2d + 5)2 (2e + 5)2 (2 f + 5)2

2.14. If a, b, c, d are nonnegative real numbers such that a ≥ b ≥ 1 ≥ c ≥ d, then

a + b + c + d = 4,

1 1 1 1 4 + 3 + 3 + 3 ≤ . + 5 2b + 5 2c + 5 2d + 5 7

2a3

106

Vasile Cîrtoaje

2.15. If a1 , a2 , . . . , a8 are nonnegative real numbers such that a1 ≥ a2 ≥ a3 ≥ a4 ≥ 1 ≥ a5 ≥ a6 ≥ a7 ≥ a8 ,

a1 + a2 + · · · + a8 = 8,

then (a12 + 1)(a22 + 1) · · · (a82 + 1) ≥ (a1 + 1)(a2 + 1) · · · (a8 + 1).

2.16. If a, b, c, d are positive real numbers such that a ≥ b ≥ 1 ≥ c ≥ d, then

a bcd = 1,

‹ 1 1 1 1 a + b + c + d − 4 ≥ 18 a + b + c + d − − − − . a b c d 2

2

2



2

2.17. If a, b, c, d are positive real numbers such that a ≥ b ≥ 1 ≥ c ≥ d,

a bcd = 1,

then p

a2 − a + 1 +

p

b2 − b + 1 +

p

c2 − c + 1 +

p

d 2 − d + 1 ≥ a + b + c + d.

2.18. If a, b, c, d are positive real numbers such that a ≥ 1 ≥ b ≥ c ≥ d, then a3

a bcd = 1,

1 1 1 1 2 + 3 + 3 + 3 ≥ . + 3a + 2 b + 3b + 2 c + 3c + 2 d + 3d + 2 3

2.19. Let a1 , a2 , . . . , an be positive real numbers such that a1 ≥ 1 ≥ a2 ≥ · · · ≥ an , If k ≥ 1, then

a1 a2 · · · an = 1.

1 1 1 n + + ··· + ≥ . 1 + ka1 1 + ka2 1 + kan 1+k

HCF Method for Ordered Variables

107

2.20. If a1 , a2 , . . . , a9 are positive real numbers such that a1 ≥ 1 ≥ a2 ≥ · · · ≥ a9 , then

a1 a2 · · · a9 = 1,

1 1 1 + + ··· + ≥ 1. (a1 + 2)2 (a2 + 2)2 (a9 + 2)2

2.21. If a1 , a2 , . . . , an (n ≥ 4) are positive real numbers such that a1 ≥ a2 ≥ a3 ≥ 1 ≥ a4 ≥ · · · ≥ an , then

a1 a2 · · · an = 1,

1 1 1 n + + ··· + ≥ . (a1 + 1)2 (a2 + 1)2 (an + 1)2 4

2.22. If a1 , a2 , . . . , an are positive real numbers such that a1 ≤ 1 ≤ a2 ≤ · · · ≤ an , then

a1 a2 · · · an = 1,

1 1 1 n + + ··· + ≤ . 2 2 2 (a1 + 3) (a2 + 3) (an + 3) 16

2.23. Let a1 , a2 , . . . , an be positive real numbers such that a1 ≤ 1 ≤ a2 ≤ · · · ≤ an ,

a1 a2 · · · an = 1.

If p, q ≥ 0 such that p + q ≤ 1, then 1 1+

pa1 + qa12

+

1 1+

pa2 + qa22

+ ··· +

1 n ≤ . 2 1 + pan + qan 1+p+q

2.24. If a1 , a2 , . . . , an are positive real numbers such that a1 ≤ a2 ≤ 1 ≤ a3 ≤ · · · ≤ an , then

a1 a2 · · · an = 1,

1 1 1 n + + ··· + ≤ . 2 2 2 (a1 + 5) (a2 + 5) (an + 5) 36

108

Vasile Cîrtoaje

2.25. If a1 , a2 , . . . , an are positive real numbers such that a1 ≥ 1 ≥ a2 ≥ · · · ≥ an , then

1 p

1 + 3a1

+p

1 1 + 3a2

a1 a2 · · · an = 1,

+ ··· + p

1 1 + 3an



n . 2

2.26. If a1 , a2 , . . . , an are positive real numbers such that a1 ≤ 1 ≤ a2 ≤ · · · ≤ an , then

1 p

1 + 2a1

+p

1 1 + 2a2

a1 a2 · · · an = 1,

+ ··· + p

1

n ≤p . 3 1 + 2an

HCF Method for Ordered Variables

2.3

109

Solutions

P 2.1. Let a1 , a2 , . . . , an be nonnegative real numbers such that a1 + a2 + · · · + an = n. (a) If at least n − m of a1 , a2 , . . . , an are smaller than or equal to 1, then m(a13 + a23 + · · · + an3 − n) ≥ (2m + 1)(a12 + a22 + · · · + an2 − n); (b) If at least n − m of a1 , a2 , . . . , an are greater than or equal to 1, then a13 + a23 + · · · + an3 − n ≤ (m + 2)(a12 + a22 + · · · + an2 − n). (Vasile Cirtoaje, 2007) Solution. (a) Write the inequality as f (a1 ) + f (a2 ) + · · · + f (an ) ≥ n f (s),

s=

a1 + a2 + · · · + an = 1, n

where f (u) = mu3 − (2m + 1)u2 ,

u ∈ [0, n].

From f 00 (u) = 2(3mu − 2m − 1), it follows that f is convex on [1, n]. By HCF-OV Theorem (or RHCF-OV Corollary) and Remark 1, it suffices to show that h(x, y) ≥ 0 for all x, y ≥ 0 such that x + m y = m + 1. We have g(u) =

f (u) − f (1) = m(u2 + u + 1) − (2m + 1)(u + 1) = mu2 − (m + 1)u − m − 1, u−1 h(x, y) =

g(x) − g( y) = m(x + y) − m − 1 = (m − 1)x ≥ 0. x−y

From x + m y = m + 1 and h(x, y) = 0, we get x = 0, y = (m + 1)/m. Therefore, in accordance with Remark 7, the equality holds for a1 = a2 = · · · = an = 1, and also for a1 = · · · = am =

m+1 , m

am+1 = · · · = an−1 = 1,

an = 0

(or any thereof permutation). (b) Write the inequality as f (a1 ) + f (a2 ) + · · · + f (an ) ≥ n f (s),

s=

a1 + a2 + · · · + an = 1, n

where f (u) = (m + 2)u2 − u3 ,

u ∈ [0, n].

110

Vasile Cîrtoaje

From f 00 (u) = 2(m + 2 − 3u), it follows that f is convex on [0, 1]. By HCF-OV Theorem (or LRHCF-OV Corollary) and Remark 1, it suffices to show that h(x, y) ≥ 0 for all x, y ≥ 0 such that x + m y = m + 1. We have g(u) =

f (u) − f (1) = (m + 2)(u + 1) − (u2 + u + 1) = −u2 + (m + 1)u + m + 1, u−1 h(x, y) =

g(x) − g( y) = −(x + y) + m + 1 = (m − 1) y ≥ 0. x−y

From x +m y = m+1 and h(x, y) = 0, we get x = m+1, y = 0. Therefore, in accordance with Remark 8, the equality holds for a1 = a2 = · · · = an = 1, and also for a1 = m + 1,

a2 = · · · = an−m = 1,

an−m+1 = · · · = an = 0

(or any thereof permutation). Remark 1. For m = n − 1, we get the following results: • If a1 , a2 , . . . , an are nonnegative real numbers such that a1 + a2 + · · · + an = n, then (n − 1)(a13 + a23 + · · · + an3 − n) ≥ (2n − 1)(a12 + a22 + · · · + an2 − n), with equality for a1 = a2 = · · · = an = 1, and also for a1 = 0 and a2 = a3 = · · · = an = n (or any cyclic permutation). n−1 • If a1 , a2 , . . . , an are nonnegative real numbers such that a1 + a2 + · · · + an = n, then a13 + a23 + · · · + an3 − n ≤ (n + 1)(a12 + a22 + · · · + an2 − n), with equality for a1 = a2 = · · · = an = 1, and also for a1 = n and a2 = a3 = · · · = an = 0 (or any cyclic permutation). Remark 2. For m = 1, we get the following results: • If a1 , a2 , . . . , an are nonnegative real numbers such that a1 ≥ 1 ≥ a2 ≥ · · · ≥ an ,

a1 + a2 + · · · + an = n,

then a13 + a23 + · · · + an3 + 2n ≥ 3(a12 + a22 + · · · + an2 ), with equality for a1 = a2 = · · · = an = 1, and also for a1 = 2, a2 = · · · = an−1 = 1, an = 0. • If a1 , a2 , . . . , an are nonnegative real numbers such that a1 ≥ · · · ≥ an−1 ≥ 1 ≥ an ,

a1 + a2 + · · · + an = n,

HCF Method for Ordered Variables

111

then a13 + a23 + · · · + an3 + 2n ≤ 3(a12 + a22 + · · · + an2 ), with equality for a1 = a2 = · · · = an = 1, and also for a1 = 2, a2 = · · · = an−1 = 1, an = 0. Remark 3. Replacing n with 2n and choosing m = n, we get the following results: • If a1 , a2 , . . . , a2n are nonnegative real numbers such that a1 ≥ · · · ≥ an ≥ 1 ≥ an+1 ≥ · · · ≥ a2n ,

a1 + a2 + · · · + a2n = 2n,

then 2 3 − 2n), − 2n) ≥ (2n + 1)(a12 + a22 + · · · + a2n n(a13 + a23 + · · · + a2n

with equality for a1 = a2 = · · · = a2n = 1, and also for a1 = · · · = an =

n+1 , n

an+2 = · · · = a2n−1 = 1,

a2n = 0.

• If a1 , a2 , . . . , a2n are nonnegative real numbers such that a1 ≥ · · · ≥ an ≥ 1 ≥ an+1 ≥ · · · ≥ a2n ,

a1 + a2 + · · · + a2n = 2n,

then 2 3 − 2n), − 2n ≤ (n + 2)(a12 + a22 + · · · + a2n a13 + a23 + · · · + a2n

with equality for a1 = a2 = · · · = a2n = 1, and also for a1 = n + 1,

a2 = · · · = an = 1,

an+1 = · · · = a2n = 0.

P 2.2. If a, b, c, d are real numbers such that a ≥ b ≥ 1 ≥ c ≥ d,

a + b + c + d = 4,

then (3a2 − 2)(a − 1)2 + (3b2 − 2)(b − 1)2 + (3c 2 − 2)(c − 1)2 + (3d 2 − 2)(d − 1)2 ≥ 0. (Vasile Cirtoaje, 2007)

112

Vasile Cîrtoaje

Solution. Write the inequality as f (a) + f (b) + f (c) + f (d) ≥ 4 f (s),

s=

a+b+c+d = 1, 4

where f (u) = (3u2 − 2)(u − 1)2 ,

u ∈ R.

From f 00 (u) = 2(18u2 − 18u + 1), it follows that f 00 (u) > 0 for u ≥ 1, hence f is convex for u ≥ 1. Therefore, we may apply HCF-OV Theorem or RHCF-OV Corollary for n = 4 and m = 2. By these, it suffices to show that f (x) + 2 f ( y) ≥ 3 f (1) for all real x, y such that x + 2 y = 3. Using Remark 1, we only need to show that h(x, y) ≥ 0, where h(x, y) =

g(x) − g( y) , x−y

g(u) =

f (u) − f (1) . u−1

We have g(u) = 3(u3 + u2 + u + 1) − 6(u2 + u + 1) + u + 1 = 3u3 − 3u2 − 2u − 2, h(x, y) = 3(x 2 + x y + y 2 ) − 3(x + y) − 2 = (3 y − 4)2 ≥ 0. From x + 2 y = 3 and h(x, y) = 0, we get x = 1/3, y = 4/3. Therefore, in accordance with Remark 7, the equality holds for a = b = c = d = 1, and also for a = b = 4/3, c = 1 and d = 1/3. Remark. Similarly, we can prove the following generalization: • Let a1 , a2 , . . . , a2n be real numbers such that a1 ≥ · · · ≥ an ≥ 1 ≥ an+1 ≥ · · · ≥ a2n , If k =

n2

a1 + a2 + · · · + a2n = 2n.

n , then −n+1 2 (a12 − k)(a1 − 1)2 + (a22 − k)(a2 − 1)2 + · · · + (a2n − k)(a2n − 1)2 ≥ 0,

with equality for a1 = a2 = · · · = a2n = 1, and also for a1 = · · · = an =

n2 , n2 − n + 1

an+1 = · · · = a2n−1 = 1,

a2n =

n2

1 . −n+1

HCF Method for Ordered Variables

P 2.3. If a1 , a2 , . . . , a2n ≥

113

−2n − 1 such that n−1

a1 ≥ · · · ≥ an ≥ 1 ≥ an+1 ≥ · · · ≥ a2n ,

a1 + a2 + · · · + a2n = 2n,

then 3 a13 + a23 + · · · + a2n ≥ 2n.

(Vasile Cirtoaje, 2007) Solution. Write the inequality as f (a1 ) + f (a2 ) + · · · + f (a2n ) ≥ 2n f (s), where f (u) = u3 ,

u≥

s=

a1 + a2 + · · · + a2n = 1, 2n

−2n − 1 . n−1

From f 00 (u) = 6u, it follows that f is convex for u ≥ 1. Therefore, we may apply HCF-OV Theorem or RHCF-OV Corollary for 2n numbers and m = n. By Remark 1, it suffices to −2n − 1 show that h(x, y) ≥ 0 for all x, y ≥ such that x + n y = n + 1. We have n−1 g(u) = h(x, y) =

f (u) − f (1) = u2 + u + 1, u−1

g(x) − g( y) (n − 1)x + 2n + 1 = x + y +1= ≥ 0. x−y n−1

n+2 −2n − 1 and y = . Therefore, n−1 n−1 in accordance with Remark 7, the equality holds for a1 = a2 = · · · = a2n = 1, and also for n+2 −2n − 1 a1 = · · · = an = , an+1 = · · · = a2n−1 = 1, a2n = . n−1 n−1 From x + n y = n + 1 and h(x, y) = 0, we get x =

P 2.4. Let a1 , a2 , . . . , an be real numbers such that a1 + a2 + · · · + an = n. (a) If a1 ≥ 1 ≥ a2 ≥ · · · ≥ an , then a14 + a24 + · · · + an4 − n ≥ 6(a12 + a22 + · · · + an2 − n); (b) If a1 ≥ a2 ≥ 1 ≥ a3 ≥ · · · ≥ an , then a14 + a24 + · · · + an4 − n ≥

14 2 (a + a22 + · · · + an2 − n); 3 1

114

Vasile Cîrtoaje (c) If a1 ≥ · · · ≥ an−2 ≥ 1 ≥ an−1 ≥ an , then a14 + a24 + · · · + an4 − n ≥

2(n2 − 3n + 3) 2 (a1 + a22 + · · · + an2 − n). n2 − 5n + 7 (Vasile Cirtoaje, 2009)

Solution. Consider the inequality a14 + a24 + · · · + an4 − n ≥ k(a12 + a22 + · · · + an2 − n),

k ≤ 6,

and write it as f (a1 ) + f (a2 ) + · · · + f (an ) ≥ n f (s),

s=

a1 + a2 + · · · + an = 1, n

where f (u) = u4 − ku2 ,

u ≥ 0.

From f 00 (u) = 2(6u2 − k), it follows that f is convex for u ≥ 1. Therefore, we may apply HCF-OV Theorem or RHCF-OV Corollary for m = 1, m = 2 and m = n − 2, respectively. By Remark 1, it suffices to show that h(x, y) ≥ 0 for all x, y ≥ 0 such that x +m y = m+1. We have f (u) − f (1) g(u) = = u3 + u2 + u + 1 − k(u + 1), u−1 h(x, y) =

g(x) − g( y) = x 2 + x y + y 2 + x + y − k + 1. x−y

(a) We have k = 6, m = 1 and x + y = 2, which involve h(x, y) = 1 − x y =

1 (x − y)2 ≥ 0. 4

From x + y = 2 and h(x, y) = 0, we get x = y = 1. Therefore, in accordance with Remark 7, the equality holds for a1 = a2 = · · · = an = 1. (b) We have k = 14/3, m = 2 and x + 2 y = 3, which involve h(x, y) =

1 (3 y − 5)2 ≥ 0. 3

From x + 2 y = 3 and h(x, y) = 0, we get x = −1/3 and y = 5/3. Therefore, in accordance with Remark 7, the equality holds for a1 = a2 = · · · = an = 1, and also for a1 = a2 =

5 , 3

a3 = · · · = an−1 = 1,

an =

−1 . 3

HCF Method for Ordered Variables

(c) We have k =

115

2(n2 − 3n + 3) , m = n − 2 and x + (n − 2) y = n − 1, which involve n2 − 5n + 7

h(x, y) =

n2

1 [(n2 − 5n + 7) y − n2 + 3n − 1]2 ≥ 0. − 5n + 7

−n2 + 5n − 4 n2 − 3n + 1 and . n2 − 5n + 7 n2 − 5n + 7 Therefore, in accordance with Remark 7, the equality holds for a1 = a2 = · · · = an = 1, and also for From x + (n − 2) y = n − 1 and h(x, y) = 0, we get x =

a1 = · · · = an−2 =

n2 − 3n + 1 , n2 − 5n + 7

an−1 = 1,

an =

−n2 + 5n − 4 . n2 − 5n + 7

P 2.5. Let a, b, c, d, e be nonnegative real numbers such that a + b + c + d + e = 5. (a) If a ≥ b ≥ 1 ≥ c ≥ d ≥ e, then 21(a2 + b2 + c 2 + d 2 + e2 ) ≥ a4 + b4 + c 4 + d 4 + e4 + 100; (b) If a ≥ b ≥ c ≥ 1 ≥ d ≥ e, then 13(a2 + b2 + c 2 + d 2 + e2 ) ≥ a4 + b4 + c 4 + d 4 + e4 + 60. (Vasile Cirtoaje, 2009) Solution. Consider the inequality k(a2 + b2 + c 2 + d 2 + e2 − 5) ≥ a4 + b4 + c 4 + d 4 + e4 − 5,

k ≥ 6,

and write it as f (a) + f (b) + f (c) + f (d) + f (e) ≥ 5 f (s),

s=

a+b+c+d+e = 1, 5

where f (u) = ku2 − u4 ,

u ≥ 0.

From f 00 (u) = 2(k − 6u2 ), it follows that f is convex on [0, 1]. Therefore, we may apply HCF-OV Theorem or LHCF-OV Corollary for m = 3 and m = 2, respectively. By Remark 1, it suffices to show that h(x, y) ≥ 0 for all x, y ≥ 0 such that x + m y = m + 1. We have f (u) − f (1) g(u) = = k(u + 1) − (u3 + u2 + u + 1), u−1

116

Vasile Cîrtoaje h(x, y) =

g(x) − g( y) = k − (x 2 + x y + y 2 + x + y + 1). x−y

(a) We have k = 21, m = 3 and x + 3 y = 4, which involve y ≤ 4/3 and h(x, y) = 20 − (x 2 + x y + y 2 + x + y) = y(22 − 7 y) ≥ 0. From x + 3 y = 4 and h(x, y) = 0, we get x = 4 and y = 0. Therefore, in accordance with Remark 8, the equality holds for a = b = c = d = e = 1 and also for a = 4, b = 1, c = d = e = 0. (b) We have k = 13, m = 2 and x + 2 y = 3, which involve y ≤ 3/2 and h(x, y) = 12 − (x 2 + x y + y 2 + x + y) = y(10 − 3 y) ≥ 0. From x + 2 y = 3 and h(x, y) = 0, we get x = 3 and y = 0. Therefore, in accordance with Remark 8, the equality holds for a = b = c = d = e = 1 and also for a = 3, b = c = 1, d = e = 0.

P 2.6. Let a1 , a2 , . . . , an be nonnegative real numbers such that a1 + a2 + · · · + an = n. (a) If a1 ≥ · · · ≥ an−1 ≥ 1 ≥ an , then 7(a13 + a23 + · · · + an3 ) ≥ 3(a14 + a24 + · · · + an4 ) + 4n; (b) If a1 ≥ · · · ≥ an−2 ≥ 1 ≥ an−1 ≥ an , then 13(a13 + a23 + · · · + an3 ) ≥ 4(a14 + a24 + · · · + an4 ) + 9n. (Vasile Cirtoaje, 2009) Solution. Consider the inequality k(a13 + a23 + · · · + an3 − n) ≥ a14 + a24 + · · · + an4 − n,

k > 2,

and write it as f (a1 ) + f (a2 ) + · · · + f (an ) ≥ n f (s),

s=

a1 + a2 + · · · + an = 1, n

where f (u) = ku3 − u4 ,

u ≥ 0.

From f 00 (u) = 6u(k − 2u2 ), it follows that f is convex on [0, 1]. Therefore, we may apply HCF-OV Theorem or LHCF-OV Corollary for m = 1 and m = 2, respectively. By

HCF Method for Ordered Variables

117

Remark 1, it suffices to show that h(x, y) ≥ 0 for all x, y ≥ 0 such that x + m y = m + 1. We have f (u) − f (1) g(u) = = k(u2 + u + 1) − (u3 + u2 + u + 1), u−1 g(x) − g( y) h(x, y) = = −(x 2 + x y + y 2 ) + (k − 1)(x + y + 1). x−y (a) We have k = 7/3, m = 1 and x + y = 2, which involve h(x, y) = x y ≥ 0. From x + y = 2 and h(x, y) = 0, we get x = 2 and y = 0. Therefore, in accordance with Remark 8, the equality holds for a1 = a2 = · · · = an = 1, and also for a1 = 2,

a2 = · · · = an−1 = 1,

an = 0.

(b) We have k = 13/4, m = 2 and x + 2 y = 3, which involve h(x, y) = 3 y(9 − 4 y) = 3 y(3 + 2x) ≥ 0. From x + 2 y = 3 and h(x, y) = 0, we get x = 3 and y = 0. Therefore, in accordance with Remark 8, the equality holds for a1 = a2 = · · · = an = 1, and also for a1 = 3,

a2 = · · · = an−2 = 1,

an−1 = an = 0.

P 2.7. If a1 , a2 , . . . , an are positive real numbers such that a1 ≥ · · · ≥ an−m ≥ 1 ≥ an−m+1 ≥ · · · ≥ an , then (m + 1)2



a1 + a2 + · · · + an = n,

‹ 1 1 1 + + ··· + − n ≥ 4m(a12 + a22 + · · · + an2 − n). a1 a2 an (Vasile Cirtoaje, 2007)

Solution. Write the inequality as f (a1 ) + f (a2 ) + · · · + f (an ) ≥ n f (s), where f (u) =

s=

a1 + a2 + · · · + an = 1, n

(m + 1)2 − 4mu2 , u ∈ (0, n). u

118

Vasile Cîrtoaje

For u ∈ (0, 1], we have f 00 (u) =

2(m + 1)2 − 8m ≥ 2(m + 1)2 − 8m = 2(m − 1)2 ≥ 0. u3

Since f is convex on (0, 1], we may apply HCF-OV Theorem or LHCF-OV Corollary. By Remark 1, it suffices to show that h(x, y) ≥ 0 for all x, y > 0 such that x + m y = m + 1. We have f (u) − f (1) −(m + 1)2 g(u) = = − 4m(u + 1), u−1 u h(x, y) =

[m + 1 − 2m y]2 (m + 1)2 − 4m = ≥ 0. xy xy

1+m 1+m and y = . Therefore, in 2 2m accordance with Remark 8, the equality holds for a1 = a2 = · · · = an = 1, and also for From x + m y = m + 1 and h(x, y) = 0, we get x =

a1 =

1+m , 2

a2 = a3 = · · · = an−m = 1,

an−m+1 = · · · = an =

1+m . 2m

Remark 1. For m = 1, we get the following elegant statement: • If a1 , a2 , . . . , an are positive real numbers such that a1 ≥ · · · ≥ an−1 ≥ 1 ≥ an , then

a1 + a2 + · · · + an = n,

1 1 1 + + ··· + ≥ a12 + a22 + · · · + an2 , a1 a2 an

with equality for a1 = a2 = · · · = an = 1 Remark 2. Replacing n with 2n and choosing m = n, we get the statement: • If a1 , a2 , . . . , a2n are positive real numbers such that a1 ≥ · · · ≥ an ≥ 1 ≥ an+1 ≥ · · · ≥ a2n , then (n + 1)2



a1 + a2 + · · · + a2n = 2n,

‹ 1 1 1 2 + + ··· + − 2n ≥ 4n(a12 + a22 + · · · + a2n − 2n), a1 a2 a2n

with equality for a1 = a2 = · · · = a2n = 1, and also for a1 =

1+n , 2

a2 = a3 = · · · = an = 1,

an+1 = · · · = a2n =

1+n . 2n

HCF Method for Ordered Variables

119

P 2.8. If a1 , a2 , . . . , an are positive real numbers such that 1 1 1 + + ··· + = n, a1 a2 an

a1 ≤ · · · ≤ an−m ≤ 1 ≤ an−m+1 ≤ · · · ≤ an , then a12

+ a22

p  ‹ m −n≥2 1+ (a1 + a2 + · · · + an − n). 1+m

+ · · · + an2

(Vasile Cirtoaje, 2007) Solution. Replacing each ai by 1/ai , we need to prove that a1 ≥ · · · ≥ an−m ≥ 1 ≥ an−m+1 ≥ · · · ≥ an ,

a1 + a2 + · · · + an = n

involves f (a1 ) + f (a2 ) + · · · + f (an ) ≥ n f (s), where

s=

a1 + a2 + · · · + an = 1, n

p  ‹ 1 1 m , u ∈ (0, n). f (u) = 2 − 2 1 + u 1+m u

For u ∈ (0, 1], we have 00

f (u) =

€ 6−4 1+

p Š m 1+m u

u4



€ 6−4 1+

p Š m m+1

u4

p 2( m − 1)2 = ≥ 0. (1 + m)u4

Thus, f is convex on (0, 1]. By HCF-OV Theorem (or LHCF-OV Corollary) and Remark 1, it suffices to show that h(x, y) ≥ 0 for x, y > 0 such that x + m y = 1 + m, where h(x, y) = We have

g(x) − g( y) , x−y

g(u) =

f (u) − f (1) . u−1

p ‹  2 m 1 −1 g(u) = 2 + 1 + u 1+m u

and h(x, y) = We only need to show that

1 xy



p ‹ 1 1 2 m + −1− . x y 1+m

p 1 1 2 m + ≥1+ . x y 1+m

Indeed, using the Cauchy-Schwarz inequality, we get p p p (1 + m)2 (1 + m)2 2 m 1 1 + ≥ = =1+ . x y x + my 1+m 1+m

120

Vasile Cîrtoaje

1+m 1+m p and y = p . In 1+ m m+ m accordance with Remark 8, we have f (a1 ) + f (a2 ) + · · · + f (an ) ≥ n f (1) for a1 = a2 = · · · = an = 1, and also for From x + m y = 1 + m and h(x, y) = 0, we get x =

a1 =

1+m p , 1+ m

a2 = a3 = · · · = an−m = 1,

an−m+1 = · · · = an =

1+m p . m+ m

Therefore, the original inequality becomes an equality for a1 = a2 = · · · = an = 1, and also for p p m+ m 1+ m , a2 = a3 = · · · = an−m = 1, an−m+1 = · · · = an = . a1 = 1+m 1+m Remark. Replacing n with 2n and choosing m = n, we get the statement: • If a1 , a2 , . . . , a2n are positive real numbers such that a1 ≤ · · · ≤ an ≤ 1 ≤ an+1 ≤ · · · ≤ a2n , then a12

+ a22

1 1 1 + + ··· + = 2n, a1 a2 a2n

p ‹  n − 2n ≥ 2 1 + (a1 + a2 + · · · + a2n − 2n). 1+n

2 + · · · + a2n

with equality for a1 = a2 = · · · = a2n = 1, and also for a1 =

p 1+ n , 1+n

a2 = a3 = · · · = an = 1,

an+1 = · · · = a2n =

p n+ n . 1+n

P 2.9. Let a1 , a2 , . . . , an be nonnegative real numbers such that a1 + a2 + · · · + an = n. (a) If a1 ≥ 1 ≥ a2 ≥ · · · ≥ an , then 1 a12

+2

+

1 a22

+2

+ ··· +

1 n ≥ ; an2 + 2 3

(b) If a1 ≥ a2 ≥ 1 ≥ a3 ≥ · · · ≥ an , then 1 2a12

+3

+

1 2a22

+3

+ ··· +

1 n ≥ . 2an2 + 3 5 (Vasile Cirtoaje, 2007)

HCF Method for Ordered Variables

121

Solution. Consider the inequality 1 a12

+k

+

1 a22

+k

+ ··· +

an2

1 n ≥ , +k 1+k

k ∈ [0, 3];

and write it as f (a1 ) + f (a2 ) + · · · + f (an ) ≥ n f (s), where f (u) =

1 , u2 + k

s=

a1 + a2 + · · · + an = 1, n

u ∈ [0, n].

For u ∈ [1, n], we have f 00 (u) =

2(3u2 − k) 2(3 − k) ≥ 2 ≥ 0, 2 3 (u + k) (u + k)3

hence f is convex on [1, n]. Therefore, we may apply HCF-OV Theorem or RHCF-OV Corollary for m = 1 and m = 2, respectively. By Remark 1, it suffices to show that h(x, y) ≥ 0 for all x, y ≥ 0 such that x + m y = m + 1. Since g(u) = h(x, y) =

f (u) − f (1) −u − 1 = , u−1 (1 + k)(u2 + k)

xy+x + y−k g(x) − g( y) = , x−y (1 + k)(x 2 + k)( y 2 + k)

we only need to show that x y + x + y − k ≥ 0. (a) We have k = 2, m = 1 and x + y = 2, which involve x y + x + y − k = x y ≥ 0. From x + y = 2 and h(x, y) = 0, we get x = 0 and y = 2. Therefore, in accordance with Remark 7, the equality holds for a1 = a2 = · · · = an = 1, and also for a1 = 2,

a2 = · · · = an−1 = 1,

an = 0.

(b) We have k = 3/2, m = 2 and x + 2 y = 3, which involve xy+x + y−k=

x(1 + 2 y) x(4 − x) = ≥ 0. 2 2

From x + 2 y = 3 and h(x, y) = 0, we get x = 0 and y = 3/2. Therefore, in accordance with Remark 7, the equality holds for a1 = a2 = · · · = an = 1, and also for a1 = a2 = 3/2,

a3 = · · · = an−1 = 1,

an = 0.

122

Vasile Cîrtoaje

P 2.10. If a1 , a2 , . . . , a2n are nonnegative real numbers such that a1 ≥ · · · ≥ an ≥ 1 ≥ an+1 ≥ · · · ≥ a2n ,

a1 + a2 + · · · + a2n = 2n,

then 1 na12

+ n2

+n+1

+

1 na22

+ n2

+n+1

+ ··· +

1 2 na2n

+ n2

+n+1



2n . (n + 1)2

(Vasile Cirtoaje, 2007) Solution. Write the inequality as f (a1 ) + f (a2 ) + · · · + f (a2n ) ≥ 2n f (s), where f (u) =

−1 , nu2 + n2 + n + 1

s=

a1 + a2 + · · · + a2n = 1, 2n

u ∈ [0, 2n].

For u ∈ [0, 1], we have f 00 (u) =

2nu(n2 + n + 1 − 3nu2 ) 2nu(n2 + n + 1 − 3n) ≥ ≥ 0, (nu2 + n2 + n + 1)3 (nu2 + n2 + n + 1)3

hence f is convex on [0, 1]. Therefore, we may apply HCF-OV Theorem or LHCF-OV Corollary for m = n. By Remark 1, it suffices to show that h(x, y) ≥ 0 for all x, y ≥ 0 such that x + n y = n + 1. Since g(u) = h(x, y) =

f (u) − f (1) n(u + 1) = , 2 u−1 (n + 1) (nu2 + n2 + n + 1)

g(x) − g( y) n(n2 + n + 1 − nx − n y − nx y) = , x−y (n + 1)2 (nx 2 + n2 + n + 1)(n y 2 + n2 + n + 1)

we only need to show that n2 + n + 1 − n(x + y + x y) ≥ 0. Indeed, n2 + n + 1 − n(x + y + x y) = (n y − 1)2 ≥ 0. From x + n y = n + 1 and h(x, y) = 0, we get x = n and y = 1/n. Therefore, in accordance with Remark 7, the equality holds for a1 = a2 = · · · = a2n = 1, and also for a1 = n,

a2 = · · · = an = 1,

an+1 = · · · = an = 1/n.

HCF Method for Ordered Variables

123

P 2.11. If a, b, c, d, e, f are nonnegative real numbers such that a ≥ b ≥ c ≥ 1 ≥ d ≥ e ≥ f, then

a + b + c + d + e + f = 6,

3f + 4 3b + 4 3c + 4 3d + 4 3e + 4 3a + 4 + 2 + 2 + 2 + 2 + ≤ 6. 2 3a + 4 3b + 4 3c + 4 3d + 4 3e + 4 3 f 2 + 4 (Vasile Cirtoaje, 2009)

Solution. Write the inequality as f (a) + f (b) + f (c) + f (d) + f (e) + f ( f ) ≥ 6 f (s), where f (u) =

−3u − 4 , 3u2 + 4

s=

a+b+c+d+e+ f = 1, 6

u ∈ [0, 6].

For u ∈ [0, 1], we have f 00 (u) =

6(16 − 9u3 ) + 216u(1 − u) ≥ 0, (3u2 + 4)3

hence f is convex on [0, 1]. Therefore, we may apply HCF-OV Theorem or LHCF-OV Corollary for m = 3. By Remark 1, it suffices to show that h(x, y) ≥ 0 for all x, y ≥ 0 such that x + 3 y = 4. We have g(u) = h(x, y) =

f (u) − f (1) 3u = 2 , u−1 3u + 4

g(x) − g( y) 3(4 − 3x y) 3(x − 2)2 = = ≥ 0. x−y (3x 2 + 4)(3 y 2 + 4) (3x 2 + 4)(3 y 2 + 4)

From x + 3 y = 4 and h(x, y) = 0, we get x = 2 and y = 2/3. Therefore, in accordance with Remark 8, the equality holds only for a = b = c = d = e = f = 1, and also for a = 2, b = c = 1, d = e = f = 2/3.

P 2.12. If a, b, c, d, e, f are nonnegative real numbers such that a ≥ b ≥ 1 ≥ c ≥ d ≥ e ≥ f,

a + b + c + d + e + f = 6,

then f2−1 b2 − 1 c2 − 1 d2 − 1 e2 − 1 a2 − 1 + + + + + ≥ 0. (2a + 7)2 (2b + 7)2 (2c + 7)2 (2d + 7)2 (2e + 7)2 (2 f + 7)2 (Vasile Cirtoaje, 2009)

124

Vasile Cîrtoaje

Solution. Write the inequality as f (a) + f (b) + f (c) + f (d) + f (e) + f ( f ) ≥ 6 f (s), where f (u) =

u2 − 1 , (2u + 7)2

For u ∈ [0, 1], we have f 00 (u) =

s=

a+b+c+d+e+ f = 1, 6

u ∈ [0, 6].

2(37 − 28u) ≥ 0, (2u + 7)4

hence f is convex on [0, 1]. Therefore, we may apply HCF-OV Theorem or LHCF-OV Corollary for m = 4. By Remark 1, it suffices to show that h(x, y) ≥ 0 for all x, y ≥ 0 such that x + 4 y = 5. We have g(u) = h(x, y) =

f (u) − f (1) u+1 = , u−1 (2u + 7)2

g(x) − g( y) 21 − 4x − 4 y − 4x y (x − 4)2 = = ≥ 0. x−y (2x + 7)2 (2 y + 7)2 (2x + 7)2 (2 y + 7)2

From x + 4 y = 5 and h(x, y) = 0, we get x = 4 and y = 1/4. Therefore, in accordance with Remark 8, the equality holds only for a = b = c = d = e = f = 1, and also for a = 4, b = 1, c = d = e = f = 1/4.

P 2.13. If a, b, c, d, e, f are nonnegative real numbers such that a ≥ b ≥ c ≥ d ≥ 1 ≥ e ≥ f,

a + b + c + d + e + f = 6,

then f2−1 a2 − 1 b2 − 1 c2 − 1 d2 − 1 e2 − 1 + + + + + ≤ 0. (2a + 5)2 (2b + 5)2 (2c + 5)2 (2d + 5)2 (2e + 5)2 (2 f + 5)2 (Vasile Cirtoaje, 2009) Solution. Write the inequality as f (a) + f (b) + f (c) + f (d) + f (e) + f ( f ) ≥ 6 f (s), where f (u) =

1 − u2 , (2u + 5)2

s=

u ∈ [0, 6].

a+b+c+d+e+ f = 1, 6

HCF Method for Ordered Variables

125

For u ≥ 1, we have f 00 (u) =

2(20u − 13) ≥ 0, (2u + 5)4

hence f is convex on [1, 6]. Therefore, we may apply HCF-OV Theorem or RHCF-OV Corollary for m = 4. By Remark 1, it suffices to show that h(x, y) ≥ 0 for all x, y ≥ 0 such that x + 4 y = 5. We have g(u) = h(x, y) =

f (u) − f (1) −u − 1 = , u−1 (2u + 5)2

g(x) − g( y) 4x y + 4x + 4 y − 5 x(8 − x) = = ≥ 0. 2 2 x−y (2x + 5) (2 y + 5) (2x + 5)2 (2 y + 5)2

From x + 4 y = 5 and h(x, y) = 0, we get x = 0 and y = 5/4. Therefore, in accordance with Remark 7, the equality holds only for a = b = c = d = e = f = 1, and also for a = b = c = d = 5/4, e = 1, f = 0.

P 2.14. If a, b, c, d are nonnegative real numbers such that a ≥ b ≥ 1 ≥ c ≥ d, then

a + b + c + d = 4,

1 1 1 4 1 + 3 + 3 + 3 ≤ . + 5 2b + 5 2c + 5 2d + 5 7 (Vasile Cirtoaje, 2009)

2a3

Solution. Write the inequality as f (a) + f (b) + f (c) + f (d) ≥ 4 f (s), where f (u) =

−1 , 2u3 + 5

From f 00 (u) =

s=

a+b+c+d = 1, 4

u ∈ [0, 4].

12u(5 − 4u3 ) , (2u3 + 5)3

it follows that f 00 (u) ≥ 0 for u ∈ [0, 1], hence f is convex on [0, 1]. Therefore, we may apply HCF-OV Theorem or LHCF-OV Corollary for n = 4 and m = 2. Thus, it suffices to show that f (x) + 2 f ( y) ≥ 3 f (1) for all x, y ≥ 0 such that x + 2 y = 3. Using Remark 1, we only need to show that h(x, y) ≥ 0. We have g(u) =

f (u) − f (1) 2(u2 + u + 1) = , u−1 7(2u3 + 5)

126

Vasile Cîrtoaje h(x, y) =

g(x) − g( y) 2E = , 3 x−y 7(2x + 5)(2 y 3 + 5)

where E = −2x 2 y 2 − 2x y(x + y) − 2(x 2 + x y + y 2 ) + 5(x + y) + 5. Since E = (1 − 2 y)2 (2 + 3 y − 2 y 2 ) = (1 − 2 y)2 (2 + x y) ≥ 0, the proof is completed. From x + 2 y = 3 and h(x, y) = 0, we get x = 2, y = 1/2. Therefore, in accordance with Remark 8, the equality holds for a = b = c = d = 1, and also for a = 2, b = 1, c = d = 1/2. Remark. Similarly, we can prove the following generalization: • If a1 , a2 , . . . , a2n are nonnegative real numbers such that a1 ≥ · · · ≥ an ≥ 1 ≥ an+1 ≥ · · · ≥ a2n ,

a1 + a2 + · · · + a2n = 2n.

then 1 a13 + n +

1 n

+

1 a23 + n +

1 n

+ ··· +

1 3 a2n +n+

1 n



2n2 , n2 + n + 1

with equality for a1 = a2 = · · · = a2n = 1, and also for a1 = n,

a2 = · · · = an = 1,

an+1 = · · · = a2n = 1/n.

P 2.15. If a1 , a2 , . . . , a8 are nonnegative real numbers such that a1 ≥ a2 ≥ a3 ≥ a4 ≥ 1 ≥ a5 ≥ a6 ≥ a7 ≥ a8 ,

a1 + a2 + · · · + a8 = 8,

then (a12 + 1)(a22 + 1) · · · (a82 + 1) ≥ (a1 + 1)(a2 + 1) · · · (a8 + 1). (Vasile Cirtoaje, 2008) Solution. Write the inequality as f (a1 ) + f (a2 ) + · · · + f (a8 ) ≥ 8 f (s),

s=

a1 + a2 + · · · + a8 = 1, 8

where f (u) = ln(u2 + 1) − ln(u + 1),

u ∈ [0, 8].

HCF Method for Ordered Variables

127

For u ∈ [0, 1], we have f 00 (u) =

1 (u2 − u4 ) + 4u(1 − u2 ) + u2 + 3 2(1 − u2 ) + = > 0. (u2 + 1)2 (u + 1)2 (u2 + 1)2 (u + 1)2

Therefore, f is convex on [0, 1]. According to HCF-OV Theorem or LHCF-OV Corollary applied for n = 8 and m = 4, it suffices to show that f (x) + 4 f ( y) ≥ 5 f (1) for all x, y ≥ 0 such that x + 4 y = 5. Using Remark 2, we only need to show that H(x, y) ≥ 0 for x, y ≥ 0 such that x + 4 y = 5, where H(x, y) =

f 0 (x) − f 0 ( y) 2(1 − x y) 1 = 2 + . 2 x−y (x + 1)( y + 1) (x + 1)( y + 1)

Since 2(x 2 + 1) ≥ (x + 1)2 and 2( y 2 + 1) ≥ ( y + 1)2 , it suffices to prove that 8(1 − x y) + (x + 1)( y + 1) ≥ 0. Indeed, 8(1 − x y) + (x + 1)( y + 1) = 28x 2 − 38x + 14 = 28(x − 19/28)2 + 31/28 > 0. The proof is completed. The equality holds for a1 = a2 = · · · = a8 .

P 2.16. If a, b, c, d are positive real numbers such that a ≥ b ≥ 1 ≥ c ≥ d, then

a bcd = 1,

‹ 1 1 1 1 . a + b + c + d − 4 ≥ 18 a + b + c + d − − − − a b c d 2

2

2

2



(Vasile Cirtoaje, 2008) Solution. Using the substitution a = ex ,

b = ey,

c = ez ,

d = ew,

we need to show that f (x) + f ( y) + f (z) + f (w) ≥ 4 f (s), where x ≥ y ≥ 0 ≥ z ≥ w,

s=

x + y +z+w = 0, 4

f (u) = e2u − 1 − 18(eu − e−u ), u ∈ R. For u ≤ 0, we have

f 00 (u) = 4e2u + 18(e−u − eu ) > 0,

128

Vasile Cîrtoaje

hence f is convex on (−∞, 0]. By HCF-OV Theorem or LHCF-OV Corollary applied for n = 4 and m = 2, it suffices to show that f (x) + 2 f ( y) ≥ 3 f (0) for all real x, y such that x + 2 y = 0; that is, to show that  ‹ 1 2 2 2 a + 2b − 3 − 18 a + 2b − − ≥0 a b for all a, b > 0 such that a b2 = 1. This inequality is equivalent to (b2 − 1)2 (2b2 + 1) 18(b − 1)3 (b + 1) + ≥ 0, b4 b2 (b − 1)2 (2b − 1)2 (b + 1)(5b + 1) ≥ 0. b4 The proof is completed. The equality holds for a = b = c = d = 1, and also for a = 4, b = 1, c = d = 1/2.

P 2.17. If a, b, c, d are positive real numbers such that a ≥ b ≥ 1 ≥ c ≥ d,

a bcd = 1,

then p

a2 − a + 1 +

p

b2 − b + 1 +

p

c2 − c + 1 +

p

d 2 − d + 1 ≥ a + b + c + d. (Vasile Cirtoaje, 2008)

Solution. Using the substitution a = ex ,

b = ey,

c = ez ,

d = ew,

we need to show that f (x) + f ( y) + f (z) + f (w) ≥ 4 f (s), where x ≥ y ≥ 0 ≥ z ≥ w, f (u) =

p

s=

x + y +z+w = 0, 4

e2u − eu + 1 − eu , u ∈ R.

We claim that f is convex for u ≥ 0. Since e−u f 00 (u) =

4e3u − 6e2u + 9eu − 2 − 1, 4(e2u − eu + 1)3/2

HCF Method for Ordered Variables

129

we need to show that (4t 3 − 6t 2 + 9t − 2)2 ≥ 16(t 2 − t + 1)3 , where t = eu ≥ 1. Indeed, (4t 3 − 6t 2 + 9t − 2)2 − 16(t 2 − t + 1)3 = 12t 3 (t − 1) + 9t 2 + 12(t − 1) > 0. By HCF-OV Theorem or RHCF-OV Corollary applied for n = 4 and m = 2, it suffices to show that f (x) + 2 f ( y) ≥ 3 f (0) for all real x, y such that x + 2 y = 0; that is, to show that p p a2 − a + 1 + 2 b2 − b + 1 ≥ a + 2b for all a, b > 0 such that a b2 = 1. This inequality is equivalent to p p b4 − b2 + 1 1 + 2 b2 − b + 1 ≥ 2 + 2b, 2 b b p

b2 − 1 b4 − b2 + 1 + 1

Since p it suffices to show that

+p

b2 − 1 b4 − b2 + 1

2(1 − b) b2 − b + 1 + b ≥

≥ 0.

b2 − 1 , b2 + 1

b2 − 1 2(1 − b) +p ≥ 0, 2 b +1 b2 − b + 1 + b

which is equivalent to ˜ b+1 2 ≥ 0, − p b2 + 1 b2 − b + 1 + b ” — p (b − 1) (b + 1) b2 − b + 1 − b2 + b − 2 ≥ 0, (b − 1)

•

(b − 1)2 (3b2 − 2b + 3) ≥ 0. p (b + 1) b2 − b + 1 + b2 − b + 2 Clearly, the last inequality is true. The equality holds for a = b = c = d = 1.

P 2.18. If a, b, c, d are positive real numbers such that a ≥ 1 ≥ b ≥ c ≥ d, then a3

a bcd = 1,

1 1 1 1 2 + 3 + 3 + 3 ≥ . + 3a + 2 b + 3b + 2 c + 3c + 2 d + 3d + 2 3 (Vasile Cirtoaje, 2007)

130

Vasile Cîrtoaje

Solution. Using the substitution a = ex ,

b = ey,

c = ez ,

d = ew,

we need to show that f (x) + f ( y) + f (z) + f (w) ≥ 4 f (s), where x ≥ y ≥ 0 ≥ z ≥ w, f (u) =

s=

x + y +z+w = 0, 4

1 , u ∈ R. e3u + 3eu + 2

We claim that f is convex for u ≥ 0. Indeed, denoting t = eu , t ≥ 1, we have 3t(3t 5 + 2t 3 − 6t 2 + 3t − 2) (t 3 + 3t + 2)3 3t(t − 1)(3t 4 + 3t 3 + 5t 2 − t + 2) = ≥ 0. (t 3 + 3t + 2)3

f 00 (u) =

By HCF-OV Theorem or RHCF-OV Corollary applied for n = 4 and m = 1, it suffices to show that f (x) + f ( y) ≥ 2 f (0) for all real x, y such that x + y = 0; that is, to show that 1 1 1 + ≥ a3 + 3a + 2 b3 + 3b + 2 3 for all a, b > 0 such that a b = 1. This inequality is equivalent to (a − 1)4 (a2 + a + 1) ≥ 0, which is clearly true. The equality holds for a = b = c = d = 1.

P 2.19. Let a1 , a2 , . . . , an be positive real numbers such that a1 ≥ 1 ≥ a2 ≥ · · · ≥ an , If k ≥ 1, then

a1 a2 · · · an = 1.

1 1 1 n + + ··· + ≥ . 1 + ka1 1 + ka2 1 + kan 1+k (Vasile Cirtoaje, 2007)

HCF Method for Ordered Variables

131

Solution. Using the substitutions ai = e x i for i = 1, 2, . . . , n, we need to show that f (x 1 ) + f (x 2 ) + · · · + f (x n ) ≥ n f (s), where x1 ≥ 0 ≥ x2 ≥ · · · ≥ x n,

s=

x1 + x2 + · · · + x n = 0, n

f (u) =

1 , u ∈ R. 1 + keu

f 00 (u) =

keu (keu − 1) ≥ 0, (1 + keu )3

For u ≥ 0, we have

therefore f (u) is convex for u ≥ 0. By HCF-OV Theorem or RHCF-OV Corollary applied for m = 1, it suffices to show that f (x) + f ( y) ≥ 2 f (0) for all real x, y such that x + y = 0; that is, to show that 1 1 2 + ≥ 1 + ka 1 + k b 1+k for all a, b > 0 such that a b = 1. This is true since 1 1 2 k(k − 1)(a − 1)2 + − = ≥ 0. 1 + ka 1 + k b 1 + k (1 + ka)(a + k) The equality holds for a1 = a2 = · · · = an = 1.

P 2.20. If a1 , a2 , . . . , a9 are positive real numbers such that a1 ≥ 1 ≥ a2 ≥ · · · ≥ a9 , then

a1 a2 · · · a9 = 1,

1 1 1 + + ··· + ≥ 1. (a1 + 2)2 (a2 + 2)2 (a9 + 2)2 (Vasile Cirtoaje, 2007)

Solution. Using the substitution ai = e x i ,

i = 1, 2, . . . , 9,

we can write the inequality as f (x 1 ) + f (x 2 ) + · · · + f (x 9 ) ≥ 9 f (s),

132

Vasile Cîrtoaje

where x1 ≥ 0 ≥ x2 ≥ · · · ≥ x9, f (u) =

(eu

For u ∈ [0, ∞), we have f 00 (u) =

s=

x1 + x2 + · · · + x9 9

1 , + 2)2

= 0,

u ∈ R.

4eu (eu − 1) ≥ 0, (eu + 2)4

hence f is convex on [0, ∞). According to HCF-OV Theorem or RHCF-OV Corollary (case m = 1), it suffices to show that f (x) + f ( y) ≥ 2 f (0) for all real x, y such that x + y = 0; that is, to show that 1 1 2 + ≥ 2 2 (a + 2) (b + 2) 9 for all a, b > 0 such that a b = 1. Write this inequality as b2 1 2 + ≥ , 2 2 (2b + 1) (b + 2) 9 which is equivalent to the obvious inequality (b − 1)4 ≥ 0. The equality holds for a1 = a2 = · · · = a9 = 1. Remark. Similarly, we can prove the following generalization. • Let a1 , a2 , · · · , an be positive real numbers such that a1 a2 · · · an = 1.

a1 ≥ 1 ≥ a2 ≥ · · · ≥ an , If p, q ≥ 0 such that p+q ≥1+ then

1 1+

pa1 + qa12

+

1 1+

pa2 + qa22

with equality for a1 = a2 = · · · = an = 1.

2pq , p + 4q

+ ··· +

1 n ≥ . 2 1 + pan + qan 1+p+q

HCF Method for Ordered Variables

133

P 2.21. If a1 , a2 , . . . , an (n ≥ 4) are positive real numbers such that a1 a2 · · · an = 1,

a1 ≥ a2 ≥ a3 ≥ 1 ≥ a4 ≥ · · · ≥ an , then

1 1 1 n + + ··· + ≥ . 2 2 2 (a1 + 1) (a2 + 1) (an + 1) 4 (Vasile Cirtoaje, 2007)

Solution. Using the substitution ai = e x i ,

i = 1, 2, . . . , n,

we can write the inequality as f (x 1 ) + f (x 2 ) + · · · + f (x n ) ≥ n f (s), where x1 ≥ x2 ≥ x3 ≥ 0 ≥ x4 ≥ · · · ≥ x n, f (u) =

(eu

For u ∈ [0, ∞), we have f 00 (u) =

1 , + 1)2

s=

x1 + x2 + · · · + x n = 0, n

u ∈ R.

2eu (2eu − 1) > 0, (eu + 1)4

hence f is convex on [0, ∞). According to HCF-OV Theorem or RHCF-OV Corollary (case m = 3), it suffices to show that f (x) + 3 f ( y) ≥ 4 f (0) for all real x, y such that x + 3 y = 0; that is, to show that 1 3 + ≥1 2 (a + 1) (b + 1)2 for all a, b > 0 such that a b3 = 1. The inequality is equivalent to 3 b6 + ≥ 1. 3 2 (b + 1) (b + 1)2 Using the Cauchy-Schwarz inequality, it suffices to show that (b3 + 3)2 ≥ 1, (b3 + 1)2 + 3(b + 1)2 which is equivalent to the obvious inequality (b − 1)2 (4b + 5) ≥ 0. The equality holds for a1 = a2 = · · · = an = 1.

134

Vasile Cîrtoaje

P 2.22. If a1 , a2 , . . . , an are positive real numbers such that a1 ≤ 1 ≤ a2 ≤ · · · ≤ an , then

a1 a2 · · · an = 1,

1 1 n 1 + + ··· + ≤ . 2 2 2 (a1 + 3) (a2 + 3) (an + 3) 16 (Vasile Cirtoaje, 2007)

Solution. Using the substitution ai = e x i ,

i = 1, 2, . . . , n,

we can write the inequality as f (x 1 ) + f (x 2 ) + · · · + f (x n ) ≥ n f (s), where s=

x1 ≤ 0 ≤ x2 ≤ · · · ≤ x n, f (u) =

−1 , + 3)2

(eu

x1 + x2 + · · · + x n = 0, n u ∈ R.

For u ∈ (−∞, 0], we have f 00 (u) =

2eu (3 − 2eu ) > 0, (eu + 3)4

hence f is convex on (−∞, 0]. According to HCF-OV Theorem or LHCF-OV Corollary (case m = 1), it suffices to show that f (x) + f ( y) ≥ 2 f (0) for all real x, y such that x + y = 0; that is, to show that 1 1 1 + ≤ 2 2 (a + 3) (b + 3) 8 for all a, b > 0 such that a b = 1. Write this inequality as b2 1 1 + ≤ , 2 2 (3b + 1) (b + 3) 8 which is equivalent to the obvious inequality (b2 − 1)2 + 12b(b − 1)2 ≥ 0. The equality holds for a1 = a2 = · · · = an = 1. Remark. Similarly, we can prove the following generalization:

HCF Method for Ordered Variables

135

• Let a1 , a2 , . . . , an be positive real numbers such that a1 a2 · · · an = 1.

a1 ≤ 1 ≤ a2 ≤ · · · ≤ an , p If k ≥ 1 + 2, then

1 1 1 n + + ··· + ≤ , (a1 + k)2 (a2 + k)2 (an + k)2 (1 + k)2 with equality for a1 = a2 = · · · = an = 1.

P 2.23. Let a1 , a2 , . . . , an be positive real numbers such that a1 a2 · · · an = 1.

a1 ≤ 1 ≤ a2 ≤ · · · ≤ an , If p, q ≥ 0 such that p + q ≤ 1, then 1 1+

pa1 + qa12

+

1 1+

pa2 + qa22

+ ··· +

n 1 ≤ . 1 + pan + qan2 1+p+q (Vasile Cirtoaje, 2007)

Solution. Using the substitution ai = e x i ,

i = 1, 2, . . . , n,

we can write the inequality as f (x 1 ) + f (x 2 ) + · · · + f (x n ) ≥ n f (s), where x1 ≤ 0 ≤ x2 ≤ · · · ≤ x n, f (u) =

s=

x1 + x2 + · · · + x n = 0, n

−1 , 1 + peu + qe2u

u ∈ R.

For u ≤ 0, we have eu [−4q2 e3u − 3pqe2u + (4q − p2 )eu + p] (1 + peu + qe2u )3 e2u [−4q2 − 3pq + (4q − p2 ) + p] ≥ (1 + peu + qe2u )3 e2u [(p + 4q)(1 − p − q) + 2pq] = ≥ 0, (1 + peu + qe2u )3

f 00 (u) =

136

Vasile Cîrtoaje

therefore f (u) is convex for u ≤ 0. According to HCF-OV Theorem or LHCF-OV Corollary (case m = 1), it suffices to show that f (x) + f ( y) ≥ 2 f (0) for all real x, y such that x + y = 0; that is, to show that 1 1 2 + ≤ 2 2 1 + pa + qa 1 + pb + qb 1+p+q for all a, b > 0 such that a b = 1. Write this inequality as (a − 1)2 [q(1 − p − q)a2 + (p + 2q − p2 − pq − 2q2 )a + q(1 − p − q)] ≥ 0, which is true because p + 2q − p2 − pq − 2q2 ≥ (p + 2q)(p + q) − p2 − pq − 2q2 = 2pq ≥ 0. The equality holds for a1 = a2 = · · · = an = 1.

P 2.24. If a1 , a2 , . . . , an are positive real numbers such that a1 ≤ a2 ≤ 1 ≤ a3 ≤ · · · ≤ an , then

a1 a2 · · · an = 1,

1 1 1 n + + ··· + ≤ . 2 2 2 (a1 + 5) (a2 + 5) (an + 5) 36 (Vasile Cirtoaje, 2007)

Solution. Using the substitution ai = e x i ,

i = 1, 2, . . . , n,

we can write the inequality as f (x 1 ) + f (x 2 ) + · · · + f (x n ) ≥ n f (s), where x1 ≤ x2 ≤ 0 ≤ x3 ≤ · · · ≤ x n, f (u) =

s=

−1 , (eu + 5)2

x1 + x2 + · · · + x n = 0, n u ∈ R.

For u ∈ (−∞, 0], we have f 00 (u) =

2eu (5 − 2eu ) > 0, (eu + 5)4

HCF Method for Ordered Variables

137

hence f is convex on (−∞, 0]. According to HCF-OV Theorem or LHCF-OV Corollary (case m = 2), it suffices to show that f (x) + 2 f ( y) ≥ 3 f (0) for all real x, y such that x + 2 y = 0; that is, to show that 1 2 1 + ≤ 2 2 (a + 5) (b + 5) 12 for all a, b > 0 such that a b2 = 1. Since 1 b4 b4 b2 = ≤ = , (a + 5)2 (5b2 + 1)2 (4b2 + 2b)2 4(2b + 1)2 it suffices to show that

2 1 b2 + ≤ , 4(2b + 1)2 (b + 5)2 12

which is equivalent to the obvious inequality (b − 1)2 (b2 + 16b + 1) ≥ 0. The equality holds for a1 = a2 = · · · = an = 1. Remark. Similarly, we can prove the following generalization: • Let a1 , a2 , . . . , an be positive real numbers such that a1 ≤ a2 ≤ 1 ≤ a3 ≤ · · · ≤ an , If k ≥ 2 +

a1 a2 · · · an = 1.

p 6, then 1 1 1 n + + ··· + ≤ , 2 2 2 (a1 + k) (a2 + k) (an + k) (1 + k)2

with equality for a1 = a2 = · · · = an = 1.

P 2.25. If a1 , a2 , . . . , an are positive real numbers such that a1 ≥ 1 ≥ a2 ≥ · · · ≥ an , then

1 p

1 + 3a1

+p

1 1 + 3a2

a1 a2 · · · an = 1,

+ ··· + p

1 1 + 3an



n . 2 (Vasile Cirtoaje, 2007)

138

Vasile Cîrtoaje

Solution. Using the substitution ai = e x i ,

i = 1, 2, . . . , n,

we can write the inequality as f (x 1 ) + f (x 2 ) + · · · + f (x n ) ≥ n f (s), where x1 ≥ 0 ≥ x2 ≥ · · · ≥ x n,

s=

x1 + x2 + · · · + x n = 0, n

1 f (u) = p , 1 + 3eu

u ∈ R.

For u ≥ 0, we have

eu (3eu − 2) > 0, 2(1 + 3eu )5/2 hence f is convex on [0, ∞). According to HCF-OV Theorem or RHCF-OV Corollary (case m = 1), it suffices to show that f (x) + f ( y) ≥ 2 f (0) for all real x, y such that x + y = 0; that is, to show that f 00 (u) =

1 1 +p ≥1 p 1 + 3a 1 + 3b for all a, b > 0 such that a b = 1. Write this inequality as s 1 a + ≥ 1. p a+3 1 + 3a 1 Substituting p = t, 0 < t < 1, the inequality becomes 1 + 3a v t 1 − t2 ≥ 1 − t. 8t 2 + 1 By squaring, we get t(1 − t)(2t − 1)2 ≥ 0, which is true. The equality holds for a1 = a2 = · · · = an = 1.

P 2.26. If a1 , a2 , . . . , an are positive real numbers such that a1 ≤ 1 ≤ a2 ≤ · · · ≤ an , then

1 p

1 + 2a1

+p

1 1 + 2a2

a1 a2 · · · an = 1,

+ ··· + p

1

n ≤p . 3 1 + 2an (Vasile Cirtoaje, 2007)

HCF Method for Ordered Variables

139

Solution. Using the substitution ai = e x i ,

i = 1, 2, . . . , n,

we can write the inequality as f (x 1 ) + f (x 2 ) + · · · + f (x n ) ≥ n f (s), where x1 ≤ 0 ≤ x2 ≤ · · · ≤ x n,

s=

x1 + x2 + · · · + x n = 0, n

−1 f (u) = p , 1 + 2eu For u ≤ 0, we have f 00 (u) =

u ∈ R.

eu (1 − eu ) > 0, (1 + 2eu )5/2

hence f is convex on (−∞, 0]. According to HCF-OV Theorem or LHCF-OV Corollary (case m = 1), it suffices to show that f (x) + f ( y) ≥ 2 f (0) for all real x, y such that x + y = 0; that is, to show that v v t 3 t 3 + ≤2 1 + 2a 1 + 2b for all a, b > 0 such that a b = 1. By the Cauchy-Schwarz inequality, we get v v v ‹ ‹ t 3 t 3 t 3 3 + ≤ +1 1+ = 2. 1 + 2a 1 + 2b 1 + 2a 1 + 2b The equality holds for a1 = a2 = · · · = an = 1.

140

Vasile Cîrtoaje

Chapter 3

Partially Convex Function Method 3.1

Theoretical Basis

Right Partially Convex Function Theorem (Vasile Cirtoaje, 2012). Let f be a function defined on a real interval I and convex on [s, s0 ], where s, s0 ∈ I, s < s0 . In addition, f (u) is decreasing on Iu≤s0 and min f (u) = f (s0 ). u∈I

The inequality a + a + ··· + a  1 2 n f (a1 ) + f (a2 ) + · · · + f (an ) ≥ n f n holds for all a1 , a2 , · · · , an ∈ I satisfying a1 + a2 + · · · + an = ns if and only if f (x) + (n − 1) f ( y) ≥ n f (s) for all x, y ∈ I such that x ≤ s ≤ y and x + (n − 1) y = ns. Proof. The necessity is obvious. By Lemma below, to prove the sufficiency, it suffices to consider that a1 , a2 , . . . , an ∈ J, where J = Iu≤s0 . Because f (u) is convex on Ju≥s , the desired inequality follows from HCF Theorem applied to the interval J. Lemma. Let f be a function defined on a real interval I and convex on [s, s0 ], where s, s0 ∈ I, s < s0 . In addition, f (u) is decreasing on Iu≤s0 and min f (u) = f (s0 ). u∈I

If the inequality f (a1 ) + f (a2 ) + · · · + f (an ) ≥ n f (s) 141

142

Vasile Cîrtoaje

holds for all a1 , a2 , · · · , an ∈ Iu≤s0 such that a1 + a2 + · · · + an = ns, then it holds for all a1 , a2 , . . . , an ∈ I such that a1 + a2 + · · · + an = ns. Proof. For i = 1, 2, . . . , n, define the numbers bi ∈ Iu≤s0 as follows ( bi =

a i , a i ≤ s0 s0 ,

a i > s0 .

We have bi ≤ ai and f (bi ) ≤ f (ai ) for i = 1, 2, · · · , n. Therefore, b1 + b2 + · · · + bn ≤ a1 + a2 + · · · + an = ns and f (b1 ) + f (b2 ) + · · · + f (bn ) ≤ f (a1 ) + f (a2 ) + · · · + f (an ). Thus, it suffices to show that f (b1 ) + f (b2 ) + · · · + f (bn ) ≥ n f (s) for all b1 , b2 , . . . , bn ∈ Iu≤s0 such that b1 + b2 +· · ·+ bn ≤ ns. By hypothesis, this inequality is true for b1 , b2 , . . . , bn ∈ Iu≤s0 and b1 + b2 + · · · + bn = ns. Since f (u) is decreasing on Iu≤s0 , the more we have f (b1 ) + f (b2 ) + · · · + f (bn ) ≥ n f (s) for b1 , b2 , . . . , bn ∈ Iu≤s0 and b1 + b2 + · · · + bn ≤ ns. Similarly, we can prove Left Partially Convex Function Theorem (Vasile Cirtoaje, 2012). Left Partially Convex Function Theorem. Let f be a function defined on a real interval I and convex on [s0 , s], where s0 , s ∈ I, s0 < s. In addition, f (u) is increasing on Iu≥s0 and satisfies min f (u) = f (s0 ). u∈I

The inequality f (a1 ) + f (a2 ) + · · · + f (an ) ≥ n f

a + a + ··· + a  1 2 n n

holds for all a1 , a2 , · · · , an ∈ I satisfying a1 + a2 + · · · + an = ns if and only if f (x) + (n − 1) f ( y) ≥ n f (s) for all x, y ∈ I such that x ≥ s ≥ y and x + (n − 1) y = ns. Remark 1. Let us denote g(u) =

f (u) − f (s) , u−s

h(x, y) =

g(x) − g( y) . x−y

Partially Convex Function Method

143

As shown in Remark 1 from 1.1, we may replace the hypothesis condition in Right Partially Convex Function Theorem (RPCF Theorem) and Left Partially Convex Function Theorem (LPCF Theorem), namely f (x) + (n − 1) f ( y) ≥ n f (s), by the condition h(x, y) ≥ 0 for all x, y ∈ I such that x + (n − 1) y = ns. Remark 2. Assume that f is differentiable on I, and let H(x, y) =

f 0 (x) − f 0 ( y) . x−y

As shown in Remark 2 from 1.1, the inequalities in RPCF Theorem and LPCF Theorem hold true by replacing the hypothesis f (x) + (n − 1) f ( y) ≥ n f (s) with the more restrictive condition H(x, y) ≥ 0 for all x, y ∈ I such that x + (n − 1) y = ns. Remark 3. The inequality in RPCF Theorem or LPCF Theorem becomes an equality for a1 = a2 = · · · = an = s and also for a1 = x,

a2 = · · · = an = y

(or any cyclic permutation), where x, y ∈ I satisfy the equations x + (n − 1) y = ns,

f (x) + (n − 1) f ( y) = n f (s).

For x 6= y, these equations are equivalent to x + (n − 1) y = ns, h(x, y) = 0. Remark 4. RPCF Theorem is also valid in the case when I = [a, b] \ {u0 } or I = (a, b) \ {u0 }, where a, b, u0 are real numbers such that a < s < s0 < u0 < b. Similarly, LPCF Theorem is also valid in the case when I = [a, b]\{u0 } or I = (a, b)\{u0 }, where a, b, u0 are real numbers such that a < u0 < s0 < s < b.

144

Vasile Cîrtoaje

Remark 5. The desired inequality in RPCF Theorem holds true by replacing the condition f is decreasing on Iu≤s0 with the sufficient condition ns − (n − 1)s0 ≤ inf I. To prove this claim, we define the function ( f (u), u ≤ s0 , u ∈ I f0 (u) = f (s0 ), u ≥ s0 , u ∈ I, which is convex for u ∈ I, u ≥ s. Taking into account that f0 (s) = f (s) and f0 (u) ≤ f (u) for all u ∈ I, it suffices to prove that f0 (x 1 ) + f0 (x 2 ) + · · · + f0 (x n ) ≥ n f0 (s) for all x 1 , x 2 , · · · , x n ∈ I satisfying x 1 + x 2 + · · · + x n = ns. According to HCF Theorem and Remark 5 from Section 1, we only need to show that f0 (x) + (n − 1) f0 ( y) ≥ n f0 (s)

(*)

for all x, y ∈ I such that x ≤ s ≤ y and x +(n−1) y = ns. The case y > s0 is not possible because x = ns − (n − 1) y < ns − (n − 1)s0 ≤ inf I involves x 6∈ I. For the possible case y ≤ s0 , (*) becomes f (x) + (n − 1) f ( y) ≥ n f (s), which holds (by hypothesis) for all x, y ∈ I such that x + (n − 1) y = ns. Similarly, we can show that the desired inequality in LPCF Theorem holds true by replacing the condition f is increasing on Iu≥s0 with the sufficient condition ns − (n − 1)s0 ≥ sup I.

Partially Convex Function Method

3.2

145

Applications

3.1. If a, b, c are real numbers such that a + b + c = 3, then 16a − 5 16b − 5 16c − 5 + + ≤ 1. 32a2 + 1 32b2 + 1 32c 2 + 1

3.2. If a, b, c, d are real numbers such that a + b + c + d = 4, then 18b − 5 18c − 5 18d − 5 18a − 5 + + + ≤ 4. 12a2 + 1 12b2 + 1 12c 2 + 1 12d 2 + 1

3.3. If a, b, c, d, e, f are real numbers such that a + b + c + d + e + f = 6, then 5f − 1 5a − 1 5b − 1 5c − 1 5d − 1 5e − 1 + + + + + ≤ 4. 5a2 + 1 5b2 + 1 5c 2 + 1 5d 2 + 1 5e2 + 1 5 f 2 + 1

3.4. If a1 , a2 , · · · , an (n ≥ 3) are real numbers such that a1 + a2 + · · · + an = n, then n(n + 1) − 2a1 n2 + (n − 2)a12

+

n(n + 1) − 2a2 n2 + (n − 2)a22

+ ··· +

n(n + 1) − 2an ≤ n. n2 + (n − 2)an2

3.5. If a, b, c, d are real numbers such that a + b + c + d = 4, then a(a − 1) b(b − 1) c(c − 1) d(d − 1) + + 2 + ≥ 0. 3a2 + 4 3b2 + 4 3c + 4 3d 2 + 4

3.6. Let a1 , a2 , · · · , an 6= −k be real numbers such that a1 + a2 + · · · + an = n, where n k≥ p . 2 n−1 Then,

an (an − 1) a1 (a1 − 1) a2 (a2 − 1) + + ··· + ≥ 0. 2 2 (a1 + k) (a2 + k) (an + k)2

146

Vasile Cîrtoaje

3.7. Let a1 , a2 , . . . , an 6= −k be real numbers such that a1 + a2 + · · · + an = n. If k ≥1+ p then

a12 − 1 (a1 + k)2

+

a22 − 1 (a2 + k)2

n n−1

+ ··· +

,

an2 − 1 (an + k)2

≥ 0.

3.8. Let a1 , a2 , . . . , an be nonnegative real numbers such that a1 + a2 + · · · + an ≥ n. If k > 1, then a1 a1k

+ a2 + · · · + an

+

a2 a1 + a2k

+ · · · + an

+ ··· +

an ≤ 1. a1 + a2 + · · · + ank

3.9. Let a1 , a2 , . . . , an be nonnegative real numbers such that a1 + a2 + · · · + an ≥ n. If k > 1, then 1 a1k

+ a2 + · · · + an

+

1 a1 + a2k

+ · · · + an

+ ··· +

1 ≤ 1. a1 + a2 + · · · + ank

3.10. If a1 , a2 , . . . , an are nonnegative real numbers such that a1 + a2 + · · · + an = n, then an − 1 a1 − 1 a2 − 1 + + ··· + ≤ 0. 2 2 a1 + a2 + · · · + an2 a1 + a2 + · · · + an a1 + a2 + · · · + an 3.11. Let a1 , a2 , · · · , an be positive real numbers such that a1 + a2 + · · · + an = n. If n , then 0 0. (32u2 + 1)3 Therefore, f is convex on [1/2, 1], hence on [s0 , 1]. According to LPCF Theorem, we only need to show that f (x) + 2 f ( y) ≥ 3 f (1) for all real x, y such that x + 2 y = 3. Using Remark 1, it suffices to prove that h(x, y) ≥ 0, where h(x, y) =

g(x) − g( y) , x−y

g(u) =

f (u) − f (1) . u−1

152

Vasile Cîrtoaje

Indeed, we have g(u) = h(x, y) =

32(2u − 1) , 3(32u2 + 1)

64(1 + 16x + 16 y − 32x y) 64(4x − 5)2 = ≥ 0. 3(32x 2 + 1)(32 y 2 + 1) 3(32x 2 + 1)(32 y 2 + 1)

Thus, the proof is completed. From x + 2 y = 3 and h(x, y) = 0, we get x = 5/4 and, y = 7/8. Therefore, in accordance with Remark 3, the equality holds for a = b = c = 1, and also for a = 5/4 and b = c = 7/8 (or any cyclic permutation).

P 3.2. If a, b, c, d are real numbers such that a + b + c + d = 4, then 18b − 5 18c − 5 18d − 5 18a − 5 + + + ≤ 4. 12a2 + 1 12b2 + 1 12c 2 + 1 12d 2 + 1 (Vasile Cîrtoaje, 2012) Solution. Write the inequality as f (a) + f (b) + f (c) + f (d) ≥ 4 f (s), where

5 − 18u , 12u2 + 1

f (u) =

s=

a+b+c+d = 1, 4

u ∈ R.

From

6(36u2 − 20u − 3) , (12u2 + 1)2    p p  5 − 52 5 − 52 ∪[s0 , ∞) and decreasing on , s0 , it follows that f is increasing on −∞, 18 18 p 5 + 52 ≈ 0.678. Since where s0 = 18 f 0 (u) =

lim f (u) = 0

u→−∞

and f (s0 ) < f (5/18) = 0, it follows that min f (u) = f (s0 ). u∈R

For u ∈ [5/18, 1], we have 1 00 −216u3 + 180u2 + 54u − 5 f (u) = 24 (12u2 + 1)3 216u2 (1 − u) + 36u(1 − u) + (18u − 5) = > 0. (32u2 + 1)3

Partially Convex Function Method

153

Therefore, f is convex on [1/2, 1], hence on [s0 , 1]. According to LPCF Theorem and Remark 1, we only need to show that h(x, y) ≥ 0 for x, y ∈ R such that x + 3 y = 4. We have f (u) − f (1) 6(2u − 1) g(u) = = , u−1 12u2 + 1 h(x, y) =

g(x) − g( y) 12(1 + 6x + 6 y − 12x y) 12(2x − 3)2 = = ≥ 0. x−y (12x 2 + 1)(12 y 2 + 1) (12x 2 + 1)(12 y 2 + 1)

Thus, the proof is completed. From x + 3 y = 4 and h(x, y) = 0, we get x = 3/2 and, y = 5/6. Therefore, in accordance with Remark 3, the equality holds for a = b = c = d = 1, and also for a = 3/2 and b = c = d = 5/6 (or any cyclic permutation).

P 3.3. If a, b, c, d, e, f are real numbers such that a + b + c + d + e + f = 6, then 5f − 1 5b − 1 5c − 1 5d − 1 5e − 1 5a − 1 + 2 + 2 + 2 + 2 + ≤ 4. 2 5a + 1 5b + 1 5c + 1 5d + 1 5e + 1 5 f 2 + 1 (Vasile Cîrtoaje, 2012) Solution. Write the inequality as f (a) + f (b) + f (c) + f (d) + f (e) + f ( f ) ≥ 4 f (s), where f (u) =

1 − 5u , 5u2 + 1

s=

a+b+c+d+e+ f = 1, 6

u ∈ R.

From

5(5u2 − 2u − 1) , (5u2 + 1)2    p  p 1− 6 1− 6 , s0 , it follows that f is increasing on −∞, ∪[s0 , ∞) and decreasing on 5 5 p 1+ 6 where s0 = ≈ 0.69. Since 5 f 0 (u) =

lim f (u) = 0

u→−∞

and f (s0 ) < f (1/5) = 0, it follows that min f (u) = f (s0 ). u∈R

154

Vasile Cîrtoaje

For u ∈ [1/5, 1], we have −25u3 + 15u2 + 15u − 1 1 00 f (u) = 10 (5u2 + 1)3 2 25u (1 − u) + 10u(1 − u) + (5u − 1) = > 0. (3u2 + 4)3 Therefore, f is convex on [1/5, 1], hence on [s0 , 1]. According to LPCF Theorem and Remark 1, we only need to show that h(x, y) ≥ 0 for x, y ∈ R such that x + 5 y = 6. We have f (u) − f (1) 5(2u − 1) g(u) = = , u−1 3(5u2 + 1) h(x, y) =

g(x) − g( y) 5(2 + 5x + 5 y − 10x y) 10(x − 2)2 = = ≥ 0. x−y 3(5x 2 + 1)(5 y 2 + 1) 3(5x 2 + 1)(5 y 2 + 1)

Thus, the proof is completed. In accord with Remark 3, the equality holds for a = b = c = d = e = f = 1, and also for a = 2 and b = c = d = e = f = 4/5 (or any cyclic permutation).

P 3.4. If a1 , a2 , · · · , an (n ≥ 3) are real numbers such that a1 + a2 + · · · + an = n, then n(n + 1) − 2a1 n2

+ (n − 2)a12

+

n(n + 1) − 2a2 n2

+ (n − 2)a22

+ ··· +

n(n + 1) − 2an ≤ n. n2 + (n − 2)an2 (Vasile Cîrtoaje, 2008)

Solution. The desired inequality is true for a1 > n(n + 1) − 2a1 n2 + (n − 2)a12 and

n(n + 1) − 2ai n2

+ (n − 2)ai2


(n − 1)(ai + 1)2 ≥ 0.

Partially Convex Function Method

155

Consider further that a1 , a2 , · · · , an ≤

n(n + 1) and rewrite the desired inequality as 2

f (a1 ) + f (a2 ) + · · · + f (an ) ≥ n f (s), where

2u − n(n + 1) , f (u) = (n − 2)u2 + n2

s=

a1 + a2 + · · · + an = 1, n

 ˜ n(n + 1) u ∈ I = −∞, . 2

We have f 0 (u) n2 + n(n + 1)u − u2 = 2(n − 2) [(n − 2)u2 + n2 ]2 and

f1 (u) f 00 (u) = , 2(n − 2) [(n − 2)u2 + n2 ]3

where f1 (u) = 2(n − 2)u3 − 3n(n + 1)(n − 2)u2 − 2n2 (2n − 3)u + n3 (n + 1). From of f 0 , it follows that f is decreasing on (−∞, s0 ] and increasing • the expression ˜ n(n + 1) , where on s0 , 2 s0 =

p n (n + 1 − n2 + 2n + 5 ) ∈ (−1, 0); 2

therefore, min f (u) = f (s0 ). u∈I

On the other hand, for −1 ≤ u ≤ 1, we have f1 (u) > −2(n − 2) − 3n(n + 1)(n − 2) − 2n2 (2n − 3) + n3 (n + 1) = n2 (n − 3)2 + 4(n + 1) > 0, and hence f 00 (u) > 0. Since [s0 , 1] ⊂ [−1, 1], f is convex on [s0 , 1]. By LPCF Theorem and Remark 1, we only need to show that h(x, y) ≥ 0 for x, y ∈ R and x + (n − 1) y = n, where g(x) − g( y) f (u) − f (1) h(x, y) = , g(u) = . x−y u−1 Indeed, we have g(u) =

(n − 2)u + n (n − 2)u2 + n2

156

Vasile Cîrtoaje

and n2 − n(x + y) − (n − 2)x y h(x, y) = n−2 [(n − 2)x 2 + n2 ][(n − 2) y 2 + n2 ] (n − 1)(n − 2) y 2 = ≥ 0. [(n − 2)x 2 + n2 ][(n − 2) y 2 + n2 ] The proof is completed. In accord with Remark 3, the equality holds for a1 = a2 = · · · = an = 1, and also for a1 = n and a2 = · · · = an = 0 (or any cyclic permutation).

P 3.5. If a, b, c, d are real numbers such that a + b + c + d = 4, then a(a − 1) b(b − 1) c(c − 1) d(d − 1) + + 2 + ≥ 0. 3a2 + 4 3b2 + 4 3c + 4 3d 2 + 4 (Vasile Cîrtoaje, 2012) Solution. Write the inequality as f (a) + f (b) + f (c) + f (d) ≥ 4 f (s), where f (u) =

u2 − u , 3u2 + 4

s=

a+b+c+d = 1, 4

u ∈ R.

From

3u2 + 8u − 4 , (3u2 + 4)2    p p  −4 − 2 7 −4 − 2 7 it follows that f is increasing on −∞, ∪[s0 , ∞) and decreasing on , s0 , 3 3 p −4 + 2 7 ≈ 0.43. Since where s0 = 3 f 0 (u) =

lim f (u) = 1/3

u→−∞

and f (s0 ) < f (0) = 0, it follows that min f (u) = f (s0 ). u∈R

For u ∈ [0, 1], we have 1 00 −9u3 − 36u2 + 36u + 14 f (u) = 2 (3u2 + 4)3 9u2 (1 − u) + 45u(1 − u) + (16 − 9u) = > 0. (3u2 + 4)3

Partially Convex Function Method

157

Therefore, f is convex on [0, 1], hence on [s0 , 1]. According to LPCF Theorem and Remark 1, we only need to show that h(x, y) ≥ 0 for x, y ∈ R such that x + 3 y = 4. We have f (u) − f (1) u) g(u) = = 2 , u−1 3u + 4 h(x, y) =

g(x) − g( y) 4 − 3x y (x − 2)2 = = ≥ 0. x−y (3x 2 + 4)(3 y 2 + 4) (12x 2 + 1)(12 y 2 + 1)

The proof is completed. From x + 3 y = 4 and h(x, y) = 0, we get x = 2 and, y = 2/3. In accord with Remark 3, the equality holds for a = b = c = d = 1, and also for a = 2 and b = c = d = 2/3 (or any cyclic permutation). Remark. Similarly, we can prove the following generalization. • If a1 , a2 , . . . , an are real numbers such that a1 + a2 + · · · + an = n, then a1 (a1 − 1) 4(n − 1)a12

+ n2

+

a2 (a2 − 1) 4(n − 1)a22

+ n2

+ ··· +

an (an − 1) ≥ 0, 4(n − 1)an2 + n2

n with equality for a1 = a2 = · · · = an = 1, and also for a1 = and a2 = a3 = · · · = an = 2 n (or any cyclic permutation). 2(n − 1)

P 3.6. Let a1 , a2 , · · · , an 6= −k be real numbers such that a1 + a2 + · · · + an = n, where n k≥ p . 2 n−1 Then,

an (an − 1) a1 (a1 − 1) a2 (a2 − 1) + + ··· + ≥ 0. 2 2 (a1 + k) (a2 + k) (an + k)2 (Vasile Cîrtoaje, 2008)

Solution. Write the inequality as f (a1 ) + f (a2 ) + · · · + f (an ) ≥ n f (s), where f (u) =

s=

a1 + a2 + · · · + an = 1, n

u(u − 1) , u ∈ I = R \ {−k}. (u + k)2

From f 0 (u) =

(2k + 1)u − k , (u + k)3

158

Vasile Cîrtoaje

it follows that f is increasing on (−∞, −k) ∪ [s0 , ∞) and decreasing on (−k, s0 ], where s0 =

k < 1. 2k + 1

Since lim f (u) = 1

u→−∞

and f (s0 ) < f (1) = 0, we have min f (u) = f (s0 ). u∈I

From

1 00 k(k + 2) − (2k + 1)u f (u) = , 2 (u + k)4 ˜ • k(k + 2) , hence on [s0 , 1]. According to LPCF Theoit follows that f is convex on 0, 2k + 1 rem, Remark 4 and Remark 1, it suffices to show that h(x, y) ≥ 0 for all x, y ∈ I which satisfy x + (n − 1) y = n, where h(x, y) =

g(x) − g( y) , x−y

Indeed, we have g(u) = and h(x, y) =

g(u) =

f (u) − f (1) . u−1

u (u + k)2

k2 − x y ≥ 0, (x + k)2 ( y + k)2

because

[2(n − 1) y − n]2 n2 −xy = ≥ 0. 4(n − 1) 4(n − 1) n The equality holds for a1 = a2 = · · · = an = 1. If k = p , then the equality holds 2 n−1 n n (or any cyclic permutation). also for a1 = and a2 = · · · = an = 2 2(n − 1) k2 − x y ≥

P 3.7. Let a1 , a2 , . . . , an 6= −k be real numbers such that a1 + a2 + · · · + an = n. If k ≥1+ p then

a12 − 1 (a1 + k)2

+

a22 − 1 (a2 + k)2

n n−1

+ ··· +

, an2 − 1 (an + k)2

≥ 0. (Vasile Cîrtoaje, 2008)

Partially Convex Function Method

159

Solution. Write the inequality as f (a1 ) + f (a2 ) + · · · + f (an ) ≥ n f (s), where f (u) =

s=

a1 + a2 + · · · + an = 1, n

u2 − 1 , u ∈ I = R \ {−k}. (u + k)2

From f 0 (u) =

2(ku + 1) , (u + k)3

it follows that f is increasing on (−∞, −k) ∪ [s0 , ∞) and decreasing on (−k, s0 ], where s0 =

−1 ∈ (−1, 0). k

Since lim f (u) = 1

u→−∞

and f (s0 ) < f (−1) = 0, we have min f (u) = f (s0 ). u∈I

From f 00 (u) =

2(k2 − 3 − 2ku) , (u + k)4

it follows that f is convex on [−1, 1], hence on [s0 , 1], since k2 − 3 − 2ku ≥ k2 − 3 − 2k = (k + 1)(k − 3) ≥ 0. According to LPCF Theorem, Remark 4 and Remark 1, it suffices to show that h(x, y) ≥ 0 for x, y ∈ I which satisfy x + (n − 1) y = n. We have g(u) = h(x, y) =

f (u) − f (1) u+1 = , u−1 (u + k)2

g(x) − g( y) (k − 1)2 − 1 − x − y − x y = ≥ 0, x−y (x + k)2 ( y + k)2

since [(n − 1) y − 1]2 n2 −1− x − y − xy = ≥ 0. n−1 n−1 n The equality holds for a1 = a2 = · · · = an = 1. If k = 1 + p , then the equality n−1 1 holds also for a1 = n − 1 and a2 = · · · = an = (or any cyclic permutation). n−1 (k − 1)2 − 1 − x − y − x y ≥

160

Vasile Cîrtoaje

P 3.8. Let a1 , a2 , . . . , an be nonnegative real numbers such that a1 + a2 + · · · + an ≥ n. If k > 1, then a1 a1k

+ a2 + · · · + an

+

a2 a1 + a2k

+ · · · + an

+ ··· +

an ≤ 1. a1 + a2 + · · · + ank (Vasile Cîrtoaje, 2006)

Solution. According to the proof of P 2.106 from Volume 2, it suffices to consider the case where a1 + a2 + · · · + an = n and k > n + 1. Write the desired inequality as f (a1 ) + f (a2 ) + · · · + f (an ) ≥ n f (s), where f (u) =

uk

−u , −u+n

We have

s=

a1 + a2 + · · · + an = 1, n

u ∈ I = [0, n].

f 0 (u) =

(k − 1)uk − n (uk − u + n)2

f 00 (u) =

f1 (u) , (uk − u + n)3

and

where f1 (u) = k(k − 1)uk−1 (uk − u + n) − 2(kuk−1 − 1)[(k − 1)uk − n]. From the expression of f 0 , it follows that f is decreasing on [0, s0 ] and increasing on [s0 , n], where  n 1/k s0 = < 1. k−1 On the other hand, for u ∈ [s0 , 1], we have (k − 1)uk − n ≥ (k − 1)s0k − n = 0, and hence f1 (u) ≥ k(k − 1)uk−1 (uk − u + n) − 2kuk−1 [(k − 1)uk − n] = kuk−1 [−(k − 1)(uk + u) + n(k + 1)] ≥ kuk−1 [−2(k − 1) + 2(k + 1)] = 4kuk−1 > 0. Thus, f 00 (u) > 0, and hence f is convex on [s0 , 1]. By LPCF Theorem, we need to show that f (x) + (n − 1) f ( y) ≥ n f (1) for all positive x, y which satisfy x ≥ 1 ≥ y ≥ 0 and x + (n − 1) y = n. Consider the nontrivial case where x > 1 > y ≥ 0 and write the inequality f (x) + (n − 1) f ( y) ≥ n f (1) as follows: xk

(n − 1) y x + k ≤ 1, −x −n y − y+n

Partially Convex Function Method

161

xk − x + n ≥ xk − x ≥

x( y k − y + n) , yk − ny + n

(n − 1) y( y − y k ) . yk − ny + n

Since y < 1, it suffices to show that xk − x ≥

(n − 1)( y − y k ) , yk − y + 1

which is equivalent to h(x) ≥

y − yk , (1 − y)( y k − y + 1)

where

xk − x . x −1 By the weighted AM-GM inequality, we have h(x) =

h0 (x) =

(k − 1)x k + 1 − k x k−1 > 0, (x − 1)2

and hence h is strictly increasing. Since x − 1 = (n − 1)(1 − y) ≥ 1 − y, we get h(x) ≥ h(2 − y) =

(2 − y)k − 2 + y . 1− y

Thus, it suffices to show that (2 − y)k − 2 + y ≥

y − yk . yk − y + 1

Putting 1 − y = t, 0 < t ≤ 1, we write this inequality as (2 − y)k − 1 + y ≥ (1 + t)k − t ≥

1 , yk − y + 1

1 , (1 − t)k + t

(1 − t 2 )k + t(1 + t)k ≥ 1 + t 2 + t(1 − t)k . By Bernoulli’s inequality, (1 − t 2 )k + t(1 + t)k > 1 − kt 2 + t(1 + kt) = 1 + t. So, we only need to show that t(1 − t) ≥ t(1 − t)k , which is clearly true. The proof is completed. The equality holds for a1 = a2 = · · · = an = 1.

162

Vasile Cîrtoaje

P 3.9. Let a1 , a2 , . . . , an be nonnegative real numbers such that a1 + a2 + · · · + an ≥ n. If k > 1, then 1 a1k

+ a2 + · · · + an

+

1 a1 + a2k

+ · · · + an

+ ··· +

1 ≤ 1. a1 + a2 + · · · + ank (Vasile Cîrtoaje, 2006)

Solution. Clearly, it suffices to consider the case a1 + a2 +· · ·+ an = n, when the desired inequality can be written as f (a1 ) + f (a2 ) + · · · + f (an ) ≥ n f (s), where f (u) =

uk

−1 , −u+n

From f 0 (u) =

s=

a1 + a2 + · · · + an = 1, n

u ∈ I = [0, n].

kuk−1 − 1 , (uk − u + n)2

it follows that f is decreasing on [0, s0 ] and increasing on [s0 , n], where 1

s0 = k 1−k < 1. We will show that f is convex on [s0 , 1]. For u ∈ [s0 , 1], we have f 00 (u) =

g(u) −k(k + 1)u2k−2 + k(k + 3)uk−1 + nk(k − 1)uk−2 − 2 > k , k 3 (u − u + n) (u − u + n)3

where g(u) = −k(k + 1)u2k−2 + k(k + 3)uk−1 − 2. Denoting t = kuk−1 , we have 1 ≤ t ≤ k and k g(u) = −(k + 1)t 2 + k(k + 3)t − 2k = (k + 1)(t − 1)(k − t) + (k − 1)(t + k) > 0. By LPCF Theorem, it suffices to show that xk

1 n−1 + k ≤1 −x +n y − y+n

for x ≥ 1 ≥ y ≥ 0 and x + (n − 1) y = n. Since this inequality is trivial for x = y = 1, assume next that x > 1 > y ≥ 0, and write the desired inequality as follows: xk − x + n ≥

yk − y + n , yk − y + 1

Partially Convex Function Method

163

xk − x ≥

(n − 1)( y − y k ) , yk − y + 1

y − yk xk − x ≥ . x −1 (1 − y)( y k − y + 1) Let h(x) =

xk − x , x > 1. By the weighted AM-GM inequality, we have x −1 h0 (x) =

(k − 1)x k + 1 − k x k−1 > 0. (x − 1)2

Therefore, h is increasing. Since x − 1 = (n − 1)(1 − y) ≥ 1 − y, hence x ≥ 2 − y > 1, we get (2 − y)k + y − 2 . h(x) ≥ h(2 − y) = 1− y Thus, it suffices to show that (2 − y)k + y − 2 ≥

y − yk , yk − y + 1

which is equivalent to (2 − y)k + y − 1 ≥

yk

1 . − y +1

Substituting t = 1 − y, 0 < t ≤ 1, the inequality becomes (1 + t)k − t ≥

1 , (1 − t)k + t

(1 − t 2 )k + t(1 + t)k ≥ 1 + t 2 + t(1 − t)k . By Bernoulli’s inequality, (1 − t 2 )k + t(1 + t)k ≥ 1 − kt 2 + t(1 + kt) = 1 + t. Thus, it suffices to show that t(1 − t) ≥ t(1 − t)k , which is clearly true. The proof is completed. The equality holds for a1 = a2 = · · · = an = 1. Remark. Using this result, we can get the following statement. • Let x 1 , x 2 , . . . , x n be nonnegative real numbers such that x 1 + x 2 + · · · + x n ≥ n. If k > 1, then x 1k − x 1 x 1k + x 2 + · · · + x n

+

x 2k − x 2 x 1 + x 2k + · · · + x n

+ ··· +

x nk − x n x 1 + x 2 + · · · + x nk

≥ 0.

164

Vasile Cîrtoaje

This inequality is equivalent to 1 x 1k

+ x2 + · · · + x n

+

1 x1 +

x 2k

+ · · · + xn

Using the substitutions s=

+ ··· +

1 n ≤ . k x1 + x2 + · · · + x n x1 + x2 + · · · + x n

x1 + x2 + · · · + x n , n

s ≥ 1,

and

xi , i = 1, 2, . . . , n, s which yields a1 + a2 + · · · + an = n, the desired inequality becomes ai =

1

X s k−1 a1k

+ a2 + · · · + a n

≤ 1.

Since s k−1 ≥ 1, it suffices to show that 1

X a1k

+ a2 + · · · + an

≤ 1,

which is just the inequality from P 3.9. Since x 1 x 2 · · · x n ≥ 1 involves x 1 + x 2 + · · · + x n ≥ n, the inequality is also true under the more restrictive condition x 1 x 2 · · · x n ≥ 1. For n = 3 and k = 5/2, we get the inequality from IMO-2005: • If x, y, z are nonnegative real numbers such that x yz ≥ 1, then y5 − y2 x5 − x2 z5 − z2 + + ≥ 0. x 5 + y 2 + z2 x 2 + y 5 + z2 x 2 + y 2 + z5

P 3.10. If a1 , a2 , . . . , an are nonnegative real numbers such that a1 + a2 + · · · + an = n, then an − 1 a1 − 1 a2 − 1 + + ··· + ≤ 0. 2 2 a1 + a2 + · · · + an2 a1 + a2 + · · · + an a1 + a2 + · · · + an (Vasile Cîrtoaje, 2006) Solution. For n = 2, the inequality is equivalent to (a1 − a2 )2 ≥ 0. Consider further that n ≥ 3 and write the inequality as f (a1 ) + f (a2 ) + · · · + f (an ) ≥ n f (s),

s=

a1 + a2 + · · · + an = 1, n

Partially Convex Function Method where f (u) =

u2

165

1−u , −u+n

u ∈ I = [0, n].

From f 0 (u) =

u2 − 2u − n + 1 , (u2 − u + n)2

it follows that f is decreasing on [0, s0 ] and increasing on [s0 , n], where s0 = 1 +

p

n, s0 ∈ (1, n).

We will show that f is convex on [1, s0 ]. For u ∈ [1, s0 ], we have f 00 (u) =

(u2

2g(u) , − u + n)3

g(u) = −u3 + 3u2 + 3(n − 1)u − 2n + 1.

Since g 0 (u) = −3(u2 − 2u − n + 1) ≥ 0, g is increasing on [1, s0 ], hence g(u) ≥ g(1) = n > 0 and f 00 (u) > 0. By RPCF Theorem, it suffices to show that (n − 1)(1 − y) 1− x + ≥0 2 x −x +n y2 − y + n for 0 ≤ x ≤ 1 ≤ y and x + (n − 1) y = n. Since (n − 1)(1 − y) = x − 1, we have  ‹ (n − 1)(1 − y) 1− x 1 1 + = (x − 1) − 2 + x2 − x + n y2 − y + n x − x + n y2 − y + n (x − 1)(x − y)(x + y − 1) = (x 2 − x + n)( y 2 − y + n) n(x − 1)2 (x + y − 1) = ≥ 0. (n − 1)(x 2 − x + n)( y 2 − y + n) The proof is completed. The equality holds for a1 = a2 = · · · = an = 1.

P 3.11. Let a1 , a2 , · · · , an be positive real numbers such that a1 + a2 + · · · + an = n. If n 0 0. Thus, it suffices to prove the desired inequality for k=

n , n−1

1 < k ≤ 2.

Rewrite the desired inequality as f (a1 ) + f (a2 ) + · · · + f (an ) ≥ n f (s),

s=

a1 + a2 + · · · + an = 1, n

where f (u) = −uk/u , We have

u ∈ I = (0, n). k

f 0 (u) = ku u −2 (ln u − 1), k

f 00 (u) = ku u −4 [u + (1 − ln u)(2u − k + k ln u)]. For n = 2, when k = 2 and I = (0, 2), f is convex for u ∈ I, u ≥ 1, because 1 − ln u > 0,

2u − k + k ln u = 2u − 2 + 2 ln u ≥ 2u − 2 ≥ 0.

Therefore, we may apply HCF Theorem. Consider now that n ≥ 3. From the expression of f 0 , it follows that f is decreasing on (0, s0 ] and increasing on [s0 , n), where s0 = e. In addition, we claim that f is convex on [1, s0 ]. Indeed, since 1 − ln u ≥ 0,

2u − k + k ln u ≥ 2 − k > 0,

we have f 00 > 0 for u ∈ [1, s0 ]. Therefore, by HCF Theorem (for n = 2) and RPCF Theorem (for n ≥ 3), we only need to show that x k/x + (n − 1) y k/ y ≤ n for 0 < x ≤ 1 ≤ y and x + (n − 1) y = n. We have k/x ≥ k > 1. Also, from k n n = > = 1, y (n − 1) y x + (n − 1) y we get 0
0. u4

168

Vasile Cîrtoaje

According to LPCF Theorem and Remark 4, we only need to show that that f (x) + 4 f ( y) ≥ 5 f (1) for all nonzero real x, y such that x + 4 y = 5. Using Remark 1, it suffices to prove that h(x, y) ≥ 0, where h(x, y) =

g(x) − g( y) , x−y

From g(u) = 5 h(x, y) =



g(u) =

f (u) − f (1) . u−1

‹ 9 5 − 2 , u u

5(5x + 5 y − 9x y) ≥0 x2 y2

and 5x + 5 y − 9x y = (6 y − 5)2 ≥ 0, it follows that h(x, y) ≥ 0. The proof is completed. In accordance with Remark 3, the equality holds for a = b = c = d = e = 1, and also for a = 5/3 and b = c = d = e = 5/6 (or any cyclic permutation). Remark. Similarly, we can prove the following generalization. • Let a1 , a2 , . . . , an be nonzero real numbers such that a1 + a2 + · · · + an = n. If n , then k= p n+ n−1 ‹  ‹  ‹  k 2 k 2 k 2 + 1− + ··· + 1 − ≥ n(1 − k)2 , 1− a1 a2 an with equality for a1 = a2 = · · · = an = 1, and also for a1 = · · · = an =

n (or any cyclic permutation). p n−1+ n−1

n and a2 = a3 = p 1+ n−1

P 3.13. If a, b, c are nonnegative real numbers such that a + b + c = 3, then (1 − a + a4 )(1 − b + b4 )(1 − c + c 4 ) ≥ 1. Solution. Write the inequality as f (a) + f (b) + f (c) ≥ 3 f (s),

s=

a+b+c = 1, 3

where f (u) = ln(1 − u + u4 ),

u ∈ [0, 3].

Partially Convex Function Method

169

From f 0 (u) =

4u3 − 1 , 1 − u + u4

p 3 it follows that f is decreasing on [0, s0 1] and increasing on [s0 , 4], where s0 = 1/ 4. Also, f is convex for u ∈ [s0 , 1] because f 00 (u) =

−4u6 − 4u3 + 12u2 − 1 −4u2 − 4u2 + 12u2 − 1 4u2 − 1 ≥ = > 0. (1 − u + u4 )2 (1 − u + u4 )2 (1 − u + u4 )2

According to LPCF Theorem, we only need to show that f (x) + 2 f ( y) ≥ 3 f (1) for all x, y ≥ 0 such that x + 2 y = 3. Using Remark 2, it suffices to prove that H(x, y) ≥ 0, where f 0 (x) − f 0 ( y) . H(x, y) = x−y We have (x + y)(x − y)2 − 1 + 4(x 2 + y 2 + x y) − 2x y(x + y) − 4x 3 y 3 (1 − x + x 4 )(1 − y + y 4 ) −1 + 4(x 2 + y 2 + x y) − 2x y(x + y) − 4x 3 y 3 ≥ (1 − x + x 4 )(1 − y + y 4 ) −1 + 4(x + y)2 − 2x y(x + y) − 4x y − 4x 3 y 3 = . (1 − x + x 4 )(1 − y + y 4 )

H(x, y) =

Thus, we only need to show that −1 + 4(x + y)2 − 2x y(x + y) − 4x y − 4x 3 y 3 ≥ 0. p From 3 = x + 2 y ≥ 2 2x y and (1 − x)(1 − y) ≤ 0, we get xy ≤

9 , 8

x + y ≥ 1 + x y.

Therefore, − 1 + 4(x + y)2 − 2x y(x + y) − 4x y − 4x 3 y 3 ≥ ≥ − 1 + 2(x + y)[2(x + y) − x y] − 4x y − 4x 3 y 3 ≥ − 1 + 2(1 + x y)[2(1 + x y) − x y] − 4x y − 4x 3 y 3 =3 + 2x y + 2x 2 y 2 − 4x 3 y 3 ≥ 3 + 2x y + 2x 2 y 2 − 5x 2 y 2 =3 + 2x y − 3x 2 y 2 ≥ 3 + 2x y − 4x y = 3 − 2x y > 0. The proof is completed. In accordance with Remark 3, the equality holds for a = b = c = 1.

170

Vasile Cîrtoaje

P 3.14. If a, b, c, d are nonnegative real numbers such that a + b + c + d = 4, then (1 − a + a3 )(1 − b + b3 )(1 − c + c 3 )(1 − d + d 3 ) ≥ 1. (Vasile Cîrtoaje, 2012) Solution. Write the inequality as f (a) + f (b) + f (c) + f (d) ≥ 4 f (s),

s=

a+b+c+d = 1, 4

where f (u) = ln(1 − u + u3 ), From f 0 (u) =

u ∈ [0, 4].

3u2 − 1 , 1 − u + u3

p it follows that f is decreasing on [0, s0 ] and increasing on [s0 , 4], where s0 = 1/ 34. Also, f is convex for u ∈ [s0 , 1] because f 00 (u) =

−3u4 + 6u − 1 −3u + 6u − 1 3u − 1 ≥ = > 0. 3 2 3 2 (1 − u + u ) (1 − u + u ) (1 − u + u3 )2

According to LPCF Theorem, we only need to show that f (x) + 3 f ( y) ≥ 4 f (1) for all x, y ≥ 0 such that x + 3 y = 4. Using Remark 2, it suffices to prove that H(x, y) ≥ 0, where f 0 (x) − f 0 ( y) . H(x, y) = x−y We have H(x, y) =

(x − y)2 + 3(x + y) − 1 − 3x 2 y 2 3(x + y) − 1 − 3x 2 y 2 ≥ . (1 − x + x 3 )(1 − y + y 3 ) (1 − x + x 3 )(1 − y + y 3 )

Thus, we only need to show that 3(x + y) − 1 − 3x 2 y 2 ≥ 0. From 4 = x + 3 y ≥ 2

p

3x y and (1 − x)(1 − y) ≤ 0, we get xy ≤

4 , 3

x + y ≥ 1 + x y.

Therefore, 3(x + y) − 1 − 3x 2 y 2 ≥ 3(1 + x y) − 1 − 3x 2 y 2 ≥ 3(1 + x y) − 1 − 4x y = 2 − x y > 0. The proof is completed. The equality holds for a = b = c = d = 1.

Partially Convex Function Method

171

P 3.15. If a1 , a2 , . . . , a10 are real numbers such that a1 + a2 + · · · + a10 = 10, then 2 (1 − a1 + a12 )(1 − a2 + a22 ) · · · (1 − a10 + a10 ) ≥ 1.

(Vasile Cîrtoaje, 2006) Solution. Write the inequality as f (a1 ) + f (a2 ) + · · · + f (a10 ) ≥ 10 f (s),

s=

a1 + a2 + · · · + a10 =1 10

where f (u) = ln(1 − u + u2 ), From f 0 (u) =

u ∈ R.

2u − 1 , 1 − u + u2

it follows that f is decreasing on (−∞, s0 ] and increasing on [s0 , ∞), where s0 = 1/2. Also, from 1 + 2u(1 − u) f 00 (u) = , (1 − u + u2 )2 it follows that f 00 (u) > 0 for u ∈ [s0 , 1], hence f is convex on [s0 , 1]. According to LPCF Theorem, we only need to show that f (x) + 9 f ( y) ≥ 10 f (1) for all real x, y such that x + 9 y = 10. Using Remark 2, it suffices to prove that H(x, y) ≥ 0, where H(x, y) = Since H(x, y) =

f 0 (x) − f 0 ( y) . x−y

1 + x + y − 2x y (1 − x + x 2 )(1 − y + y 2 )

and 1 + x + y − 2x y = 18 y 2 − 8 y + 1 = 2 y 2 + (4 y − 1)2 > 0, the conclusion follows. The equality holds for a1 = a2 = · · · = a10 = 1. Remark. By replacing a1 , a2 , . . . , a10 respectively with 1 − a1 , 1 − a2 , . . . , 1 − a10 , we get the following statement. • If a1 , a2 , . . . , a10 are real numbers such that a1 + a2 + · · · + a10 = 0, then 2 (1 − a1 + a12 )(1 − a2 + a22 ) · · · (1 − a10 + a10 ) ≥ 1,

with equality for a1 = a2 = · · · = an = 0.

172

Vasile Cîrtoaje

P 3.16. If a, b, c, d, e are nonzero real numbers such that a + b + c + d + e = 5, then ‹  ‹  1 1 1 1 1 1 1 1 1 1 + + + + . 5 2 + 2 + 2 + 2 + 2 + 45 ≥ 14 a b c d e a b c d e (Vasile Cîrtoaje, 2013) Solution. Write the desired inequality as f (a) + f (b) + f (c) + f (d) + f (e) ≥ 5 f (s), s = where f (u) =

a+b+c+d+e = 1, 5

5 14 − + 9, u ∈ I = R \ {0}. 2 u u

From

2(7u − 5) , u3 it follows that f is increasing on (−∞, 0) ∪ [s0 , ∞) and decreasing on (0, s0 ], where f 0 (u) =

s0 =

5 . 7

Since lim f (u) = 9

u→−∞

and f (s0 ) < f (1) = 0, we have min f (u) = f (s0 ). u∈I

From

2(15 − 14u) , u4 it follows that f is convex on [s0 , 1]. By LPCF Theorem, Remark 4 and Remark 1, it suffices to show that h(x, y) ≥ 0 for all x, y ∈ I which satisfy x + 4 y = 5, where f 00 (u) =

h(x, y) =

g(x) − g( y) , x−y

Indeed, we have g(u) = h(x, y) =

g(u) =

f (u) − f (1) . u−1

9 5 − 2, u u

(6 y − 5)2 5x + 5 y − 9x y = ≥ 0. x2 y2 x2 y2

In accordance with Remark 3, the equality holds for a = b = c = d = e = 1, and also 5 5 for a = and b = c = d = e = (or any cyclic permutation). 3 6

Partially Convex Function Method

173

P 3.17. If a, b, c are positive real numbers such that a bc = 1, then 7 − 6a 7 − 6b 7 − 6c + + ≥ 1. 2 + a2 2 + b2 2 + c 2 (Vasile Cîrtoaje, 2008) Solution. Using the substitution a = e x , b = e y , c = ez , we need to show that f (x) + f ( y) + f (z) ≥ 3 f (s), where f (u) = From f 0 (u) =

s=

x + y +z = 0, 3

7 − 6eu , u ∈ R. 2 + e2u

2(3eu + 2)(eu − 3) , (2 + e2u )2

it follows that f is decreasing on (−∞, s0 ] and increasing on [s0 , ∞), where s0 = ln 3. Also, we have 2t · h(t) f 00 (u) = , t = eu , (2 + t 2 )3 where h(t) = −3t 4 + 14t 3 + 36t 2 − 28t − 12. We will show that h(t) > 0 for t ∈ [1, 3], hence f is convex on [0, s0 ]. We have h(t) = 3(t 2 − 1)(9 − t 2 ) + 14t 3 + 6t 2 − 28t + 15 = 3(t 2 − 1)(9 − t 2 ) + 14t 2 (t − 1) + 14(t − 1)2 + 6t 2 + 1 > 0. By RPCF Theorem, we only need to prove that f (x) + 2 f ( y) ≥ 3 f (0) for all real x, y such that x + 2 y = 0. That is, to show that the original inequality holds for b = c := t and a = 1/t 2 , where t > 0. Write this inequality as t 2 (7t 2 − 6) 2(7 − 6t) + ≥ 1, 2t 4 + 1 2 + t2 (t − 1)2 (t − 2)2 (5t 2 + 6t + 3) ≥ 0. Since the last inequality is true, the proof is completed. In accordance with Remark 3, the equality holds for a = b = c = 1, and also for a = 1/4 and b = c = 2 (or any cyclic permutation).

174

Vasile Cîrtoaje

P 3.18. If a, b, c are positive real numbers such that a bc = 1, then 1 1 1 1 + + ≤ . a + 5bc b + 5ca c + 5a b 2 (Vasile Cîrtoaje, 2008) Solution. Write the inequality as a2

a b c 1 + 2 + 2 ≤ . +5 b +5 c +5 2

Using the substitution a = e x , b = e y , c = ez , we need to show that f (x) + g( y) + g(z) ≥ 3 f (s), where f (u) =

s=

x + y +z = 0, 3

−eu , u ∈ R. e2u + 5

From f 0 (u) =

eu (e2u − 5) , (e2u + 5)2

it follows that f is decreasing on (−∞, s0 ] and increasing on [s0 , ∞), where s0 = Also, from f 00 (u) =

eu (−e4u + 30e2u − 25) , (e2u + 5)3

it follows that f is convex on [0, s0 ], because 0 ≤ u ≤ s0 involves 1 ≤ eu ≤

1 ln 5. 2

p 5, hence

−e4u + 30e2u − 25 ≥ e2u (5 − e2u ) + 25(e2u − 1) > 0. By RPCF Theorem, we only need to prove the original inequality for b = c := t and a = 1/t 2 , where t > 0. Write this inequality as t2 2t 1 + ≤ , 5t 4 + 1 t 2 + 5 2 (t − 1)2 (5t 4 − 10t 3 − 2t 2 + 6t + 5) ≥ 0, (t − 1)2 [5(t − 1)4 + 2t(5t 2 − 16t + 13)] ≥ 0. Since the last inequality is true, the proof is completed. The equality holds for a = b = c = 1.

Partially Convex Function Method

175

P 3.19. If a, b, c are positive real numbers such that a bc = 1, then 1 1 3 1 + + ≤ . 4 − 3a + 4a2 4 − 3b + 4b2 4 − 3c + 4c 2 5 (Vasile Cirtoaje, 2008) Solution. Let a = ex ,

b = ey,

c = ez .

We need to show that f (x) + f ( y) + f (z) ≥ 3 f (s), where f (u) =

s=

x + y +z = 0, 3

−1 , u ∈ R. 4 − 3eu + 4e2u

From f 0 (u) =

eu (8eu − 3) , (4 − 3eu + 4e2u )2

it follows that f is decreasing on (−∞, s0 ] and increasing on [s0 , ∞), where s0 = ln

3 < 0. 8

We claim that f is convex on [s0 , 0]. Since f 00 (u) =

eu (−64e3u + 36e2u + 55eu − 12) , (4 − 3eu + 4e2u )3

we need to show that −64x 3 + 36x 2 + 55x − 12 ≥ 0, where x = eu ∈ [3/8, 1]. Indeed, −64x 3 + 36x 2 + 55x − 12 > −72x 3 + 36x 2 + 48x − 12 = 12(1 − x)(6x 2 + 3x − 1) ≥ 0. By LPCF Theorem, we only need to prove the original inequality for b = c := t and a = 1/t 2 , where t > 0. Write this inequality as follows: t4 2 3 + ≤ , 4 2 2 4t − 3t + 4 4 − 3t + 4t 5 28t 6 − 21t 5 − 48t 4 + 27t 3 + 42t 2 − 36t + 8 ≥ 0, (t − 1)2 (28t 4 + 35t 3 − 6t 2 − 20t + 8) ≥ 0. It suffices to show that 7(4t 4 + 5t 3 − t 2 − 3t + 1) ≥ 0.

176

Vasile Cîrtoaje

Indeed, 4t 4 + 5t 3 − t 2 − 3t + 1 = t 2 (2t − 1)2 + 9t 3 − 2t 2 − 3t + 1 and 9t 3 − 2t 2 − 3t + 1 = t(3t − 1)2 + (2t − 1)2 > 0. The equality holds for a = b = c = 1. Remark. Since 1 1 1 ≥ = , 4 − 3a + 4a2 4 − 3a + 4a2 + (1 − a)2 5(1 − a + a2 ) we get the following known inequality 1 1 1 + + ≤ 3. 2 2 1−a+a 1− b+ b 1 − c + c2

P 3.20. If a, b, c are positive real numbers such that a bc = 1, then 1 1 1 3 + + ≤ . (3a + 1)(3a2 − 5a + 3) (3b + 1)(3b2 − 5b + 3) (3c + 1)(3c 2 − 5c + 3) 4 Solution. Let a = ex ,

b = ey,

c = ez .

We need to show that f (x) + f ( y) + f (z) ≥ 3 f (s), where f (u) =

(3eu

From f 0 (u) =

s=

x + y +z = 0, 3

−1 , u ∈ R. + 1)(3e2u − 5eu + 3)

(3eu − 2)(9eu − 2) , (3eu + 1)2 (3e2u − 5eu + 3)2

it follows that f is increasing on (−∞, s1 ] ∪ [s0 , ∞) and decreasing on [s1 , s0 ], where s1 = ln 2 − ln 9, s0 = ln 2 − ln 3,

s1 < s0 < 0.

Since lim f (u) = f (s0 ) = −1/3,

u→−∞

Partially Convex Function Method

177

we get min f (u) = f (s0 ). u∈R

We claim that f is convex on [s0 , 0]. We have f 00 (u) =

(3t

t · h(t) , − 5t + 3)3

+ 1)3 (3t 2

where t = eu , h(t) = −729t 5 + 1188t 4 − 648t 3 + 387t 2 − 160t + 12. Since the polynomial h(t) has the real roots t 1 ≈ 0.0933, t 2 ≈ 0.5072, t 3 ≈ 1.11008, it follows that h(t) > 0 for t ∈ [2/3, 1], hence f is convex for u ∈ [s0 , 0]. By LPCF Theorem, we only need to prove the original inequality for b = c := t and a = 1/t 2 , where t ∈ (0, 1]. Write this inequality as follows: 2 3 t6 + ≤ . 2 4 2 2 (t + 3)(3t − 5t + 3) (3t + 1)(3t − 5t + 3) 4 Since t 2 + 3 ≥ 2(t + 1) and 3t 4 − 5t 2 + 3 ≥ t(3t 2 − 5t + 3), it suffices to prove that t5 2 3 + ≤ . 2(t + 1)(3t 2 − 5t + 3) (3t + 1)(3t 2 − 5t + 3) 4 This is equivalent to the obvious inequality (1 − t)2 (1 + 15t + 5t 2 − 14t 3 − 6t 4 ) ≥ 0. The equality holds for a = b = c = 1.

P 3.21. Let a1 , a2 , · · · , an (n ≥ 3) be positive real numbers such that a1 a2 · · · an = 1. If p, q ≥ 0 such that p + 4q ≥ n − 1, then 1 − a1 1+

pa1 + qa12

+

1 − a2 1+

pa2 + qa22

+ ··· +

1 − an ≥ 0. 1 + pan + qan2 (Vasile Cîrtoaje, 2008)

178

Vasile Cîrtoaje

Solution. For q = 0, we get a known inequality (see Remark 2 from the proof of P 1.51). Consider further that q > 0. Using the substitutions ai = e x i for i = 1, 2, . . . , n, we need to show that f (x 1 ) + f (x 2 ) + · · · + f (x n ) ≥ n f (s), where

s=

x1 + x2 + · · · + x n = 0, n

f (u) =

1 − eu , u ∈ R. 1 + peu + qe2u

f 0 (t) =

eu (qe2u − 2qeu − p − 1) , (1 + peu + qe2u )2

From

it follows that f is decreasing on (−∞, s0 ] and increasing on [s0 , ∞), where v t p+1 . s0 = ln r0 > 0, r0 = 1 + 1 + q Also, we have f 00 (u) =

t · h(t) , (1 + pt + qt 2 )3

where h(t) = −q2 t 4 + q(p + 4q)t 3 + 3q(p + 2)t 2 + (p − 4q + p2 )t − p − 1,

t = eu .

We will show that h(t) ≥ 0 for t ∈ [1, r0 ], hence f is convex on [0, s0 ]. We have h0 (t) = −4q2 t 3 + 3q(p + 4q)t 2 + 6q(p + 2)t + p − 4q + p2 , h00 (t) = 6q[−2qt 2 + (p + 4q)t + p + 2]. Since h00 (t) = 6q[2(−qt 2 + 2qt + p + 1) + p(t − 1)] ≥ 12q(−qt 2 + 2qt + p + 1) ≥ 0, h0 (t) is increasing, h0 (t) ≥ h0 (1) = p2 + 9pq + 8q2 + p + 8q > 0, h is increasing, hence h(t) ≥ h(1) = p2 + 4pq + 3q2 + 2q − 1 = (p + 2q)2 − (q − 1)2 = (p + q + 1)(p + 3q − 1). Therefore, f is convex on [0, s0 ] if p + 3q − 1 ≥ 0. Indeed, p + 3q − 1 ≥ p + 3q −

p + 2q p + 4q = > 0. n−1 2

Partially Convex Function Method

179

By RPCF Theorem, we only need to prove the original inequality for a2 = · · · = an := t and a1 = 1/t n−1 , where t ≥ 1. Write this inequality as (n − 1)(1 − t) t n−1 (t n−1 − 1) + ≥ 0, t 2n−2 + pt n−1 + q 1 + pt + qt 2 or pA + qB ≥ C, where A = t n−1 (t n − nt + n − 1), B = t 2n − t n+1 − (n − 1)(t − 1), C = t n−1 [(n − 1)t n − nt n−1 + 1]. Since p+4q ≥ n−1 and C ≥ 0 (by the AM-GM inequality applied to n positive numbers), it suffices to show that (p + 4q)C pA + qB ≥ , n−1 which is equivalent to p[(n − 1)A − C] + q[(n − 1)B − 4C] ≥ 0. Clearly, this is true if (n − 1)A − C ≥ 0 and (n − 1)B − 4C ≥ 0 for t ≥ 1. By the AM-GM inequality, we have (n − 1)A − C = nt n−1 [t n−1 − (n − 1)t + n − 2] ≥ 0. For n = 3 and n = 4.... Consider further that n ≥ 5. Since t − 1 ≤ t n−1 (t − 1), we have B ≥ t 2n − t n+1 − (n − 1)t n−1 (t − 1) = t n−1 [t n+1 − t 2 − (n − 1)t + n − 1]   t2 + 1 n−1 n+1 2 ≥t t − t − (n − 1) +n−1 2 1 = t n−1 [2t n+1 − (n + 1)t 2 + n − 1] 2 hence 2(n − 1)B − 8C ≥ (n − 1)t n−1 [2t n+1 − (n + 1)t 2 + n − 1] − 8C = (n − 1)t n−1 h(t),

180

Vasile Cîrtoaje

where h(t) = 2t n+1 − 8t n +

8nt n−1 − 8 − (n + 1)t 2 + n − 1. n−1

We have h0 (t) = 2th1 (t),

h1 (t) = (n + 1)t n−1 − 4nt n−2 + 4nt n−3 − n − 1,

h01 (t) = t n−4 g(t),

g(t) = (n2 − 1)t 2 + 4n(n − 3) − 4n(n − 2)t,

where Æ

(n2 − 1)t 2 · 4n(n − 3) − 4n(n − 2)t p — p ”Æ =4 n (n2 − 1)(n − 3) − (n − 2) n t.

g(t) ≥ 2

Since (n2 − 1)(n − 3) − n(n − 2)2 = n(n − 5) + 3 > 0, we have g(t) > 0, h01 (t) > 0, h1 (t) is increasing for t ≥ 1, h1 (t) ≥ h1 (1) = 0, h0 (t) ≥ 0, h(t) is increasing for t ≥ 1, h(t) ≥ h(1) = 0, hence (n − 1)B − 4C ≥ 0. Thus, the proof is completed. In accordance with Remark 3, the equality holds for a1 = a2 = · · · = an = 1. Remark 1. For p = 0 and q = 1, we get the following inequality (Vasile Cîrtoaje, 2006): 1−a 1− b 1−c 1−d 1−e + + + + ≥ 0, 2 2 2 2 1+a 1+ b 1+c 1+d 1 + e2 where a, b, c, d, e are positive real numbers such that a bcd e = 1. Replacing a, b, c, d, e by 1/a, 1/b, 1/c, 1/d, 1/e, we get the inequality 1+a 1+ b 1+c 1+d 1+e + + + + ≤ 5, 1 + a2 1 + b2 1 + c 2 1 + d 2 1 + e2 where a, b, c, d, e are positive real numbers such that a bcd e = 1. Notice that the inequality 1 − a1 1 + a12

+

1 − a2 1 + a22

+

1 − a3 1 + a32

+

1 − a4 1 + a42

+

1 − a5 1 + a52

+

1 − a6 1 + a62

≥0

is not true for all positive numbers a1 , a2 , a3 , a4 , a5 , a6 satisfying a1 a2 a3 a4 a5 a6 = 1. Indeed, for a2 = a3 = a4 = a5 = a6 = 2, the inequality becomes 1 − a1 1 + a12 which is false for a1 > 0.

− 1 ≥ 0,

Partially Convex Function Method

181

P 3.22. If a, b, c are positive real numbers such that a bc = 1, then 1−a 1− b 1−c + + ≥ 0. 2 2 17 + 4a + 6a 17 + 4b + 6b 17 + 4c + 6c 2 (Vasile Cîrtoaje, 2008) Solution. Using the substitution a = e x , b = e y , c = ez , we need to show that f (x) + g( y) + g(z) ≥ 3 f (s), where f (u) =

s=

x + y +z = 0, 3

1 − eu , u ∈ R, 1 + peu + qe2u

with p=

4 , 17

q=

6 . 17

As we have shown in the proof of the preceding P 3.21, f is decreasing on (−∞, s0 ] and increasing on [s0 , ∞), where s0 = ln r0 > 0,

r0 = 1 +

v t

v t9 p+1 1+ =1+ . q 2

In addition, f is convex on [0, s0 ] if p + 3q − 1 ≥ 0. Indeed, p + 3q − 1 =

5 > 0. 17

By RPCF Theorem, we only need to prove the original inequality for b = c := t and a = 1/t 2 , where t > 0. Write this inequality as follows: t 2 (t 2 − 1) 2(1 − t) + ≥ 0, 4 2 t + pt + q 1 + pt + qt 2 pA + qB ≥ C, where A = t 2 (t − 1)2 (t + 2), B = (t − 1)2 (t 4 + 2t 3 + 2t 2 + 2t + 2), C = t 2 (t − 1)2 (2t + 1).

182

Vasile Cîrtoaje

Indeed, we have pA + qB − C =

3(t − 1)2 (t − 2)2 (2t 2 + 2t + 1) ≥ 0. 17

In accordance with Remark 3, the equality holds for a = b = c = 1, and also for a = 1/4 and b = c = 2 (or any cyclic permutation).

P 3.23. If a1 , a2 , · · · , a8 are positive real numbers such that a1 a2 · · · a8 = 1, then 1 − a8 1 − a1 1 − a2 + + · · · + ≥ 0. (1 + a1 )2 (1 + a2 )2 (1 + a8 )2 (Vasile Cîrtoaje, 2006) Solution. Using the substitutions ai = e x i for i = 1, 2, . . . , 8, we need to show that f (x 1 ) + f (x 2 ) + · · · + f (x 8 ) ≥ 8 f (s), where f (u) =

s=

x1 + x2 + · · · + x8 = 0, 8

1 − eu , u ∈ R. (1 + eu )2

From f 0 (t) =

eu (eu − 3) , (1 + eu )3

it follows that f is decreasing on (−∞, s0 ] and increasing on [s0 , ∞), where s0 = ln 3 > 1. Also, we have f 00 (u) =

eu (8eu − e2u − 3) . (1 + eu )4

We will show that 8eu − e2u − 3 > 0 for u ∈ [0, s0 ], hence f is convex on [0, s0 ]. Indeed, 8eu − e2u − 3 ≥ 8eu − 3eu − 3 = 5eu − 3 > 5 − 3 > 0. By RPCF Theorem, we only need to prove the original inequality for a2 = · · · = a8 := t and a = 1/t 7 , where t ≥ 1. For the nontrivial case t > 1, write this inequality as follows: t 7 (t 7 − 1) 7(t − 1) ≥ . (t 7 + 1)2 (t + 1)2

Partially Convex Function Method

183

t 7 (t 7 − 1)(t + 1)2 ≥ 7, (t − 1)(t 7 + 1)2 t 7 (t 6 + t 5 + t 4 + t 3 + t 2 + t + 1) ≥ 7. (t 6 − t 5 + t 4 − t 3 + t 2 − t + 1)2 Since t 6 − t 5 + t 4 − t 3 + t 2 − t + 1 = t 4 (t 2 − t + 1) − (t − 1)(t 2 + 1) > t 4 (t 2 − t + 1), it suffices to show that t6 + t5 + t4 + t3 + t2 + t + 1 ≥ 7, t(t 2 − t + 1)2 which is equivalent to the obvious inequality (t − 1)6 ≥ 0. Thus, the proof is completed. The equality holds for a1 = a2 = · · · = a8 = 1. Remark. The inequality 1 − a9 1 − a1 1 − a2 + + · · · + ≥0 (1 + a1 )2 (1 + a2 )2 (1 + a9 )2 is not true for all positive numbers a1 , a2 , · · · , a9 satisfying a1 a2 · · · a9 = 1. Indeed, for a2 = a3 = · · · = a9 = 3, the inequality becomes 1 − a1 − 1 ≥ 0, (1 + a1 )2 which is false for a1 > 0.

˜ −13 13 P 3.24. Let a, b, c be positive real numbers such that a bc = 1. If k ∈ p , p , then 3 3 3 3 •

a+k b+k c+k 3(1 + k) + 2 + 2 ≤ . 2 a +1 b +1 c +1 2 (Vasile Cîrtoaje, 2012) Solution. The desired inequality is equivalent to X (a − 1)2 a2 + 1

≥k

X

‹ 2 − 3 . a2 + 1

184

Vasile Cîrtoaje

13 Thus, it suffices to prove this inequality for only |k| = p . On the other hand, replacing 3 3 a, b, c by 1/a, 1/b, 1/c, the inequality becomes X (a − 1)2 a2 + 1

 X ≥ k 3−

‹ 2 . a2 + 1

Thus, we only need to prove the desired inequality for 13 k= p . 3 3 Using the substitution a = e x , b = e y , c = ez , we need to show that f (x) + g( y) + g(z) ≥ 3 f (s), where f (u) =

s=

x + y +z = 0, 3

−eu − k , u ∈ R. e2u + 1

From f 0 (t) =

e2u + 2keu − 1 , (e2u + 1)2

it follows that f is decreasing on (−∞, s0 ] and increasing on [s0 , ∞), where s0 = ln r0 < 0,

1 r0 = p . 3 3

Also, we have f 00 (u) =

t · h(t) , (1 + t 2 )3

where h(t) = −t 4 − 4kt 3 + 6t 2 + 4kt − 1,

t = eu .

We will show that h(t) > 0 for t ∈ [r0 , 1], hence f is convex on [s0 , 0]. Indeed, since 52t 52 4kt = p ≥ > 1, 3 3 27 we have h(t) = −t 4 + 6t 2 − 1 + 4kt(1 − t 2 ) ≥ −t 4 + 6t 2 − 1 + (1 − t 2 ) = t 2 (5 − t 2 ) > 0.

Partially Convex Function Method

185

By LPCF Theorem, we only need to prove the original inequality for b = c := t and a = 1/t 2 , where t > 0. Write this inequality as t 2 (kt 2 + 1) 2(t + k) 3(1 + k) + 2 ≤ , t4 + 1 t +1 2 3t 6 − 4t 5 + t 4 + t 2 − 4t + 3 − k(1 − t 2 )3 ≥ 0, (t − 1)2 [(3 + k)t 4 + 2(1 + k)t 3 + 2t 2 + 2(1 − k)t + 3 − k] ≥ 0 p p p p (t − 1)2 (t − 2 + 3)2 [(27 + 13 3)t 2 + 24(2 + 3)t + 33 + 17 3] ≥ 0. The last inequality is clearly true. Thus, the proof is completed. The equality holds p 13 for a = b = c = 1. If k = p , then the equality holds also for a = 7 + 4 3 and 3 3 p −13 b = c = 2 − 3 (or any cyclic permutation). If k = p , then the equality holds also 3 3 p p for a = 7 − 4 3 and b = c = 2 + 3 (or any cyclic permutation).

p P 3.25. If a, b, c are positive real numbers and 0 < k ≤ 2 + 2 2, then b3 c3 a+b+c a3 + + ≥ . ka2 + bc k b2 + ca kc 2 + a b k+1 (Vasile Cîrtoaje, 2011) p p Solution. For k < 2 + 2 2, the proof is similar to the one of the main case k = 2 + 2 2. For this reason, we consider further only the case where p k = 2 + 2 2. Due to homogeneity, we may assume that a bc = 1. On this hypothesis, X

 X  a4 a3 1 X a 1 X a4 − a − a = − = . ka2 + bc k + 1 ka3 + 1 k + 1 k+1 ka3 + 1

Thus, we can write the inequality as a4 − a b4 − b c4 − c + + ≥ 0. ka3 + 1 k b3 + 1 kc 3 + 1 Using the substitution a = e x , b = e y , c = ez ,

186

Vasile Cîrtoaje

we need to show that f (x) + g( y) + g(z) ≥ 3 f (s), where f (u) = From f 0 (t) =

s=

x + y +z = 0, 3

e4u − eu , u ∈ R. ke3u + 1

ke6u + 2(k + 2)e3u − 1 , (ke3u + 1)2

it follows that f is decreasing on (−∞, s0 ] and increasing on [s0 , ∞), where s0 = ln r0 < 0,

r0 =

v p u t 3 −k − 2 + (k + 1)(k + 4) k

≈ 0.4149.

Also, we have f 00 (u) =

t · h(t) , (kt 3 + 1)3

where h(t) = k2 t 9 − k(4k + 1)t 6 + (13k + 16)t 3 − 1,

t = eu .

We will show that h(t) > 0 for t ∈ [r0 , 1], hence f is convex on [s0 , 0]. Indeed, we have h(t) ≥ 0 for t ∈ [t 1 , t 2 ], where t 1 ≈ 0.2345 and t 2 ≈ 1.02. Since [r0 , 1] ⊂ [t 1 , t 2 ], the conclusion follows. By LPCF Theorem, we only need to prove the original inequality for b = c. Due to homogeneity, we may consider that b = c = 1. Thus, we need to show that a3 2 a+2 + ≥ , 2 ka + 1 a + k k+1 which is equivalent to the obvious inequality p (a − 1)2 (a − 2)2 ≥ 0. Thus, the proof is completed. In accordance with Remark 3, the equality holds for p a a = b = c. If k = 2 + 2 2, then the equality holds also for p = b = c (or any cyclic 2 permutation).

P 3.26. If a1 , a2 , . . . , an (n ≥ 3) are real numbers such that a1 , a2 , . . . , an ≥

−1 , n−2

a1 + a2 + · · · + an = n,

Partially Convex Function Method then

a12 a12 − a1 + 1

+

187

a22 a22 − a2 + 1

+ ··· +

an2 an2 − an + 1

≤ n. (Vasile Cirtoaje, 2012)

Solution. Write the inequality as f (a1 ) + f (a2 ) + · · · + f (an ) ≥ n f (s),

s=

a1 + a2 + · · · + an = 1, n

where   1−u −1 n2 − n − 1 f (u) = 2 , u∈I= , . u −u+1 n−2 n−2 From f 0 (u) =

u(u − 2) , (u2 − u + 1)2

f 00 (u) =

2(3u2 − u3 − 1) 2u2 (2 − u) + 2(u2 − 1) = , (u2 − u + 1)3 (u2 − u + 1)3

it follow that f is convex on [1, s0 ], decreasing for 1 ≤ u ≤ s0 and increasing for u ≥ s0 , where s0 = 2 ∈ I. Since f is not decreasing on Iu≤s0 , we can not apply RPCF Theorem in its original form. However, according to Remark 5, we may replace this condition with the condition ns − (n − 1)s0 ≤ inf I. Indeed, we have ns − (n − 1)s0 = n − 2(n − 1) = −n + 2 ≤

−1 = inf I. n−2

So, it suffices to show that f (x)+(n−1) f ( y) ≥ n f (1) for all x, y ∈ I such that x ≤ 1 ≤ y and x + (n − 1) y = n. According to Remark 1, we only need to show that h(x, y) ≥ 0, where f (u) − f (1) g(x) − g( y) g(u) = , h(x, y) = . u−1 x−y We have g(u) = h(x, y) =

(x 2

u2

−1 , −u+1

x + y −1 (n − 2)x + 1 = ≥ 0. 2 2 − x + 1)( y − y + 1) (n − 1)(x − x + 1)( y 2 − y + 1)

The proof is completed. The equality holds for a1 = a2 = · · · = an = 1, and also for −1 n−1 a1 = and a2 = a3 = · · · = an = (or any cyclic permutation). n−2 n−2

188

Vasile Cîrtoaje

P 3.27. If a1 , a2 , . . . , an (n ≥ 3) are nonzero real numbers such that a1 , a2 , . . . , an ≥ then

1 a12

+

1 a22

−n , n−2

+ ··· +

a1 + a2 + · · · + an = n,

1 1 1 1 ≥ + + ··· + . an2 a1 a2 an (Vasile Cirtoaje, 2012)

Solution. According to P 2.25-(a) in Volume 1, the inequality is true for n = 3. Assume further that n ≥ 4 and write the inequality as f (a1 ) + f (a2 ) + · · · + f (an ) ≥ n f (s), where

s=

a1 + a2 + · · · + an = 1, n

• ˜ 1 1 −n n(2n − 3) f (u) = 2 − , u ∈ I = , \ {0}. u u n−2 n−2

From f 0 (u) =

u−2 , u3

f 00 (u) =

2(3 − u) , u4

it follow that f is convex on [1, 3], decreasing for 1 ≤ u ≤ s0 and increasing for u ≥ s0 , where s0 = 2 ∈ I. We see that f is not decreasing on Iu≤s0 . Therefore, according to Remark 4 and Remark 5, we may replace this condition in RPCF Theorem with the condition ns − (n − 1)s0 ≤ inf I. Indeed, for n ≥ 4, we have ns − (n − 1)s0 = n − 2(n − 1) = −n + 2 ≤

−n = inf I. n−2

So, it suffices to show that f (x)+(n−1) f ( y) ≥ n f (1) for all x, y ∈ I such that x ≤ 1 ≤ y and x + (n − 1) y = n. According to Remark 1, we only need to show that h(x, y) ≥ 0, where f (u) − f (1) g(x) − g( y) g(u) = , h(x, y) = . u−1 x−y We have g(u) =

x+y −1 (n − 2)x + n , h(x, y) = 2 2 = ≥ 0. 2 u x y (n − 1)x 2 y 2

The proof is completed. In accordance with Remark 3, the equality holds for a1 = a2 = −n n · · · = an = 1, and also for a1 = and a2 = a3 = · · · = an = (or any cyclic n−2 n−2 permutation). Remark. Similarly, we can prove the following generalization.

Partially Convex Function Method • Let a1 , a2 , · · · , an ≥

189

−n such that a1 + a2 + · · · + an = n. If n ≥ 3 and k ≥ 0, then n−2 1 − a1 k + a12

+

1 − a2 k + a22

+ ··· +

1 − an ≥ 0, k + an2

−n with equality for a1 = a2 = · · · = an = 1, and also for a1 = and a2 = a3 = · · · = n−2 n (or any cyclic permutation). an = n−2

P 3.28. If a1 , a2 , . . . , an (n ≥ 3) are real numbers such that a1 , a2 , . . . , an ≥ then

−(3n − 2) , n−2

a1 + a2 + · · · + an = n,

1 − an 1 − a1 1 − a2 + + ··· + ≥ 0. 2 2 (1 + a1 ) (1 + a2 ) (1 + an )2 (Vasile Cîrtoaje, 2014)

Solution. According to P 2.25-(b) in Volume 1, the inequality is true for n = 3. Assume further that n ≥ 4 and write the inequality as f (a1 ) + f (a2 ) + · · · + f (an ) ≥ n f (s), where

s=

a1 + a2 + · · · + an = 1, n

  −(3n − 2) 4n2 − 7n + 2 1−u f (u) = , u∈I= , \ {−1}. (1 + u)2 n−2 n−2

From f 0 (u) =

u−3 , (u + 1)3

f 00 (u) =

2(5 − u) , (u + 1)4

it follow that f is convex on [1, 5], decreasing for 1 ≤ u ≤ s0 and increasing for u ≥ s0 , where s0 = 3 ∈ I. We see that f is not decreasing on Iu≤s0 . Therefore, according to Remark 4 and Remark 5, we may apply RPCF Theorem by replacing this condition with the sufficient condition ns − (n − 1)s0 ≤ inf I. Indeed, for n ≥ 4, we have ns − (n − 1)s0 = n − 3(n − 1) = −2n + 3 ≤

−(3n − 2) = inf I. n−2

Thus, it suffices to show that f (x) + (n − 1) f ( y) ≥ n f (1) for all x, y ∈ I such that x ≤ 1 ≤ y and x + (n − 1) y = n. According to Remark 1, we only need to show that h(x, y) ≥ 0, where g(u) =

f (u) − f (1) , u−1

h(x, y) =

g(x) − g( y) . x−y

190

Vasile Cîrtoaje

We have g(u) =

x + y +2 (n − 2)x + 3n − 2 −1 , h(x, y) = = ≥ 0. 2 2 2 (u + 1) (1 + x) (1 + y) (n − 1)(1 + x)2 (1 + y)2

The proof is completed. In accordance with Remark 3, the equality holds for a1 = a2 = −(3n − 2) n+2 · · · = an = 1, and also for a1 = and a2 = a3 = · · · = an = (or any cyclic n−2 n−2 permutation).

P 3.29. Let a1 , a2 , . . . , an be nonnegative real numbers such that a1 + a2 + · · · + an = n. If 2 n ≥ 3 and k ≥ 2 − , then n 1 − an 1 − a1 1 − a2 + + ··· + ≥ 0. (1 − ka1 )2 (1 − ka2 )2 (1 − kan )2 (Vasile Cîrtoaje, 2012) Solution. According to P 3.96 in Volume 1, the inequality is true for n = 3. Assume further that n ≥ 4 and write the inequality as f (a1 ) + f (a2 ) + · · · + f (an ) ≥ n f (s), where f (u) = From f 0 (u) =

s=

a1 + a2 + · · · + an = 1, n

1−u , u ∈ I = [0, 3] \ {1/k}. (1 − ku)2

−ku + 2k − 1 , (1 − ku)3

f 00 (u) =

2k(−ku + 3k − 2) , (1 − ku)4

it follow that f is convex on [1, s0 ], increasing on [0, 1/k) ∪ [s0 , 3] and decreasing on (1/k, s0 ], where 3n − 4 s0 = 2 − 1/k ≥ > 1. 2(n − 1) Since f is not decreasing on Iu≤s0 , we can not apply RPCF Theorem in its original form. However, according to Remark 5, we may replace this condition with the condition s1 ≤ inf I, where s1 = ns − (n − 1)s0 . Indeed, we have s1 ≤ n − (n − 1) ·

3n − 4 4−n = ≤ 0 = inf I. 2(n − 1) 2

Partially Convex Function Method

191

So, it suffices to show that f (x)+(n−1) f ( y) ≥ n f (1) for all x, y ∈ I such that x ≤ 1 ≤ y and x + (n − 1) y = n. According to Remark 1, we only need to show that h(x, y) ≥ 0, where f (u) − f (1) g(x) − g( y) g(u) = , h(x, y) = . u−1 x−y Since g(u) = h(x, y) =

−1 , (1 − ku)2

k[k(x + y) − 2] , (1 − k x)2 (1 − k y)2

we need to show that k(x + y) − 2 ≥ 0. Indeed, k(x + y) =

kn k[(n − 2)x + n] ≥ ≥ 2. n−1 n−1

2 The proof is completed. The equality holds for a1 = a2 = · · · = an = 1. If k = 2 − , n n then the equality also holds for a1 = 0 and a2 = a3 = · · · = an = (or any cyclic n−1 permutation).

192

Vasile Cîrtoaje

Chapter 4

Partially Convex Function Method for Ordered Variables 4.1

Theoretical Basis

Right Partially Convex Function Theorem for Ordered Variables (Vasile Cirtoaje, 2012). Let f be a function defined on a real interval I and convex on [s, s0 ], where s, s0 ∈ I, s < s0 . In addition, f (u) is decreasing on Iu≤s0 and min f (u) = f (s0 ). u∈I

The inequality f (a1 ) + f (a2 ) + · · · + f (an ) ≥ n f

a + a + ··· + a  1 2 n n

holds for all a1 , a2 , . . . , an ∈ I such that a1 + a2 + · · · + an = ns and at least n − m of a1 , a2 , . . . , an are smaller than or equal to s if and only if f (x) + m f ( y) ≥ (1 + m) f (s) for all x, y ∈ I such that x ≤ s ≤ y and x + m y = (1 + m)s. Proof. The necessity is obvious. By Lemma from section 3, to prove the sufficiency, it suffices to consider that a1 , a2 , . . . , an ∈ J, where J = Iu≤s0 . Because f (u) is convex on Ju≥s , the desired inequality follows from HCF-OV Theorem applied to the interval J. Similarly, we can prove Left Partially Convex Function Theorem for Ordered Variables (Vasile Cirtoaje, 2012). 193

194

Vasile Cîrtoaje

Left Partially Convex Function Theorem for Ordered Variables. Let f be a function defined on a real interval I and convex on [s0 , s], where s0 , s ∈ I, s0 < s. In addition, f (u) is increasing on Iu≥s0 and satisfies min = f (s0 ). u∈I

The inequality f (a1 ) + f (a2 ) + · · · + f (an ) ≥ n f

a + a + ··· + a  1 2 n n

holds for all a1 , a2 , · · · , an ∈ I such that a1 + a2 + · · · + an = ns and at least n − m of a1 , a2 , . . . , an are greater than or equal to s if and only if f (x) + m f ( y) ≥ (1 + m) f (s) for all x, y ∈ I such that x ≥ s ≥ y and x + m y = (1 + m)s. Remark 1. Let us denote g(u) =

f (u) − f (s) , u−s

h(x, y) =

g(x) − g( y) . x−y

We may replace the hypothesis condition in RPCF-OV Theorem and LPCF-OV Theorem, namely f (x) + m f ( y) ≥ (1 + m) f (s), by the condition h(x, y) ≥ 0 for all x, y ∈ I such that x + m y = (1 + m)s. Remark 2. Assume that f is differentiable on I, and let H(x, y) =

f 0 (x) − f 0 ( y) . x−y

Then, the desired inequality of Jensen’s type in RPCF-OV Theorem and LPCF-OV Theorem holds true by replacing the hypothesis f (x) + m f ( y) ≥ (1 + m) f (s) with the more restrictive condition H(x, y) ≥ 0 for all x, y ∈ I such that x + m y = (1 + m)s. Remark 3. RPCF-OV Theorem is a generalization of LPCF Theorem, because the last theorem can be obtained from the first theorem for m = n − 1. Similarly, LPCF-OV Theorem is a generalization of LPCF Theorem.

PCF Method for Ordered Variables

195

Remark 4. If a1 ≥ · · · ≥ am ≥ s ≥ am+1 ≥ · · · ≥ an , then at least n − m of a1 , a2 , . . . , an are smaller than or equal to s. Also, if a1 ≥ · · · ≥ an−m ≥ s ≥ an−m+1 ≥ · · · ≥ an , then at least n − m of a1 , a2 , . . . , an are greater than or equal to s. Thus, from RPCF-OV Theorem and LPCF-OV Theorem, we get the following corollaries. RPCF-OV Corollary. Let f be a function defined on a real interval I and convex on [s, s0 ], where s, s0 ∈ I, s < s0 . In addition, f (u) is decreasing on Iu≤s0 and min f (u) = f (s0 ). u∈I

The inequality f (a1 ) + f (a2 ) + · · · + f (an ) ≥ n f

a + a + ··· + a  1 2 n n

holds for all a1 , a2 , . . . , an ∈ I satisfying a1 ≥ · · · ≥ am ≥ s ≥ am+1 ≥ · · · ≥ an ,

a1 + a2 + · · · + an = ns,

if and only if f (x) + m f ( y) ≥ (1 + m) f (s) for all x, y ∈ I such that x ≤ s ≤ y and x + m y = (1 + m)s. LPCF-OV Corollary. Let f be a function defined on a real interval I and convex on [s0 , s], where s0 , s ∈ I, s0 < s. In addition, f (u) is increasing on Iu≥s0 and satisfies min f (u) = f (s0 ). u∈I

The inequality f (a1 ) + f (a2 ) + · · · + f (an ) ≥ n f

a + a + ··· + a  1 2 n n

holds for all a1 , a2 , . . . , an ∈ I satisfying a1 ≤ · · · ≤ am ≤ s ≤ am+1 ≤ · · · ≤ an ,

a1 + a2 + · · · + an = ns,

if and only if f (x) + m f ( y) ≥ (1 + m) f (s) for all x, y ∈ I such that x ≥ s ≥ y and x + m y = (1 + m)s.

196

Vasile Cîrtoaje

Remark 5. The inequality in RPCF-OV Corollary becomes an equality for a1 = a2 = · · · = an = s and also for a1 = · · · = am = y,

am+1 = · · · = an−1 = s,

an = x (x < y),

where x, y ∈ I satisfy the equations x + m y = (1 + m)s,

f (x) + m f ( y) = (1 + m) f (s).

For x 6= y, these equations are equivalent to x + m y = (1 + m)s, h(x, y) = 0. Remark 6. The inequality in LPCF-OV Corollary becomes an equality for a1 = a2 = · · · = an = s and also for a1 = x,

a2 = · · · = an−m = s,

an−m+1 = · · · = an = y (x > y),

where x, y ∈ I satisfy the equations x + m y = (1 + m)s,

f (x) + m f ( y) = (1 + m) f (s).

For x 6= y, these equations are equivalent to x + m y = (1 + m)s, h(x, y) = 0. Remark 7. RPCF-OV Theorem and RPCF-OV Corollar are also valid in the case when I = [a, b] \ {u0 } or I = (a, b) \ {u0 }, where a, b, u0 are real numbers such that a < s < s0 < u0 < b. Similarly, LPCF Theorem is also valid in the case when I = [a, b]\{u0 } or I = (a, b)\{u0 }, where a, b, u0 are real numbers such that a < u0 < s0 < s < b.

PCF Method for Ordered Variables

4.2

197

Applications

4.1. If a, b, c, d are real numbers such that a ≤ 1 ≤ b ≤ c ≤ d, then

a + b + c + d = 4,

b c d a + 2 + 2 + 2 ≤ 1. + 1 3b + 1 3c + 1 3d + 1

3a2

4.2. If a, b, c, d are real numbers such that a ≥ b ≥ 1 ≥ c ≥ d, then

a + b + c + d = 4,

16b − 5 16c − 5 16d − 5 4 16a − 5 + + + ≤ . 32a2 + 1 32b2 + 1 32c 2 + 1 32d 2 + 1 3

4.3. If a, b, c, d, e are real numbers such that a ≥ b ≥ 1 ≥ c ≥ d ≥ e, then

a + b + c + d + e = 5,

18a − 5 18b − 5 18c − 5 18d − 5 18e − 5 + + + + ≤ 5. 2 2 2 2 12a + 1 12b + 1 12c + 1 12d + 1 12e2 + 1

4.4. If a, b, c, d, e are real numbers such that a ≥ b ≥ 1 ≥ c ≥ d ≥ e, then

a + b + c + d + e = 5,

a(a − 1) b(b − 1) c(c − 1) d(d − 1) e(e − 1) + + 2 + + 2 ≥ 0. 3a2 + 4 3b2 + 4 3c + 4 3d 2 + 4 3e + 4

4.5. Let a1 , a2 , . . . , a2n 6= −k be real numbers such that a1 ≥ · · · ≥ an ≥ 1 ≥ an+1 ≥ · · · ≥ a2n , If k ≥

a1 + a2 + · · · + a2n = 2n.

n+1 p , then 2 n a2n (a2n − 1) a1 (a1 − 1) a2 (a2 − 1) + + ··· + ≥ 0. 2 2 (a1 + k) (a2 + k) (a2n + k)2

198

Vasile Cîrtoaje

4.6. Let a1 , a2 , . . . , a2n 6= −k be real numbers such that a1 + a2 + · · · + a2n = 2n.

a1 ≥ · · · ≥ an ≥ 1 ≥ an+1 ≥ · · · ≥ a2n , n+1 If k ≥ 1 + p , then n a12 − 1 (a1 + k)2

+

a22 − 1 (a2 + k)2

+ ··· +

2 a2n −1

(a2n + k)2

≥ 0.

4.7. If a1 , a2 , · · · , an are positive real numbers such that a1 ≥ 1 ≥ a2 ≥ · · · ≥ an , then

2/a1

a1

2/a2

+ a2

a1 + a2 + · · · + an = n,

+ · · · + an2/an ≤ n.

4.8. If a1 , a2 , · · · , a11 are real numbers such that a1 ≥ a2 ≥ 1 ≥ a3 ≥ · · · ≥ a11 ,

a1 + a2 + · · · + a11 = 11,

then 2 ) ≥ 1. (1 − a1 + a12 )(1 − a2 + a22 ) · · · (1 − a11 + a11

4.9. If a1 , a2 , · · · , a8 are nonzero real numbers such that a1 ≥ a2 ≥ a3 ≥ a4 ≥ 1 ≥ a5 ≥ a6 ≥ a7 ≥ a8 , then

 5

1 a12

+

1 a22

+ ··· +

1 a82



a1 + a2 + · · · + a8 = 8,

‹ 1 1 1 + 72 ≥ 14 + + ··· + . a1 a2 a8 

4.10. If a, b, c, d are positive real numbers such that a ≥ b ≥ 1 ≥ c ≥ d, then

a bcd = 1,

7 − 6a 7 − 6b 7 − 6c 7 − 6d 4 + + + ≥ . 2 2 2 2 2+a 2+ b 2+c 2+d 3

PCF Method for Ordered Variables

199

4.11. If a, b, c are positive real numbers such that a ≥ 1 ≥ b ≥ c, then

a bc = 1,

7 − 4a 7 − 4b 7 − 4c + + ≥ 3. 2 + a2 2 + b2 2 + c 2

4.12. Let a1 , a2 , · · · , an be positive real numbers such that a1 ≥ 1 ≥ a2 ≥ · · · ≥ an ,

a1 a2 · · · an = 1.

If p, q ≥ 0 such that p + 3q ≥ 1, then 1 − a1 1+

pa1 + qa12

+

1 − a2 1+

pa2 + qa22

+ ··· +

1 − an ≥ 0. 1 + pan + qan2

200

Vasile Cîrtoaje

PCF Method for Ordered Variables

4.3

201

Solutions

P 4.1. If a, b, c, d are real numbers such that a ≤ 1 ≤ b ≤ c ≤ d, then

a + b + c + d = 4,

b c d a + 2 + 2 + 2 ≤ 1. + 1 3b + 1 3c + 1 3d + 1

3a2

Solution. Write the inequality as f (a) + f (b) + f (c) + f (d) ≥ 4 f (s), where f (u) =

−u , 3u2 + 1

From

s=

a+b+c+d = 1, 4

u ∈ R.

3u2 − 1 , (3u2 + 1)2

f 0 (u) =

it follows p that f is increasing on (−∞, −s0 ]∪[s0 , ∞) and decreasing on [−s0 , s0 ], where s0 = 1/ 3. Since lim f (u) = 0 u→−∞

and f (s0 ) < 0, it follows that

min f (u) = f (s0 ). u∈R

From f 00 (u) =

18u(1 − u2 ) , (3u2 + 1)3

it follows that f is convex on [0, 1], hence on [s0 , 1]. Therefore, we apply LPCF-OV Theorem or LPCF-OV Corollary for n = 4 and m = 1. We only need to show that f (x) + f ( y) ≥ 2 f (1) for all real x, y such that x + y = 2. Using Remark 1, it suffices to prove that h(x, y) ≥ 0, where h(x, y) =

g(x) − g( y) , x−y

Indeed, we have g(u) = h(x, y) =

g(u) =

f (u) − f (1) . u−1

3u − 1 , 4(3u2 + 1)

3(1 + x + y − 3x y) 9(1 − x y) = ≥ 0, 2 2 2 4(3x + 1)(3 y + 1) 4(3x + 1)(3 y 2 + 1)

202

Vasile Cîrtoaje

since 4(1 − x y) = (x + y)2 − 4x y = (x − y)2 ≥ 0. Thus, the proof is completed. The equality holds for a = b = c = d = 1. Remark. Similarly, we can prove the following generalization. • If a1 , a2 , . . . , an are real numbers such that a1 + a2 + · · · + an = n,

a1 ≤ 1 ≤ a2 ≤ · · · ≤ an , then

a1 3a12

+1

+

a2 3a22

+1

+ ··· +

an n ≤ , 3an2 + 1 4

with equality for a1 = a2 = · · · = an = 1.

P 4.2. If a, b, c, d are real numbers such that a ≥ b ≥ 1 ≥ c ≥ d, then

a + b + c + d = 4,

16a − 5 16b − 5 16c − 5 16d − 5 4 + + + ≤ . 2 2 2 2 32a + 1 32b + 1 32c + 1 32d + 1 3 (Vasile Cîrtoaje, 2012)

Solution. Write the inequality as f (a) + f (b) + f (c) + f (d) ≥ 4 f (s),

s=

a+b+c+d = 1, 4

where

5 − 16u , u ∈ R. 32u2 + 1 As shown in the proof of P 3.1, f is convex on [s0 , 1], increasing for u ≥ s0 and f (u) =

min f (u) = f (s0 ), u∈R

where

p 5 + 33 s0 = ≈ 0.6715. 16 Therefore, we may apply LPCF-OV Theorem or LPCF-OV Corollary for n = 4 and m = 2. We only need to show that f (x) + 2 f ( y) ≥ 3 f (1) for all real x, y such that x + 2 y = 3. Using Remark 1, it suffices to prove that h(x, y) ≥ 0, where h(x, y) =

g(x) − g( y) , x−y

g(u) =

f (u) − f (1) . u−1

PCF Method for Ordered Variables

203

Indeed, we have g(u) = h(x, y) =

32(2u − 1) , 3(32u2 + 1)

64(1 + 16x + 16 y − 32x y) 64(4x − 5)2 = ≥ 0. 3(32x 2 + 1)(32 y 2 + 1) 3(32x 2 + 1)(32 y 2 + 1)

Thus, the proof is completed. From x + 2 y = 3 and h(x, y) = 0, we get x = 5/4 and, y = 7/8. Therefore, in accordance with Remark 6, the equality holds for a = b = c = d = 1, and also for a = 5/4, b = 1 and c = d = 7/8. Remark. Similarly, we can prove the following generalization. • If a1 , a2 , . . . , an (n ≥ 3) are real numbers such that a1 ≥ · · · ≥ an−2 ≥ 1 ≥ an−1 ≥ an , then

16a1 − 5 32a12

+1

+

16a2 − 5 32a22

+1

+ ··· +

a1 + a2 + · · · + an = n, 16an − 5 n ≤ , 2 32an + 1 3

with equality for a1 = a2 = · · · = an = 1, and also for a1 = 5/4, a2 = · · · = an−2 = 1 and an−1 = an = 7/8.

P 4.3. If a, b, c, d, e are real numbers such that a ≥ b ≥ 1 ≥ c ≥ d ≥ e, then

a + b + c + d + e = 5,

18a − 5 18b − 5 18c − 5 18d − 5 18e − 5 + + + + ≤ 5. 2 2 2 2 12a + 1 12b + 1 12c + 1 12d + 1 12e2 + 1 (Vasile Cîrtoaje, 2012)

Solution. Write the inequality as f (a) + f (b) + f (c) + f (d) + f (e) ≥ 5 f (s), where f (u) =

5 − 18u , 12u2 + 1

s=

a+b+c+d+e = 1, 5

u ∈ R.

As shown in the proof of P 3.2, f is convex on [s0 , 1], increasing for u ≥ s0 and min f (u) = f (s0 ), u∈R

204

Vasile Cîrtoaje

where

p 5 + 52 s0 = ≈ 0.678. 18

Therefore, we may apply LPCF-OV Theorem or LPCF-OV Corollary for n = 5 and m = 3. We only need to show that f (x) + 3 f ( y) ≥ 4 f (1) for all real x, y such that x + 3 y = 4. Using Remark 1, it suffices to prove that h(x, y) ≥ 0, where h(x, y) =

g(x) − g( y) , x−y

g(u) =

f (u) − f (1) . u−1

Indeed, we have g(u) = h(x, y) =

6(2u − 1) , 12u2 + 1

12(1 + 6x + 6 y − 12x y) 12(2x − 3)2 = ≥ 0. (12x 2 + 1)(12 y 2 + 1) (12x 2 + 1)(12 y 2 + 1)

Thus, the proof is completed. From x + 3 y = 4 and h(x, y) = 0, we get x = 3/2 and, y = 5/6. Therefore, in accordance with Remark 6, the equality holds for a = b = c = d = e = 1, and also for a = 3/2, b = 1 and c = d = e = 5/6. Remark. Similarly, we can prove the following generalization. • If a1 , a2 , . . . , an (n ≥ 4) are real numbers such that a1 ≥ · · · ≥ an−3 ≥ 1 ≥ an−2 ≥ an−1 ≥ an , then

18a1 − 5 12a12

+1

+

18a2 − 5 12a22

+1

+ ··· +

a1 + a2 + · · · + an = n, 18an − 5 ≤ n, 12an2 + 1

with equality for a1 = a2 = · · · = an = 1, and also for a1 = 3/2, a2 = · · · = an−3 = 1 and an−2 = an−1 = an = 5/6.

P 4.4. If a, b, c, d, e are real numbers such that a ≥ b ≥ 1 ≥ c ≥ d ≥ e,

a + b + c + d + e = 5,

then a(a − 1) b(b − 1) c(c − 1) d(d − 1) e(e − 1) + + 2 + + 2 ≥ 0. 3a2 + 4 3b2 + 4 3c + 4 3d 2 + 4 3e + 4 (Vasile Cîrtoaje, 2012)

PCF Method for Ordered Variables

205

Solution. Write the inequality as f (a) + f (b) + f (c) + f (d) + f (e) ≥ 5 f (s),

s=

a+b+c+d+e = 1, 5

where

u2 − u , u ∈ R. 3u2 + 4 As shown in the proof of P 3.5, f is convex on [s0 , 1], increasing for u ≥ s0 and f (u) =

min f (u) = f (s0 ), u∈R

where

p −4 + 2 7 s0 = ≈ 0.43. 3 Therefore, we may apply LPCF-OV Theorem or LPCF-OV Corollary for n = 5 and m = 3. We only need to show that f (x) + 3 f ( y) ≥ 4 f (1) for all real x, y such that x + 3 y = 4. Using Remark 1, it suffices to prove that h(x, y) ≥ 0, where h(x, y) =

g(x) − g( y) , x−y

g(u) =

f (u) − f (1) . u−1

Indeed, we have g(u) = h(x, y) =

u) , +4

3u2

4 − 3x y (x − 2)2 = ≥ 0. (3x 2 + 4)(3 y 2 + 4) (12x 2 + 1)(12 y 2 + 1)

Thus, the proof is completed. From x + 3 y = 4 and h(x, y) = 0, we get x = 2 and, y = 2/3. Therefore, in accordance with Remark 6, the equality holds for a = b = c = d = e = 1, and also for a = 2, b = 1, c = d = e = 2/3.

Remark. Similarly, we can prove the following generalizations. • If a1 , a2 , . . . , an (n ≥ 4) are real numbers such that a1 ≥ · · · ≥ an−3 ≥ 1 ≥ an−2 ≥ an−1 ≥ an , then

a1 (a1 − 1) 3a12

+4

+

a2 (a2 − 1) 3a22

+4

+ ··· +

a1 + a2 + · · · + an = n, an (an − 1) ≥ 0, 3an2 + 4

206

Vasile Cîrtoaje

with equality for a1 = a2 = · · · = an = 1, and also for a1 = 2, a2 = · · · = an−3 = 1 and an−2 = an−1 = an = 2/3. • If a1 , a2 , . . . , an (n ≥ 3) are real numbers such that a1 ≥ a2 ≥ 1 ≥ a3 ≥ · · · ≥ an ,

a1 + a2 + · · · + an = n,

then a1 (a1 − 1) 4(n − 2)a12

+ (n − 1)2

+

a2 (a2 − 1) 4(n − 2)a22

+ (n − 1)2

+ ··· +

an (an − 1) ≥ 0, 4(n − 2)an2 + (n − 1)2

with equality for a1 = a2 = · · · = an = 1, and also for a1 = an =

n−1 . 2(n − 2)

n−1 , a2 = 1 and a3 = · · · = 2

P 4.5. Let a1 , a2 , . . . , a2n 6= −k be real numbers such that a1 ≥ · · · ≥ an ≥ 1 ≥ an+1 ≥ · · · ≥ a2n , If k ≥

a1 + a2 + · · · + a2n = 2n.

n+1 p , then 2 n a2n (a2n − 1) a1 (a1 − 1) a2 (a2 − 1) + + ··· + ≥ 0. 2 2 (a1 + k) (a2 + k) (a2n + k)2 (Vasile Cîrtoaje, 2012)

Solution. Write the inequality as f (a1 ) + f (a2 ) + · · · + f (a2n ) ≥ 2n f (s), where f (u) =

s=

a1 + a2 + · · · + a2n = 1, 2n

u(u − 1) , u ∈ I = R \ {−k}. (u + k)2

As shown in the proof of P 3.6, f is convex on [s0 , 1], increasing for u ≥ s0 and min f (u) = f (s0 ), u∈I

where s0 =

k < 1. 2k + 1

PCF Method for Ordered Variables

207

Therefore, we may apply LPCF-OV Theorem or LPCF-OV Corollary for 2n real numbers and m = n. We only need to show that f (x) + n f ( y) ≥ (n + 1) f (1) for all real x, y such that x + n y = n + 1. Using Remark 1, it suffices to prove that h(x, y) ≥ 0, where h(x, y) =

g(x) − g( y) , x−y

Indeed, we have g(u) = h(x, y) =

g(u) =

f (u) − f (1) . u−1

u , (u + k)2

k2 − x y ≥ 0, (x + k)2 ( y + k)2

because

[2n y − n − 1]2 (n + 1)2 −xy = ≥ 0. 4n 4n n+1 The equality holds for a1 = a2 = · · · = an = 1. If k = p , then the equality holds also 2 n n+1 n+1 for a1 = , a2 = · · · = an = 1 and an+1 = · · · = a2n = . 2 2n k2 − x y ≥

P 4.6. Let a1 , a2 , . . . , a2n 6= −k be real numbers such that a1 ≥ · · · ≥ an ≥ 1 ≥ an+1 ≥ · · · ≥ a2n ,

a1 + a2 + · · · + a2n = 2n.

n+1 If k ≥ 1 + p , then n a12 − 1 (a1 + k)2

a22 − 1

+

(a2 + k)2

+ ··· +

2 a2n −1

(a2n + k)2

≥ 0. (Vasile Cîrtoaje, 2012)

Solution. Write the inequality as f (a1 ) + f (a2 ) + · · · + f (a2n ) ≥ 2n f (s), where f (u) =

s=

a1 + a2 + · · · + a2n = 1, 2n

u2 − 1 , u ∈ I = R \ {−k}. (u + k)2

As shown in the proof of P 3.7, f is convex on [s0 , 1], increasing for u ≥ s0 and min f (u) = f (s0 ), u∈I

208

Vasile Cîrtoaje

where

−1 ∈ (−1, 0). k Therefore, we may apply LPCF-OV Theorem or LPCF-OV Corollary for 2n real numbers and m = n. We only need to show that f (x) + n f ( y) ≥ (n + 1) f (1) for all real x, y such that x + n y = n + 1. Using Remark 1, it suffices to prove that h(x, y) ≥ 0, where s0 =

h(x, y) =

g(x) − g( y) , x−y

Indeed, we have g(u) = h(x, y) =

g(u) =

f (u) − f (1) . u−1

u+1 , (u + k)2

(k − 1)2 − 1 − x − y − x y ≥ 0, (x + k)2 ( y + k)2

because (k − 1)2 − 1 − x − y − x y ≥

(n y − 1)2 (n + 1)2 −1− x − y − xy = ≥ 0. n n

n+1 The equality holds for a1 = a2 = · · · = an = 1. If k = 1 + p , then the equality holds n 1 also for a1 = n, a2 = · · · = an = 1 and an+1 = · · · = a2n = . n

P 4.7. If a1 , a2 , · · · , an are positive real numbers such that a1 + a2 + · · · + an = n,

a1 ≥ 1 ≥ a2 ≥ · · · ≥ an , then

2/a1

a1

2/a2

+ a2

+ · · · + an2/an ≤ n. (Vasile Cîrtoaje, 2012)

Solution. Rewrite the desired inequality as f (a1 ) + f (a2 ) + · · · + f (an ) ≥ n f (s),

s=

a1 + a2 + · · · + an = 1, n

where f (u) = −u2/u , We have

u ∈ I = (0, n). 2

f 0 (u) = 2u u −2 (ln u − 1),

PCF Method for Ordered Variables

209

2

f 00 (u) = 2u u −4 [u + 2(1 − ln u)(u − 1 + ln u)]. From the expression of f 0 , it follows that f is decreasing on (0, s0 ] and increasing on [s0 , n), where s0 = e. In addition, we claim that f is convex on [1, s0 ]. Indeed, since 1 − ln u ≥ 0 and u − 1 + ln u ≥ u − 1 ≥ 0, we have f 00 > 0 for u ∈ [1, s0 ]. Therefore, we may apply RPCF-OV Theorem or RPCF-OV Corollary for m = 1. We only need to show that f (x) + f ( y) ≥ 2 f (1) for all x, y > 0 such that x + y = 2. The inequality f (x) + f ( y) ≥ 2 f (1) is equivalent to x 2/x + y 2/ y ≤ 2, which is just the inequality in P 3.27 from Volume 2. The equality holds for a1 = a2 = · · · = an = 1.

P 4.8. If a1 , a2 , · · · , a11 are real numbers such that a1 ≥ a2 ≥ 1 ≥ a3 ≥ · · · ≥ a11 ,

a1 + a2 + · · · + a11 = 11,

then 2 ) ≥ 1. (1 − a1 + a12 )(1 − a2 + a22 ) · · · (1 − a11 + a11

(Vasile Cîrtoaje, 2012) Solution. Rewrite the desired inequality as f (a1 ) + f (a2 ) + · · · + f (a11 ) ≥ 11 f (s),

s=

a1 + a2 + · · · + a11 = 1, 11

where f (u) = ln(1 − u + u2 ), From f 0 (u) =

u ∈ R.

2u − 1 , 1 − u + u2

it follows that f is decreasing on (−∞, s0 ] and increasing on [s0 , ∞), where s0 = 1/2. Also, from 1 + 2u(1 − u) f 00 (u) = , (1 − u + u2 )2 it follows that f 00 (u) > 0 for u ∈ [s0 , 1], hence f is convex on [s0 , 1]. Therefore, we may apply LPCF-OV Theorem or LPCF-OV Corollary for n = 11 and m = 9. We only need to

210

Vasile Cîrtoaje

show that f (x) + 9 f ( y) ≥ 9 f (1) for all real x, y such that x + 9 y = 10. Using Remark 2, it suffices to prove that H(x, y) ≥ 0, where H(x, y) = Since H(x, y) =

f 0 (x) − f 0 ( y) . x−y

1 + x + y − 2x y (1 − x + x 2 )(1 − y + y 2 )

and 1 + x + y − 2x y = 18 y 2 − 8 y + 1 = 2 y 2 + (4 y − 1)2 > 0, the conclusion follows. The equality holds for a1 = a2 = · · · = a11 = 1. Remark. By replacing a1 , a2 , . . . , a11 respectively with 1 − a1 , 1 − a2 , . . . , 1 − a11 , we get the following statement. • If a1 , a2 , . . . , a11 are real numbers such that a1 ≤ a2 ≤ 0 ≤ a3 ≤ · · · ≤ a11 ,

a1 + a2 + · · · + a11 = 0,

then 2 ) ≥ 1, (1 − a1 + a12 )(1 − a2 + a22 ) · · · (1 − a11 + a11

with equality for a1 = a2 = · · · = an = 0.

P 4.9. If a1 , a2 , · · · , a8 are nonzero real numbers such that a1 ≥ a2 ≥ a3 ≥ a4 ≥ 1 ≥ a5 ≥ a6 ≥ a7 ≥ a8 , then

 5

1 a12

+

1 a22

+ ··· +

1



a82

+ 72 ≥ 14



a1 + a2 + · · · + a8 = 8,

‹ 1 1 1 + + ··· + . a1 a2 a8 (Vasile Cîrtoaje, 2012)

Solution. Write the desired inequality as f (a1 ) + f (a2 ) + · · · + f (a8 ) ≥ 8 f (s), s = where

a1 + a2 + · · · + a8 = 1, 8

5 14 − + 9, u ∈ I = R \ {0}. 2 u u As shown in the proof of P 3.16, f is convex on [s0 , 1], increasing for u ≥ s0 and f (u) =

min f (u) = f (s0 ), u∈I

PCF Method for Ordered Variables

211

where

5 . 7 Taking into account Remark 7, we may apply LPCF-OV Theorem or LPCF-OV Corollary for n = 8 and m = 4. We only need to show that f (x) + 4 f ( y) ≥ 5 f (1) for all x, y ∈ I such that x + 4 y = 5. Using Remark 1, it suffices to prove that h(x, y) ≥ 0, where s0 =

h(x, y) =

g(x) − g( y) , x−y

g(u) =

Indeed, we have g(u) = g(u) = h(x, y) =

f (u) − f (1) . u−1

5 9 − 2, u u

5x + 5 y − 9x y (6 y − 5)2 = ≥ 0. x2 y2 x2 y2

In accordance with Remark 6, the equality holds for a1 = a2 = · · · = a8 = 1, and also 5 5 for a1 = , a2 = a3 = a4 = 1 and a5 = a6 = a7 = a8 = (or any cyclic permutation). 3 6

P 4.10. If a, b, c, d are positive real numbers such that a ≥ b ≥ 1 ≥ c ≥ d, then

a bcd = 1,

7 − 6a 7 − 6b 7 − 6c 7 − 6d 4 + + + ≥ . 2 + a2 2 + b2 2 + c 2 2 + d 2 3 (Vasile Cîrtoaje, 2012)

Solution. Using the substitution a = e x , b = e y , c = ez , d = e w we need to show that f (x) + f ( y) + f (z) + f (w) ≥ 4 f (s), where x ≥ y ≥ 0 ≥ z ≥ w,

s=

x + y +z+w = 0, 4

7 − 6eu , u ∈ R. 2 + e2u As shown in the proof of P 3.17, f is convex on [0, s0 ], is decreasing on (−∞, s0 ] and increasing on [s0 , ∞), where s0 = ln 3. Therefore, we may apply RPCF-OV Theorem or f (u) =

212

Vasile Cîrtoaje

RPCF-OV Corollary for n = 4 and m = 2. We only need to show that f (x) + 2 f ( y) ≥ 3 f (0) for all real x, y such that x + 2 y = 0. That is, to prove that 7 − 6a 2(7 − 6b) + ≥1 2 + a2 2 + b2 for all a, b > 0 such that a b2 = 1. This is equivalent to (b − 1)2 (b − 2)2 (5b2 + 6bt + 3) ≥ 0, which is clearly true. In accordance with Remark 5, the equality holds for a = b = c = d = 1, and also for a = b = 2, c = 1 and d = 1/4.

P 4.11. If a, b, c are positive real numbers such that a ≥ 1 ≥ b ≥ c, then

a bc = 1,

7 − 4a 7 − 4b 7 − 4c + + ≥ 3. 2 + a2 2 + b2 2 + c 2 (Vasile Cîrtoaje, 2012)

Solution. Using the substitution a = e x , b = e y , c = ez , we need to show that f (x) + f ( y) + f (z) ≥ 3 f (s), where x ≥ 1 ≥ y ≥ z, f (u) = From f 0 (u) =

s=

x + y +z = 0, 3

7 − 4eu , u ∈ R. 2 + e2u

2eu (2eu + 1)(eu − 4) , (2 + e2u )2

it follows that f is decreasing on (−∞, s0 ] and increasing on [s0 , ∞), where s0 = ln 4. Also, we have 4t · h(t) f 00 (u) = , t = eu , (2 + t 2 )3 where h(t) = −t 4 + 7t 3 + 12t 2 − 14t − 4.

PCF Method for Ordered Variables

213

We will show that h(t) ≥ 0 for t ∈ [1, 4], hence f is convex on [0, s0 ]. Indeed, h(t) = (t − 1)[t 2 (−t + 6) + 18t + 4] ≥ 0. Therefore, we may apply RPCF-OV Theorem or RPCF-OV Corollary for n = 3 and m = 1. We only need to show that f (x) + f ( y) ≥ 2 f (0) for all real x, y such that x + y = 0. That is, to prove that 7 − 4a 7 − 4b + ≥2 2 + a2 2 + b2 for all a, b > 0 such that a b = 1. This is equivalent to (a − 1)4 ≥ 0. The equality holds for a = b = c = 1.

P 4.12. Let a1 , a2 , · · · , an be positive real numbers such that a1 a2 · · · an = 1.

a1 ≥ 1 ≥ a2 ≥ · · · ≥ an , If p, q ≥ 0 such that p + 3q ≥ 1, then 1 − a1 1+

pa1 + qa12

+

1 − a2 1+

pa2 + qa22

+ ··· +

1 − an ≥ 0. 1 + pan + qan2 (Vasile Cîrtoaje, 2012)

Solution. For q = 0, we need to show that p ≥ 1 involves 1 − an 1 − a1 1 − a2 + + ··· + ≥ 0. 1 + pa1 1 + pa2 1 + pan This is just the inequality from P 2.19. Consider next that q > 0. Using the substitutions ai = e x i for i = 1, 2, . . . , n, we need to show that f (x 1 ) + f (x 2 ) + · · · + f (x n ) ≥ n f (s), where x1 ≥ 1 ≥ x2 ≥ · · · ≥ x n, f (u) =

s=

x1 + x2 + · · · + x n = 0, n

1 − eu , u ∈ R. 1 + peu + qe2u

As shown in the proof of P 3.21, f is convex on [0, s0 ] if p + 3q − 1 ≥ 0, where v t p+1 s0 = ln r0 > 0, r0 = 1 + 1 + . q

214

Vasile Cîrtoaje

Clearly, the condition p+3q−1 ≥ 0 is satisfied by hypothesis. In addition, f is decreasing on (−∞, s0 ] and increasing on [s0 , ∞). Therefore, we may apply RPCF-OV Theorem or RPCF-OV Corollary for m = 1. We only need to show that f (x) + f ( y) ≥ 2 f (0) for all real x, y such that x + y = 0. That is, to prove that 1−a 1− b + ≥0 2 1 + pa + qa 1 + p b + q b2 for all a, b > 0 such that a b = 1. This is equivalent to (a − 1)2 [(p − 1)a + q(a2 + a + 1)] ≥ 0, which is true because (p − 1)a + q(a2 + a + 1) ≥ (p − 1)a + q(3a) = (p + 3q − 1)a ≥ 0. The equality holds for a1 = a2 = · · · = an = 1.

Chapter 5

Bibliography

215